Download as pdf or txt
Download as pdf or txt
You are on page 1of 167

SAMPLE -1

of LATEST CK-UW NOTES 2022


BLOCK 1-58

SYSTEM-WISE
Notes made by: Step1usmle59@gmail.com
https://www.reddit.com/u/USMLEe59/?
utm_source=share&utm_medium=ios_app
&utm_name=iossmf

Latest UW Notes for STEP2 CK - 2022


Features:
• Arranged system wise and then subject wise
• There are total 97 Blocks, Each BLOCK is made of 40 or some time 45 -50 different Questions hence
each BLOCK covers the NOTEs of those 40-50 questions,while the total 97 Blocks covers the whole
UWorld (3986 Qs)

• Custom BLOCK Id is mentioned with each BLOCK, so you can make Your own blocks in UWorld
according to the BLOCK id mentioned and read the respective BLOCK,s NOTES here

• Contains All UW tables edited and supplemented with UW Explanations


• All UW Diagrams/Pictures and algorithms annotated and simpli ed
• Some topics are supplemented with UPTODATE information,separate tables are there for these info so
that you can differentiate it form UW info

• Highlighted important info


• Most of the UW Tables are edited with related pictures and de cinet information added to the
tables from UW explanations

• These Notes will be updated regularly


• Notes of the Upcoming NEW Questions will be added on monthly basis

fi
fi


ENDOCRINOLOGY:
MEDICINE

BLOCK 9
Custom Id: 211426496


01)Painless Thyroiditis is associated with thyroid peroxidase autoantibodies


and is considered a variant of chronic lymphocytic (Hashimoto) thyroiditis. It is
characterized by a self-limited hyperthyroid phase due to release of preformed thyroid
hormone, followed by a hypothyroid phase or return to a euthyroid state. It does not
require specific therapy, but a beta blocker may be prescribed to control symptoms due
to hyperthyroidism (eg, palpitations, tremulousness).
Prednisone is used to treat thyroid pain in patients with subacute (de Quervain)
thyroiditis who do not respond to nonsteroidal anti-inflammatory drugs.Prednisone is
also used in some patients with amiodarone-induced thyrotoxicosis.
Thyrotoxicosis with normal or ↑ RAIU Thyrotoxicosis with ↓ RAIU

• Painless (silent) thyroiditis


• Subacute (de Quervain) thyroiditis
• Graves disease • Amiodarone-induced thyroiditis
• Toxic multinodular goiter • Excessive dose (or surreptitious intake) of levothyroxine
• Toxic nodule • Struma ovarii
• Iodine-induced
• Extensive thyroid cancer metastasis

02)Diabetic Nephropathy (DN) is characterized by persistent albuminuria (≥30 mg/g)


and/or decreased glomerular ltration rate.The diagnosis is usually made clinically,
with renal biopsy reserved for patients with an unclear presentation. In the absence of
signs suggestive of another etiology(eg, casts in the urine, rapid progression of renal
dysfunction), the diagnosis can be presumed in patients who have persistent
albuminuria and/or decreased GFR (renal dysfunction) and either a prolonged history of
diabetes (≥5 years for type 1 diabetes) or proliferative diabetic retinopathy, which
correlates with the presence of DN.
03) Clinical features of osteomalacia
Osteomalacia is characterized by reduced mineralization of osteoid at bone-forming sites.
• Malabsorption
• Intestinal bypass surgery
Causes • Celiac disease
• Chronic liver disease
• Chronic kidney disease

• May be asymptomatic
• Bone pain & muscle weakness
Symptoms/signs • Muscle cramps
• Difficulty walking, waddling gait

• ↑ Alkaline phosphatase, ↑ PTH


• ↓ Serum calcium & phosphorus(very low as compare to calcium ,bcz of urinary
loss by increased PTH), ↓ urinary calcium
• ↓ 25 OH-D levels
Diagnosis
• X-rays may show thinning of cortex with reduced bone density
• Bilateral & Symmetric Pseudofractures (cortical infarction/insufficiency
fractures traversing part of the bone-Looser zones) are characteristic radiologic
findings.
Although 25-hydroxyvitamin D levels are low (decreased total body stores), levels of 1,25-dihydroxyvitamin D are
variable because it is produced in the kidneys in response to PTH.

fi

04) Secondary Hypogonadism can be due to mass lesions in the hypothalamus


or pituitary, hyperprolactinemia, long-term use of glucocorticoids or opiates, or severe
systemic illness. Secondary hypogonadism is also more common in individuals with
obesity or type 2 diabetes mellitus. Patients should have measurement of serum
prolactin and screening for other pituitary hormone deficiencies(e.g TSH). Pituitary
imaging is indicated for patients with elevated prolactin, mass-effect symptoms, very
low testosterone levels(<150ng/dl), or disruptions in other pituitary hormones.

05) Cushing Syndrome with Associated hypokalemia suggests severe


hypercortisolism and is most often seen with ectopic ACTH-producing tumors.
Initial testing for suspected hypercortisolism can include a 24-hour assay for urine free
cortisol, a late-night salivary cortisol measurement, or a low-dose dexamethasone suppression test.
Primary versus secondary adrenal insufficiency

Primary Secondary

Mechanism • Destruction of bilateral adrenal • Disruption of hypothalamic-pituitary


cortex axis
Possible • Autoimmune adrenalitis • Chronic glucocorticoid therapy
etiologies • Infection, malignancy • Infiltrative disease

Cortisol ↓ ↓

Aldosterone ↓ Normal

Adrenal ↓ ↓
Androgen
ACTH ↑ ↓

• More severe symptoms • Less severe symptoms


• Hypovolemia • Euvolemia
Clinical features • Hyperkalemia, hyponatremia • Minimal electrolyte disturbance
• Hyperpigmentation • No hyperpigmentation
• Hyperchloremic Acidosis • No Hyperchloremic Acidosis
Lymphocytic hypophysitis (lymphocytic infiltration of the pituitary gland) and Sheehan syndrome (pituitary
infarction due to hemorrhage/hypotension during labor) are common causes of postpartum secondary AI.

Clinical manifestations of celiac disease


• Diarrhea, ± steatorrhea, weight loss


• Abdominal pain
Gastrointestinal
• Flatulence/bloating
• Late manifestations: ulcerative jejunitis, enteropathy-associated T-cell
lymphoma
• Dermatitis herpetiformis
Mucocutaneous
• Atrophic glossitis

• Vitamin D deficiency
Endocrine
• Secondary hyperparathyroidism

• Osteomalacia/osteoporosis (adults)
Bone disorders
• Rickets (children)

Hematologic • Iron deficiency anemia

• Peripheral neuropathy
Neuropsychiatric
• Depression/anxiety

08) Any mass lesion in the pituitary gland can cause a mild elevation in prolactin due
to disruption of the normal inhibitory dopaminergic regulation of prolactin secretion.
However, significant elevations in the prolactin level (eg, serum prolactin level >200
ng/mL or repeat level >100 ng/mL) suggest a prolactin-secreting tumor prolactinoma.

Multiple endocrine neoplasia type 1

Manifestation Clinical features

Pituitary adenomas • Secretion of prolactin, growth hormone, ACTH (or “nonfunctioning”


(10%-20%) tumors)
• Mass effects (eg, headache, visual field defects)

• Multiple parathyroid adenomas or parathyroid hyperplasia


Primary hyperparathyroidism
• Hypercalcemia (eg, polyuria, kidney stones, decreased bone
(>90%)
density)

• Gastrinoma - recurrent peptic ulcers


Pancreatic/gastrointestinal • Insulinoma - hypoglycemia
neuroendocrine tumors • VIPoma - secretory diarrhea, hypokalemia, hypochlorhydria
(60%-70%) • Glucagonoma - weight loss, necrolytic migratory erythema,
hyperglycemia

10) Polyuria with high serum osmolality are seen in Diabetes Insipidus and
Diabetes mellitus (DM) complicated by marked hyperglycemia (eg, diabetic

ketoacidosis). Urine osmolality and speci c gravity can be used to narrow the cause of
suspected polyuria.As DM causes osmotic diuresis (eg,glucosuria)with a high urinary
solute concentration,the urine specific gravity and urine osmolality (eg, typically >600
mOsm/kg H2O) will be high in DM. DI are characterized by impaired water
reabsorption and concentration of urine,hence urine osmolality and urine speci c
gravity (eg <1.006) will be low in diabetes insipidus.
Causes of polyuria & dilute urine
Primary polydipsia Central DI Nephrogenic DI

• ADH independent (excessive • ADH deficiency (CNS • ADH resistance (renal


Cause water intake) pathology) disease)

• Idiopathic • Chronic lithium use


• Antipsychotics (dry mouth) • Trauma • Hypercalcemia
Etiology • Psychiatric conditions • Pituitary surgery • Hereditary (AVPR2 &
• Ischemic encephalopathy aquaporin 2 mutations)

Results
of water • High urine osmolality • Low urine osmolality • Low urine osmolality
deprivation

Response to
• No change • Increased urine osmolality • No change
desmopressin

11)Fibrates (eg, gemfibrozil) are effective in reducing triglyceride levels; however,


statins have greater proven cardiovascular benefit and are the preferred first-line therapy
for prevention of ASCVD in patients with mild hypertriglyceridemia (ie, 150-500 mg/
dL). Fibrates are indicated in patients with severe hypertriglyceridemia (ie, >1000 mg/
dL) to reduce the risk of acute pancreatitis.

Mild elevations (eg;45ng/mL) of prolactin in the context of otherwise normal pituitary


function are typically due to medications (eg, certain antipsychotic drugs). Other causes
include hypothyroidism and renal failure.

Letrozole, an aromatase inhibitor, is administered for ovulation induction in


PCOS to treat infertility. It does not regulate menses.



fi

fi

12)Most patients with DKA have normal or elevated serum potassium values on laboratory testing,
but actually have a total body potassium de cit.  The overall potassium de cit is due to urinary
potassium losses from osmotic diuresis, elimination of ketoacid anions as potassium
salts (seen only in DKA), and secondary hyperaldosteronism resulting from volume
contraction.Administration of insulin during treatment drives potassium into cells and can rapidly
lead to hypokalemia.  For this reason, potassium should be monitored closely and added to
intravenous uids whenever serum potassium is <5.3 mEq/L.
13)In patients with normal hypothalamic-pituitary feedback,displacement of T4 from its binding
proteins leads to decreased thyroid hormone production and lower total T4 levels with normal free
hormone levels.  Medications with this effect include salicylates, furosemide, and heparin.
Resistance to thyroid hormone is a rare inherited disorder caused by defects in the thyroid
hormone receptor.  Patients can have elevated T4 levels with a normal or increased TSH, but
most have a goiter, and this condition is usually apparent in childhood.
Conditions that alter TBG concentration
Increased TBG Decreased TBG
• Androgenic hormones
• High-dose glucocorticoids/
• Estrogens (eg, pregnancy, OCs, HRT) &
hypercortisolism
estrogenic medications (eg, tamoxifen)
• Hypoproteinemia (eg, nephrotic
• Acute hepatitis
syndrome, starvation)
• Chronic liver disease
Estrogen increases the level of T4-binding globulin. Total thyroid hormone levels are elevated, but patients
with normal hypothalamic-pituitary-thyroid function maintain a euthyroid state and normal TSH level.

Pheochromocytoma
• Classic triad: episodic headache, sweating & tachycardia
Indications for testing • Resistant HTN or HTN accompanied by unexplained ↑ glucose
• Family history or familial syndrome (eg, MEN2, NF1, VHL)
• Urine or plasma metanephrine levels(initial evaluation)
Diagnostic approach • Confirmatory abdominal imaging for ↑ metanephrines;
(Abdominal CT scan or MRI and/or positron emission tomography to confirm
the presence and location of a pheochromocytoma.)

Notable features • 10% bilateral, 10% extraadrenal, 10% malignant


• Preoperative alpha blockade prior to beta blockade
Management
• Laparoscopic or open surgical resection
Resistant hypertension,is defined as hypertension that persists despite treatment with antihypertensive agents from 3
different classes, one being a diuretic. Resistant hypertension should raise suspicion for a secondary cause.

Adrenal insuf ciency (AI) can cause fatigue, weakness, anorexia, and weight loss.  In
women with AI, features of hypogonadism (eg, loss of libido, decreased pubic hair) can
be seen due to decreased adrenal androgen production; however, men with AI do not
develop these ndings as androgens are primarily produced in the testes. 
fl
fi
fi

fi

fi

15)Elevated C-peptide levels and proinsulin levels greater than 5 pmol/L are seen in
patients with beta cell tumors. Non-beta cell tumors, typically large mesenchymal tumors,
can lead to hypoglycemia independent of insulin. Such tumors produce insulin-like growth
factor II (IGF II), which has an insulinomimetic action after binding to insulin receptors. In
patients with suspected non-beta cell tumors, the serum IGF II level can be measured. Patients with
this condition characteristically have suppressed insulin and c-peptide levels.
16)Most patients develop VIPoma Syndrome (pancreatic cholera) with watery
diarrhea (can be tea colored and odorless), muscle weakness/cramps (due to
hypokalemia), and hypo- or achlorhydria.The stool volume can be greater than 3L/day,
leading to signi cant volume depletion. Nearly 75% of VIPomas are in the pancreatic
tail and 60%-80% have metastasized to the liver by the time of diagnosis. Treatment
involves intravenous volume repletion, octreotide to decrease diarrhea, and possible
hepatic resection in patients with metastasis to the liver.

17)Cirrhosis (especially due to alcoholic liver disease or hemochromatosis) can cause


hypogonadism due to primary gonadal injury or hypothalamic-pituitary dysfunction.
Cirrhosis is also associated with elevated circulating levels of estradiol due to increased
conversion from androgens.  Findings due to excess estrogen include telangiectasias, palmar
erythema, testicular atrophy, and gynecomastia (usually bilateral but can be unilateral).
Indications for statin therapy in prevention of ASCVD
• LDL ≥190 mg/dL
Primary prevention • Age ≥40 with diabetes mellitus
• Estimated 10-year risk of ASCVD >7.5%-10%

• Established ASCVD
◦ Acute coronary syndrome
Secondary prevention ◦ Stable angina
◦ Arterial revascularization (eg, CABG)
◦ Stroke, TIA, PAD
Diabetes mellitus (DM) accelerates atherosclerosis and increases the risk of developing clinically significant
atherosclerotic cardiovascular disease (ASCVD). Statin therapy is effective in reducing this risk and likely
does so independent of lipid levels.

fi

Joint National Committee 8 recommendations for treating hypertension

Initiate Rx Goal blood pressure

≥150 mm Hg systolic BP or

Age ≥60 <150/90 mm Hg


>90 mm Hg diastolic BP

Age <60, ≥140 mm Hg systolic BP or

<140/90 mm Hg
CKD,diabetes >90 mm Hg diastolic BP

Thiazide diuretic or CCB,


Black alone or in combination
(ACEI/ARB, not rst-line)

Thiazide diuretic, ACEI, ARB, or CCB,

Initial treatment choice Other ethnicities


alone or in combination

ACEI or ARB,
All ethnicities with

alone or in combination with


chronic kidney disease
other drug classes

Clinical features of acromegaly


Local tumor effect Headache, visual field defects, cranial nerve defects
Gigantism, frontal bossing, malocclusion of jaw, macrognathia,
Musculoskeletal
arthritis, carpal tunnel syndrome, enlargement of hands/feet
Skin Skin thickening, hyperhidrosis (odor), skin tags
Cardiovascular Cardiomyopathy, hypertension, heart failure
Respiratory Sleep apnea
Gastrointestinal Colon polyps/cancer, diverticulosis
Endocrine Galactorrhea, hypogonadism, diabetes mellitus, hypertriglyceridemia
Additional features Enlarged tongue, thyroid, salivary glands, liver, spleen, kidney,
prostate
Complications include heart failure (eg, dyspnea, crackles at bases) and arrhythmias. Cardiovascular
disease is the leading cause of death in patients with acromegaly, but normalization of growth hormone
levels following successful treatment markedly reduces cardiovascular mortality.

Cardiovascular effects of thyrotoxicosis

• Sinus tachycardia
Rhythm • Premature atrial & ventricular complexes
• Atrial fibrillation/flutter

• Systolic hypertension & ↑ pulse pressure


• ↑ Contractility & cardiac output
Hemodynamic effects
• ↓ Systemic vascular resistance
• ↑ Myocardial oxygen demand

• High-output failure
Heart failure
• Exacerbation of preexisting low-output failure

• Coronary vasospasm
Angina symptoms
• Preexisting coronary atherosclerosis
Increased oxygen demand in thyrotoxicosis is due to increased cardiac output and increased systemic oxygen consumption; this can lead to
anginal symptoms in patients with underlying coronary atherosclerosis. Angina may also occur due to coronary vasospasm (especially in young
female patients). Thyrotoxicosis may also cause new-onset heart failure or decompensation of preexisting heart failure.
fi

21)Hypomagnesemia is very common in hospitalized alcoholics and can cause


hypocalcemia by inducing resistance to parathyroid hormone (PTH) as well as by
decreasing PTH secretion. The cause of hypomagnesemia in alcoholics is
multifactorial, and may include urinary losses, malnutrition, acute pancreatitis, and
diarrhea. Although PTH levels increase rapidly after magnesium replacement, hypocalcemia takes
longer to improve because PTH resistance persists despite improvement in magnesium levels.
Despite PTH de ciency, phosphorus levels are normal or low in magnesium de ciency;
this is possibly due to intracellular phosphorus depletion.
(Serum calcium concentration falls by 0.8 mg/dL for every 1 gm/L decrease in albumin).

 22) Serum phosphorus is often normal in PHPT but can be low in moderate-to-severe PHPT.
Hypercalcemia of malignancy is usually due to secretion of PTH-related protein and is characterized
by severe hypercalcemia (typically >14 mg/dL) and a suppressed PTH level.
Polycystic ovary syndrome

• Androgen excess (eg, acne, male pattern baldness, hirsutism)


• Oligoovulation or anovulation (eg, menstrual irregularities)
Clinical features
• Obesity
• Polycystic ovaries on ultrasound

• ↑ Testosterone levels
Pathophysiology • ↑ Estrogen levels
• LH/FSH imbalance

• Metabolic syndrome (eg, diabetes, hypertension)


• Obstructive sleep apnea
Comorbidities
• Nonalcoholic steatohepatitis
• Endometrial hyperplasia/cancer

• Weight loss (first-line)


Treatment options • Oral contraceptives for menstrual regulation
• Letrozole for ovulation induction
The first-line therapy for menstrual regulation is a combination of weight loss and combined estrogen/progestin oral contraceptives.
Combined oral contraceptives contain progesterone to stimulate endometrial differentiation (ie, limit continued proliferation) and estrogen
to stabilize the uterine lining, which restores normal cycles. In addition, combined oral contraceptives reduce hirsutism by blocking
adrenal androgen secretion and increasing production of sex hormone-binding globulin, which binds and decreases free testosterone.

fi

fi

Drugs for neuropathic pain


Drug Mechanism of action

Tricyclic antidepressants
(eg, amitriptyline, nortriptyline) • ↓ Reuptake of serotonin & norepinephrine
• Inhibition of pain signals
SNRIs (eg, duloxetine)

Anticonvulsants
• Decreased depolarization of neurons in CNS
(eg, gabapentin, pregabalin)

Opioids • Activation of central opioid receptors

Capsaicin (topical) • Loss of membrane potential in nociceptive fibers

Lidocaine (topical) • Decreased depolarization of neurons in peripheral nerves

24)As diabetic neuropathy progresses, the pain may diminish but the sensory de cits persist.Initial
treatment options for painful diabetic neuropathy include tricyclic antidepressants (eg,
amitriptyline), dual serotonin norepinephrine reuptake inhibitors (eg, duloxetine), and
anticonvulsants (eg, pregabalin, gabapentin).  Pregabalin is a structural analogue of gamma-
aminobutyric acid and decreases pain by inhibiting the release of excitatory
neurotransmitters by binding to voltage-gated calcium modulators on nerve terminals. 
Common side effects are drowsiness, weight gain, and uid retention.
Milk-alkali syndrome
• Excessive intake of Calcium & Absorbable Alkali
Pathophysiology • Renal vasoconstriction & decreased GFR
• Renal loss of sodium & water, Reabsorption of bicarbonate

• Nausea, Vomiting, Constipation


Symptoms • Polyuria, polydipsia
• Neuropsychiatric symptoms

• Hypercalcemia,hypophosphatemia, hypomagnesemia
Laboratory • Metabolic alkalosis
findings • Acute kidney injury
• Suppressed PTH

• Discontinuation of causative agent


Treatment
• Isotonic saline followed by furosemide
Hypercalcemia in milk-alkali syndrome causes renal vasoconstriction and decreased glomerular blood flow.
In addition, inhibition of the Na-K-2Cl cotransporter (due to activation of calcium-sensing receptors in the thick
ascending loop) and impaired activity of antidiuretic hormone lead to the loss of sodium and free water. This
results in hypovolemia and increased reabsorption of bicarbonate (augmented by the increased intake of alkali).
Risk factors include chronic kidney disease and concurrent use of thiazide diuretics (which promote volume
depletion and calcium retention), ACE inhibitors, or nonsteroidal anti-inflammatory drugs (which decrease
glomerular filtration).

Evaluation of milk-alkali syndrome should include measurement of serum parathyroid


hormone (PTH), which would be expected to be suppressed. Additional findings can
include hypophosphatemia due to intestinal binding of phosphate by calcium
carbonate, and hypomagnesemia due to decreased renal reabsorption of magnesium.

fl

fi


25)Patients with long-standing diabetes often have a blunted autonomic response to
hypoglycemia, with reduced hypoglycemia awareness. This blunted autonomic response
is worse in patients who frequently have low circulating glucose levels. Such recurrent
or severe hypoglycemia reduces the glucose-raising effects of epinephrine and
suppresses the symptoms related to the catecholamine surge; in turn, this increases the
risk for progressively worse hypoglycemic episodes (ie, hypoglycemia-associated
autonomic failure). Management requires strict avoidance of hypoglycemia to restore awareness.  Judicious
timing of insulin administration and carbohydrate ingestion (eg, in relation to physical activity) is necessary.
Differential diagnosis of myopathy
Disorder Clinical features ESR CK

• Progressive proximal muscle weakness & atrophy without


Glucocorticoid-induced pain or tenderness Normal Normal
myopathy • Lower extremity muscles are more involved

• Muscle pain & stiffness in the shoulder & pelvic girdle


Polymyalgia rheumatica • Tenderness with decreased range of motion at shoulder, ↑ Normal
neck & hip
• Responds rapidly to glucocorticoids
• Muscle pain, tenderness & proximal muscle weakness
Inflammatory myopathies ↑ ↑
• Skin rash & inflammatory arthritis may be present

• Prominent muscle pain/tenderness with or without


Statin-induced myopathy Normal ↑
weakness
• Rare rhabdomyolysis
• Muscle pain, cramps & weakness involving the proximal
muscles
Hypothyroid myopathy Normal
• Delayed tendon reflexes & myoedema
• Occasional rhabdomyolysis
• Features of hypothyroidism are present
26)Glucocorticoid-induced muscle atrophy is due to the direct catabolic effects of
cortisol on skeletal muscle, which leads to muscle atrophy.
It is thought to be mediated by the inhibition of Akt-1, an intracellular signaling
molecule with serine/threonine kinase activity. Interference of insulin-like growth
factor 1 signaling may also contribute. Furthermore, glucocorticoids may decrease
muscle cell differentiation and protein synthesis.

27)Serum TSH levels should be obtained, along with a thyroid ultrasound to


determine nodule size and sonographic features. Certain sonographic features
(microcalcifications, irregular margins, internal vascularity) carry a much higher risk of
malignancy than others (cystic or spongiform lesions). Thyroid nodules >1 cm with
these high-risk sonographic features—and all noncystic thyroid nodules >2 cm—should
undergo ne-needle aspiration (FNA) biopsy.

28)Thyrotoxicosis in subacute thyroiditis resolves spontaneously within a few weeks


and may be followed by a hypothyroid phase lasting a few months. Most patients
eventually recover to a euthyroid state. Treatment is symptomatic with beta blockers
to control thyrotoxic symptoms and nonsteroidal anti-inflammatory drugs (NSAIDs)
for pain relief. Glucocorticoids are used for severe thyroid pain not responding to
NSAIDs.

Sheehan syndrome
• Obstetric hemorrhage complicated by hypotension

Pathogenesis
• Postpartum pituitary infarction

• Lactation failure (↓ prolactin)

• Amenorrhea, hot ashes, vaginal atrophy (↓ FSH, LH)

Clinical

• Fatigue, bradycardia (↓ TSH)

features
• Anorexia, weight loss, hypotension (↓ ACTH)

• Decreased lean body mass (↓ growth hormone)

29)Adrenal Insufficiency is characterized by cortisol deficiency. It presents with


gastrointestinal symptoms (eg, abdominal pain, nausea, anorexia) and loss of vascular
tone (eg, orthostatic hypotension). Physiologic stress (eg, surgery, illness) can trigger
adrenal crisis and lead to vascular collapse. In addition, because cortisol acts as an
inhibitor of antidiuretic hormone, AI often leads to the syndrome of inappropriate
antidiuretic hormone release (SIADH) with hyponatremia.
fi

fl

30)Diabetes mellitus is the most common cause of peripheral neuropathy, with the risk
related to the duration of the disease and glycemic status. Neuronal injury in diabetes is
due to a number of factors, including microvascular injury, demyelination, oxidative
stress, and deposition of glycation end products. The use of metformin, which
decreases intestinal absorption of vitamin B12, can also contribute. This leads to a length-
dependent axonopathy, with clinical features occurring rst in the longest nerves (eg, feet).
Symmetric distal sensorimotor polyneuropathy is the most common neuropathy
in patients with diabetes; the clinical features depend on the type of nerve fibers
involved.
• Small ber injury is characterized by the predominance of Positive symptoms (eg,
pain, paresthesia, allodynia).
• Large ber involvement is characterized by the predominance of Negative
symptoms (eg, numbness, loss of proprioception and vibration sense, diminished
ankle re exes).
Distal sensory peripheral neuropathy in diabetes mellitus
Large fiber neuropathy* Small fiber neuropathy*

Sensory
• Pressure, proprioception & balance • Pain & temperature
function

Major
• Numbness & poor balance • Burning & stabbing pain
symptoms

• Diminished/absent ankle reflexes


Examination • Reduced pinprick sensation
• Reduced/absent vibration, light touch
findings • Ankle reflexes possibly preserved
& proprioception
*Patients can have mixed large & small fiber neuropathy.

31)Most patients with PHPT Present with Mild, Asymptomatic Hypercalcemia,


although potential clinical features may include nephrolithiasis, osteoporosis, nausea,
constipation, and neuropsychiatric symptoms ("stones, bones, abdominal moans,
psychic groans"). However, in hypercalcemia due to malignancy, serum calcium is
typically much higher (often >14 mg/dL) than in patient with PHPT(which often causes
mild asymptomatic hypercalcemia) and more acute. In addition, patients are more
likely to be symptomatic (eg, polyuria, dehydration, mental status changes).

32)Patients with hypertension should be screened for diabetes mellitus with either
fasting blood glucose, which can be obtained as part of a serum chemistry panel, or
hemoglobin A1c. Even in the absence of hypertension, screening is advised for
patients age 40-70 with a BMI ≥25 kg/m2 (ie, overweight or obese).

fi
fi

fl

fi

Screening at younger ages should be considered for those with additional risk factors
(eg, family history of diabetes, high-risk racial/ethnic groups [eg, African Americans,
Native Americans]).
33)In a patient with an intact thyroid gland, thyroglobulin testing is not useful because
it can be elevated in many thyroid conditions and does not differentiate between benign
and malignant lesions.Serum thyroglobulin is useful as a tumor marker for patients who
have had a total thyroidectomy for papillary or follicular thyroid carcinoma.
Systemic hypertension may be seen in both hyperthyroidism and hypothyroidism.
Hyperthyroidism causes a decrease in systemic vascular resistance, but blood pressure
(primarily systolic) rises due to positive inotropic and chronotropic effects. In contrast,
hypothyroidism causes hypertension due to an increase in systemic vascular resistance.

34)DI can be central (decreased antidiuretic hormone [ADH] secretion from the
pituitary) or nephrogenic (normal ADH levels with renal ADH resistance). Central DI
patients usually do not have an intact thirst mechanism and can have serum sodium
>150 mEq/L.  Nephrogenic DI patients usually have an intact thirst mechanism with
lower serum sodium (~145 mEq/L).  However, some cases may not be clinically
obvious. A water deprivation test can distinguish between central and nephrogenic DI
and also definitively exclude primary polydipsia.After water Deprivation the increase in
urine osmolality >600 mOsm/kg suggests primary polydipsia due to intact ADH and
ability to concentrate urine in the absence of water intake. Central DI typically has
>50% (sometimes up to 200%-400%) increase in urine osmolality with desmopressin.
Nephrogenic DI has minimal change in urine osmolality with desmopressin.
35)The first step in evaluating hypocalcemia is to measure the serum albumin level
because hypoalbuminemia lowers total circulating calcium (measured on the basic
metabolic profile) without affecting the metabolically active ionized calcium
concentration (pseudohypocalcemia). If albumin is low, a corrected calcium level can
be calculated (corrected calcium = [measured calcium] + 0.8 × [4.0 g/dL − serum
albumin in g/dL]).

Male breast cancer

• Family history; BRCA 1/2


Risk factors • Abnormal estrogen/androgen ratio: Klinefelter syndrome, obesity, cirrhosis,
marijuana use
Klinefelter syndrome (KS) (47,XXY) is associated with testicular fibrosis and is a common cause of primary
hypogonadism. Testosterone production is reduced, and the compensatory increase in gonadotropins causes
increased expression of aromatase, leading to increased conversion of testosterone to estradiol and an elevated
estrogen/androgen ratio. In the breast, this causes gynecomastia and an elevated risk of breast cancer.

• Subareolar mass
Presentation • Skin & nipple dimpling, induration, ulceration
• Often detected at advanced stage

• Mammography
Diagnosis
• Biopsy: invasive ductal carcinoma (hormone receptor–positive) most common
Characteristic • Gynecomastia can have variable density but is typically symmetric, is centrally
features that can located with respect to the nipple, and may have an indistinct margin with surrounding
distinguish benign
gynecomastia from fat.
malignancy • Breast cancer is more likely eccentric to the nipple, has well-defined or spiculated
include: margins, and may contain calcifications.

36)Selective estrogen receptor modulators (eg, tamoxifen) are commonly used as adjuvant therapy
for estrogen receptor–positive breast cancer and are occasionally given to adolescents with KS to
prevent gynecomastia.  They have limited use for established gynecomastia.

Polycystic ovary syndrome

• Androgen excess (eg, acne, male pattern baldness, hirsutism)


• Oligoovulation or anovulation (eg, menstrual irregularities)
Clinical features
• Obesity
• Polycystic ovaries on ultrasound

• ↑ Testosterone levels
Pathophysiology • ↑ Estrogen levels
• LH/FSH imbalance

• Metabolic syndrome (eg, diabetes, hypertension,dyslipidemia)


• Obstructive sleep apnea
Comorbidities
• Nonalcoholic steatohepatitis
• Endometrial hyperplasia/cancer

• Weight loss (first-line)


Treatment options • Oral contraceptives for menstrual regulation
• Letrozole for ovulation induction
The first-line therapy for menstrual regulation is a combination of weight loss and combined estrogen/progestin oral contraceptives.
Combined oral contraceptives contain progesterone to stimulate endometrial differentiation (ie, limit continued proliferation) and estrogen to stabilize
the uterine lining, which restores normal cycles. In addition, combined oral contraceptives reduce hirsutism by blocking adrenal androgen
secretion and increasing production of sex hormone-binding globulin, which binds and decreases free testosterone.

37)BRCA mutation testing is indicated for patients with a family history of ovarian
cancer at any age or personal/family history of breast cancer at age ≤50 in a rst-degree
relative. It is not indicated for a family history of endometrial cancer or breast cancer at
an advanced age.

fi
BLOCK 10
Custom Id: 212020841


38)Thyroid storm can lead to cardiac arrhythmias, congestive heart failure,


seizures, hypotension, and shock.  The diagnosis is based on clinical evaluation.
Thyroid storm is usually seen in patients with undiagnosed or inadequately treated
hyperthyroidism.

39)Peutz-Jeghers syndrome is characterized by gastrointestinal tract hamartomatous


polyposis. Patients develop oral lesions, that are typically perioral mucocutaneous
pigmented macules.

40)Secondary hypogonadism is a common adverse effect of opioids.


Opioids suppress GnRH and LH secretion, leading to reduced Leydig cell testosterone
synthesis, decreased spermatogenesis, and testicular atrophy. Additional
manifestations can include depression, hot ashes, and osteoporosis. In women,
menstrual irregularities are common. In Primary hypogonadism the elevated
gonadotropin levels stimulate aromatization of testosterone to estradiol, leading to
gynecomastia, breast enlargement in secondary hypogonadism is less prominent.
Male secondary hypogonadism

• Fatigue, decreased libido


Clinical features • Testicular atrophy
• Laboratory: low testosterone, low/normal LH

• Pituitary tumors, hyperprolactinemia


• Medications: opioids, glucocorticoids, exogenous androgens (withdrawal phase)
Common
• In ltrative disease (eg, hemochromatosis)
causes
• Chronic/severe illness
• Eating disorders, severe weight loss
fi

fl

48)Clopidogrel is used in secondary prevention of recurrent ischemic stroke. For


certain patients with diabetes (>10% 10-year stroke risk), primary stroke prevention
with daily low-dose aspirin (not clopidogrel) can be considered.

50)Neuropathic Ulcers most commonly occur in the feet under bony prominences,
such as the metatarsal heads. They typically have a punched-out or undermined
border. Neuropathy decreases pain sensation and pressure perception, it causes muscle
imbalance leading to foot deformities, and impairs the microcirculation and integrity of
the skin.Peripheral sensory neuropathy can be documented by testing for pressure
sensation with a 10-g monofilament.
The 6-minute walk test is an assessment of functional status that measures how far a patient can
walk in a standardized time.  It is typically used for patients with chronic lung or heart disease
or with chronic pain conditions (eg, bromyalgia, osteoarthritis). 
51)Peripheral Arterial disease (PAD) can be assessed by calculating the ankle-
brachial index (ABI). However, ABI is primarily a measure of large vessel PAD and
does not accurately assess small vessel disease, which often contributes to ulcers in
diabetic patients. In addition, arterial ulcers are usually located on the tips of the toes,
rather than the plantar surface.
Complications of Graves disease treatment
Treatment Adverse effects
• Agranulocytosis
Antithyroid drugs • Methimazole: 1st-trimester teratogen, cholestasis
(thionamides)
• Propylthiouracil: Hepatic failure, ANCA-associated vasculitis

• Permanent hypothyroidism
Radioiodine ablation • Worsening of ophthalmopathy
• Possible radiation side effects
• Permanent hypothyroidism
Surgery • Risk of recurrent laryngeal nerve damage
• Risk of hypoparathyroidism

52)The most common side effect of ATDs is allergic reaction (2% of patients). The
most serious side effect is agranulocytosis (0.3% of patients), and all patients must be
informed about it.



fi

88)HHS (hyperosmolar hyperglycemic state) often occurs in a setting of type 2 diabetes.


Compared with diabetic ketoacidosis (DKA), HHS is associated with higher glucose levels
(often >1,000 mg/dL in HHS, 300-500 mg/dL in DKA), normal anion gap and pH, and absent
ketoacids. Dehydration is more severe in HHS (up to 8-10 L of total body water losses) due
to higher plasma osmolality and resulting osmotic diuresis. The best immediate step in
treatment is aggressive fluid replacement with isotonic normal saline (NS), which
replenishes extracellular volume, lowers plasma osmolality, and increases tissue perfusion and
responsiveness to insulin.The uid can be changed to 0.45% NS after the second or third hour
if the corrected serum sodium is normal or high on repeat laboratory testing.
Hyperosmolar hyperglycemic state
Patient • Type 2 diabetes mellitus
characteristics • Older age

• Gradual hyperglycemic symptoms (eg, polyuria,


Clinical polydipsia)
symptoms • Altered mentation (eg, coma, focal signs,
seizures)

• Glucose >600 mg/dL (frequently >1,000 mg/dL)


• Normal pH & bicarbonate
Laboratory
• Normal anion gap
studies
• Negative or small serum ketones
• Serum osmolality >320 mOsm/kg

• Aggressive hydration with normal saline*


Initial
• Intravenous insulin
management
• Potassium replacement if level is <5.3 mEq/L

*Normal saline for the first hour regardless of sodium levels; fluid choice may
change thereafter

Diabetic ketoacidosis Hyperosmolar hyperglycemic state

Patient • Type 1 diabetes usually • Type 2 diabetes usually


characteristics • Younger age • Older age

• Less pronounced altered mentation • More pronounced altered mentation


• More rapid onset of hyperglycemic • Gradual onset of hyperglycemic
Clinical
symptoms symptoms
symptoms
• Hyperventilation & abdominal pain • Hyperventilation & abdominal pain less
common common

• Glucose 250-500 mg/dL • Glucose >600 mg/dL (33.3 mmol/L)


• (13.9-27.8 mmol/L) • Bicarbonate >18 mEq/L (18 mmol/L)
• Bicarbonate <18 mEq/L (18 mmol/L) • Normal anion gap
Laboratory
• Elevated anion gap • Negative or small serum ketones
studies
• Positive serum ketones • Serum osmolality >320
• Serum osmolality <320 mOsm/kg
• (320 mmol/kg)
Common precipitating factors causing HHS include:
• Infection
• Medications impacting carbohydrate metabolism (eg, glucocorticoids, thiazide diuretics, atypical antipsychotics)
• Interruption of insulin therapy
• Trauma or acute illness (eg, myocardial infarction)

Severe hyperglycemia causes glycosuria, osmotic diuresis, hypovolemia, and dehydration.  With worsening
hypovolemia, glomerular ltration rate (GFR) declines, leading to reduced renal glucose excretion
and worsening hyperglycemia.  Elderly individuals are at increased risk due to altered perception of thirst and
restricted uid intake.





fl



fi

fl

89)Women normally produce a number of androgens, including testosterone (T),


androstenedione (AS), dehydroepiandrosterone (DHEA), and dehydroepiandrosterone
sulfate (DHEAS). AS, DHEA, and T are produced by both the ovaries and the adrenals.
In contrast, DHEAS is produced predominantly in the adrenal glands.
Causes of hirsutism in women

Etiology Clinical features

• Oligomenorrhea, hyperandrogenism, obesity


Polycystic ovary syndrome
• Associated with type 2 diabetes, dyslipidemia, hypertension

Nonclassic • Similar to polycystic ovary syndrome


21-hydroxylase de ciency • Elevated serum 17-hydroxyprogesterone

• More common in postmenopausal women


Androgen-secreting ovarian tumors,
ovarian hyperthecosis
• Rapidly progressive hirsutism with virilization
• Very high serum androgens

• Obesity (usually of the face, neck, trunk, abdomen)


Cushing syndrome
• Increased libido, virilization, irregular menses

• Normal menstruation
Idiopathic hirsutism
• Normal serum androgens
90)Although most women with hirsutism (excessive terminal hair growth) have
polycystic ovary syndrome (PCOS), rapid-onset hirsutism (eg, <1 year), particularly
with virilization (eg, temporal balding, excessive muscular development, enlarged
clitoris), suggests very high androgen levels due to an androgen-secreting neoplasm of
the ovaries or adrenal glands.
Causes of hyperandrogenism in women

Diagnosis Clinical features

Oligo-ovulation, clinical or biochemical


PCOS hyperandrogenemia, polycystic ovaries on
imaging, no evidence of another diagnosis

Nonclassic CAH Oligo-ovulation, hyperandrogenemia,


↑ 17-hydroxyprogesterone levels

Ovarian/adrenal Older age, rapidly progressive symptoms,


tumors ↑ androgen levels (>3 times upper limit of normal)
Cushingoid features, nonsuppressible
Cushing syndrome dexamethasone suppression test, ↑ 24-hour
urinary free cortisol

Hyperprolactinemia Amenorrhea, galactorrhea, ↑ prolactin levels

Acromegaly Excessive growth, ↑ GH & IGF-1 levels


Serum testosterone and DHEAS levels are used to delineate the site of androgen production.  Elevated
testosterone with normal DHEAS suggests an ovarian source, whereas Elevated testosterone and
elevated DHEAS suggests an adrenal source.

fi

Diabetic gastroparesis
• Autonomic neuropathy
Pathogenesis • Destruction of enteric neurons
• Failure of relaxation in fundus & uncoordinated peristalsis

• Long-standing diabetes (especially type 1)


Risk factors • Chronic poor control
• Labile blood glucose

• Postprandial bloating & vomiting


Clinical presentation • Early satiety
• Impaired nutrition & weight loss

Diagnosis • Nuclear gastric emptying study: delayed transit into duodenum

Treatment • Promotility drugs: metoclopramide, erythromycin


Diabetic autonomic neuropathy due to chronic hyperglycemia can lead to esophageal motility disorders (eg,
dysphagia), delayed gastric emptying (eg, gastroparesis), or intestinal dysfunction (eg, diarrhea, constipation,
incontinence). Delayed gastric emptying is a result of uncoordinated smooth muscle contractions and
ineffective peristalsis. Due to delayed intestinal absorption of glucose, patients can have difficulty in timing
insulin doses, leading to labile blood glucose and frequent hypoglycemia.

92)The most accurate markers indicating resolution of diabetic ketoacidosis are the
serum anion gap and serum beta-hydroxybutyrate levels.  The anion gap estimates the
unmeasured anion concentration in the blood and returns to normal with the
disappearance of ketoacid anions. Posttreatment hyperchloremic acidosis may occur
due to infusion of normal saline; however, the anion gap will be normal.
Resolution of DKA is further supported by normalization of the venous pH and serum
beta-hydroxybutyrate levels, rise in serum bicarbonate, and tolerance of oral food
intake. Measurement of serum and urine glucose alone is inappropriate for monitoring
response to treatment because ketosis and acidemia may still be present even when
glucose levels drop below 200-250 mg/dL.
Urine ketone testing primarily measures acetoacetate, not beta-hydroxybutyrate.
Therefore, although measurement of urine or serum acetoacetate is appropriate for DKA
diagnosis, these tests should not be used to monitor for DKA resolution.
93)Advanced Diabetic nephropathy (DN) can present with frank proteinuria and
nephrotic syndrome.Strict blood pressure control, particularly using ACE inhibitors or
angiotensin II receptor blockers, can reduce the progression of diabetic nephropathy
(DN). Tight glycemic control with a target hemoglobin A1c <7% also can prevent
progression of DN; further lowering is associated with an increased risk of
hypoglycemia and possibly cardiac events.

94) Aldosterone de ciency in PAI causes volume depletion and cortisol de ciency causes loss
of vascular tone; these changes can manifest as hypotension, postural dizziness, or syncope. 
PAI can cause mild hypercalcemia due to increased release of calcium from the bones, increased
proximal tubular calcium absorption, and volume contraction.  PAI with concurrent
hypoparathyroidism leading to hypocalcemia can be seen in the polyglandular autoimmune
syndrome type 1 but is extremely rare

fi

fi

95)GH stimulates hepatic insulin-like growth factor-1 (IGF-1) secretion, which mediates most of the clinical
manifestations of acromegaly.  IGF-1 levels in patients with acromegaly are consistently elevated
throughout the day and are useful as an initial test, in contrast to GH, which fluctuates widely and has low
sensitivity and specificity.

Initial treatment for most patients includes transsphenoidal resection of the pituitary adenoma.  For patients with
residual or unresectable tumors, medical therapy is indicated; options include somatostatin analogues (eg,
octreotide) or growth hormone receptor antagonists (eg, pegvisomant).
Hereditary hemochromatosis
• Hyperpigmentation
• Arthropathy
• Hepatomegaly, cirrhosis, hepatocellular carcinoma
Clinical manifestations
• Diabetes mellitus
• Hypopituitarism, secondary hypogonadism
• Cardiomyopathy

• Elevated liver transaminases


Diagnosis • Elevated serum ferritin, transferrin saturation
• HFE genetic mutations
Concurrently, hypogonadism can be assessed with serum testosterone, FSH, and LH, which are characteristically low. 
Management • Therapeutic phlebotomy (urgent if ferritin >1,000 ng/mL)

Major drug interactions of levothyroxine


• Bile acid-binding agents (eg,
↓ cholestyramine)
Levothyroxine • Iron, calcium, aluminum hydroxide
absorption
• Proton pump inhibitors, sucralfate

↑ TBG • Estrogen replacement, oral contraceptives


concentration • Tamoxifen

↓ TBG • Androgens, glucocorticoids


concentration • Anabolic steroids

↑ Thyroid • Rifampin
hormone • Phenytoin
metabolism • Carbamazepine

Patients on thyroid replacement therapy who are prescribed oral estrogens require a compensatory adjustment in levothyroxine dose.




Hypoglycemia in insulin therapy


Patients with a diminished glucagon response, such as those with pancreatogenic diabetes (eg, chronic pancreatitis with
fibrosis of the islets), can develop rapid and severe hypoglycemia with little warning. Such episodes can lead to confusion,
seizure, and loss of consciousness and can be fatal. Even mild hypoglycemia can be distressing due to the associated
autonomic activation, which can induce anxiety, tremor, palpitations, and sweating.

• Long-standing type 1 diabetes


Risk factors
• Pancreatogenic diabetes

• Mild to moderate (neurogenic symptoms): anxiety, tremor, palpitations,


Manifestations
sweating
• Severe (neuroglycopenic symptoms):confusion, seizure, loss of
• Query patients regarding hypoglycemic symptoms
Management • Individualized glycemic targets
strategies • Flexible insulin dosing regimen
• Emergency glucagon kits

SGLT-2 inhibitors*
Mechanism • Increased urinary glucose excretion (block proximal
of action tubule glucose reabsorption)

• Reduced progression of nephropathy &


albuminuria
Possible • Reduced cardiovascular morbidity & mortality
benefits
• Reduced hospitalizations for heart failure
• Weight loss

Adverse • Euglycemic ketoacidosis


effects • Increased risk of genitourinary infections

• Type 1 DM SGLT-2 inhibitors;


Contraindic
ations • History of DKA Glucagon-like peptide-1 (GLP-1)
• Impaired renal function (eGFR <30 mL/min/1.73 m2) receptor agonists (eg, semaglutide,
liraglutide);
Recommended add-on therapy for patients with established cardiovascular Both reduce cardiovascular mortality
disease (eg, heart failure, myocardial infarction, coronary artery disease) includes: and induce signi cant weight loss. 

Management of type 2 diabetes in patients with heart disease


• Smoking cessation
Lifestyle • Regular exercise
• Reduced fat intake

• Metformin
Glycemic management • GLP-1 agonists (eg, liraglutide)
• SGLT2 inhibitors (eg, empagliflozin)
Lipid-lowering therapy • Statins
• Goal BP <140/90 mm Hg (consider <130/80 mm Hg)
Blood pressure control
• ACE inhibitors preferred
Antiplatelet therapy • Low-dose aspirin


fi


Latest UW Notes for STEP2 CK - 2022
Features:
• Arranged system wise and then subject wise
• There are total 97 Blocks, Each BLOCK is made of 40 or some time 45 -50 different Questions
hence each BLOCK covers the NOTEs of those 40-50 questions,while the total 97 Blocks
covers the whole UWorld (3986 Qs)

• Custom BLOCK Id is mentioned with each BLOCK, so you can make Your own blocks in UWorld
according to the BLOCK id mentioned and read the respective BLOCK,s NOTES here

• Contains All UW tables edited and supplemented with UW Explanations


• All UW Diagrams/Pictures and algorithms annotated and simpli ed
• Some topics are supplemented with UPTODATE information,separate tables are there for
these info so that you can differentiate it form UW info

• Highlighted important info


• Most of the UW Tables are edited with related pictures and de cinet information added to
the tables from UW explanations

• These Notes will be updated regularly


• Notes of the Upcoming NEW Questions will be added on monthly basis

fi
fi


GASTROENTEROLOGY:

MEDICINE
Notes by: Step1usmle59@gmail.com
https://www.reddit.com/u/USMLEe59/?
utm_source=share&utm_medium=ios_app
&utm_name=iossmf

BLOCK 13
Custom Id: 213697741


Clinical features of acute pancreatitis


• Chronic alcohol use Amylase rises within 6-12 hours of
• Gallstones symptom onset and may remain
• Hyperlipidemia (types I, IV & V) elevated for 3-5 days.  Lipase rises
Etiology • Drugs (eg, didanosine, azathioprine, valproic acid) within 4-8 hours of symptom onset
• Infections (eg, cytomegalovirus, Legionella, Aspergillus)
but remains elevated longer than
• Trauma
• Iatrogenic (post-ERCP) amylase (8-14 days).  As a result,
lipase is more useful and sensitive
Diagnosis requires 2 of the following: than amylase for diagnosis
• Acute epigastric abdominal pain often radiating to the
back (especially in alcoholics and
• ↑ Amylase/lipase >3 times normal limit patients presenting later in the
• Abdominal imaging showing focal or diffuse pancreatic disease course).
enlargement with heterogeneous enhancement with
Clinical
presentation intravenous contrast (CT scan) or diffusely enlarged & Contrast-enhanced computed
hypoechoic pancreas (ultrasound)
tomography scan of the abdomen
Other findings:
• Nausea, vomiting, leukocytosis may be performed in patients with
• Severe disease with possible abdominal tenderness, unclear diagnosis or in those who
fever, tachypnea, hypoxemia & hypotension fail to improve with conservative
• ALT level >150 U/L → biliary pancreatitis
management (to identify infection
• Pleural effusion or necrosis). 
• Ileus
Complications
• Pancreatic pseudocyst/abscess/necrosis
• Acute respiratory distress syndrome

Glucocorticoids are used to treat autoimmune pancreatitis, a relatively rare IgG4 mediated disorder.  These drugs should not
be initiated prior to diagnostic con rmation, which typically requires pancreatic biopsy in addition to a serum IgG4 level.

Drugs associated with drug-induced pancreatitis

• Acetaminophen
• Nonsteroidal anti-inflammatory
Analgesics drugs
• Mesalamine, sulfasalazine
• Opiates

• Isoniazid
• Tetracyclines
Antibiotics
• Metronidazole
• Trimethoprim-sulfamethoxazole

• Valproic acid
Antiepileptics
• Carbamazepine

• Thiazides, furosemide
Antihypertensives
• Enalapril, losartan

• Lamivudine Patients with gallstone pancreatitis should


Antivirals
• Didanosine,Pentamidine undergo timely cholecystectomy to help
Immunosuppress • Azathioprine, mercaptopurine prevent additional episodes. Endoscopic
ants • Corticosteroids retrograde cholangiopancreatography (ERCP)
can be used to remove gallstones obstructing
• Asparaginase the biliary tract, and it is indicated in patients
Others • Estrogens with gallstone pancreatitis complicated by
• Simvastatin cholangitis.

fi

Overview of chronic pancreatitis


CA 19-9 levels should not be used to
• Alcohol use diagnose suspected pancreatic cancer (due
• Cystic fibrosis (common in children) to limited sensitivity and speci city). 
Etiology
• Ductal obstruction (eg, malignancy, stones) Abdominal CT scan is usually the rst step.
• Autoimmune
Major risk factors include recurrent episodes of
• Chronic epigastric pain that can radiate to back and
acute pancreatitis, alcohol abuse, smoking, and
is partially relieved with sitting upright or family history (eg, hereditary pancreatitis). 
leaning forward with intermittent pain-free
Clinical
intervals,worsen with meal
presentation
• Malabsorption: steatorrhea (characterized by bulky
diarrhea that is dif cult to ush), weight loss
• Diabetes mellitus

• Amylase/lipase can be normal &


Laboratory
nondiagnostic
results/
• CT scan or MRCP can show calcifications,
imaging
dilated ducts & enlarged pancreas

• Pain management
• Alcohol & smoking cessation
Treatment
• Frequent, small meals
• Pancreatic enzyme supplements

Hepatic encephalopathy 1)-Hypokalemia,can exacerbate HE as the


resultant intracellular acidosis (excreted
• Drugs (eg, sedatives, narcotics)
• Hypovolemia (eg, diarrhea) intracellular potassium replaced by hydrogen
Precipitating • Electrolyte changes (eg, hypokalemia) ions to maintain electroneutrality) causes
factors • ↑ Nitrogen load (eg, GI bleeding) increased NH3 production (glutamine
• Infection (eg, pneumonia, UTI, SBP)
• Portosystemic shunting (eg, TIPS)
conversion) in renal tubular cells
Metabolic alkalosis (elevated bicarbonate),
• Sleep pattern changes which can also exacerbate HE as it promotes
Clinical • Altered mental status
presentation • Ataxia conversion of ammonium (NH4+), which cannot
• Asterixis enter the CNS, to NH3, which can enter the CNS

• Correct precipitating causes (eg, fluids, Patients with cirrhosis tend to be malnourished; therefore,
dietary protein restriction is generally not recommended.
antibiotics) Protein-free diets can cause a negative nitrogen balance and
Treatment
• ↓ Blood ammonia concentration (eg, increase mortality.Protein restriction is generally limited to
lactulose, rifaximin) patients who have require TIPS.

Treatment of Clostridioides difficile infection


Risk factors for CDI include
• Vancomycin PO recent hospitalization,
Initial episode OR advanced age, or antibiotic
• Fidaxomicin use (most commonly
fluoroquinolones, penicillins,
• First recurrence cephalosporins, and
◦ Vancomycin PO in a prolonged pulse/taper course
◦ OR
clindamycin).
Recurrence ◦ Fidaxomicin if vancomycin was used in initial episode Unexplained leukocytosis in
• Multiple recurrences hospitalized patients should
◦ Vancomycin PO followed by rifaximin (or above regimens) raise suspicion for CDI, even
◦ Fecal microbiota transplant without diarrhea. CDI can
range from mild watery
Fulminant • Metronidazole IV plus high-dose vancomycin PO (or PR if ileus is diarrhea to fulminant colitis
(eg, hypotension,
present) with toxic megacolon.
shock,ileus,
megacolon) • Surgical evaluation

fi

fl

fi

fi

06)Minimal bright red blood per rectum (BRBPR) is most often due to benign
disorders such as hemorrhoids or rectal ssures. However, more serious disorders (eg,
proctitis, rectal ulcers, colorectal polyps, cancer) are possible.
The evaluation of BRBPR depends on the patient's presentation and risk factors.
Clinical factors associated with increased risk of serious disease include blood mixed
with stool, systemic symptoms (eg, fever, weight loss), diarrhea, anemia, change in bowel
habits, and abdominal pain. Age also correlates strongly with risk of malignancy in
patients with BRBPR.

07)Common neurologic findings of vitamin B12 deficiency include subacute


combined degeneration of the dorsal columns (eg, impaired proprioception/vibrioception)
and the lateral corticospinal tracts (eg, positive Babinski reflex), but patients often rst develop
symmetric lower extremity paresthesia due to myelinated peripheral nerve damage.
HIV can cause a distal symmetric polyneuropathy that generally begins with tingling and
numbness in the bilateral feet. However, all sensory modalities are typically diminished (not
just light touch and vibration), reflexes are generally impaired, and neuropathic pain is
usually prominent.
08)In general, Stable patients with upper gastrointestinal bleeding (UGIB) without
significant comorbid conditions should receive PRBC transfusion for hemoglobin <7 g/dL. A
higher threshold of hemoglobin <9 g/dL can be considered for patients with acute coronary
syndrome. Patients with active bleeding and hypovolemia may need PRBC transfusion at
higher hemoglobin levels due to the initial hemoglobin concentration not fully reflecting
blood loss. In addition, the hemoglobin level may drop significantly as blood volume is
replaced by the infusion of crystalloid solutions and the mobilization of interstitial fluid.

Whole blood transfusion, which includes PRBCs in addition to plasma, may be used in
patients with severe hemorrhage (eg, major trauma) requiring massive blood
transfusions to assist in volume expansion.

fi

fi

Proctalgia fugax - (A Diagnosis of Exlusion)


• Spastic contraction of the anal sphincter
Pathophysiology
• Intermittent Pudendal nerve compression

• Female sex
Risk factors • Other functional pathologies (eg,IBS) Coccydynia (pain at the coccyx) is most
• Psychosocial stress,Anxiety commonly diagnosed in women with
• Recurrent rectal pain unrelated to defecation obesity and often results from
• Episodes lasting seconds to minutes (≤30 traumatic injury.  The pain is
Manifestations exacerbated by sitting, and a rectal
min)
• No pain between episodes examination elicits tenderness of the
coccyx bone during palpation.
• Normal physical examination (eg, rectal,
Evaluation pelvic, prostate)
• No laboratory abnormalities

• Reassurance
Management • Nitroglycerin cream ± biofeedback therapy
for refractory symptoms

10)Patients with Alcoholic Hepatitis commonly have a history of chronic, heavy


alcohol use (>7 drinks/day) and sometimes develop AH symptoms after an acute increase in
consumption.Treatment involves abstinence, supportive care (eg, hydration and nutrition support), and
acid suppression.  Liver biopsy can be helpful if there is diagnostic uncertainty.

11)Pain from shingles may precede the onset of the classic vesicular rash by
several days, during which the diagnosis may not be obvious.The possibility of shingles
should be considered in patients with regional pain who have no conclusive evidence of
disease in the local internal organs.In some cases, patients may develop persistent
hypersensitivity of afferent pain bers leading to chronic pain known as post-herpetic
neuralgia. Treatment with antiviral medications (acyclovir, valacyclovir, or famciclovir)
in the first few days of a shingles outbreak can shorten the duration of symptoms and
decrease the risk of post-herpetic neuralgia.

fi

Hepatitis A DRUG INDUCED HEPATITIS

• Fecal-oral transmission Drugs and toxins typically cause hepatic


Transmission • Poor sanitation/hygiene, travel to
& risk factors endemic areas,IVDA injury, either through direct toxic effects or
• Contaminated food & water through idiosyncratic reactions.  The direct
toxic effects are dose-dependent and
Clinical • Fever, nausea, right upper quadrant pain
presentation • Jaundice, hepatomegaly have short latent periods. Examples
include acetaminophen and Amanita
• Elevated liver transaminases (>1,000 U/L) phalloides mushroom.Idiosyncratic
Diagnostic • Bilirubin and alkaline phosphatase
testing levels rise several days later. reactions are not dose-dependent and
• Anti–hepatitis A serology have variable latent periods. examples of
idiosyncratic reactions include isoniazid,
• Supportive/expectant
Management chlorpromazine, and antiretroviral therapy.
• Spontaneous recovery in most patients

Drug-induced liver disease can also be broadly categorized according to morphology:  cholestasis (anabolic steroids), fatty liver
(eg, valproate), hepatitis ( isoniazid ), toxic or fulminant liver (eg, acetaminophen), or granulomatous (eg, allopurinol).

Crohn Disease Ulcerative colitis


• Anywhere mouth to anus (mostly ileum
& colon) • Rectum (always) & colon
Involvement
• Perianal disease with rectal sparing • Continuous lesions
• Skip lesions

Microscopy • Noncaseating granulomas • No granulomas

• Transmural inflammation • Mucosal & submucosal


Gross
• Linear mucosal ulcerations inflammation
findings
• Cobblestoning, creeping fat • Pseudopolyps

Clinical • Abdominal pain (often RLQ) • Abdominal pain (varying locations)


manifestations • Watery diarrhea (bloody if colitis) • Bloody diarrhea

Intestinal • Fistulae, abscesses


• Toxic megacolon
complications • Strictures (bowel obstruction)
Initial management of both CD and UC involves administration of 5-aminosalicylic acids and, frequently,
corticosteroids. Maintenance therapy for both diseases may involve azathioprine or antitumor necrosis factors.

Ascites fluid characteristics


• Bloody: trauma, malignancy, TB (rarely)
• Milky: chylous
Color
• Turbid: possible infection
• Straw color: likely more benign causes

Neutrophils • ≥250/mm3: peritonitis (secondary or spontaneous bacterial)

• ≥2.5 g/dL (high-protein ascites)


◦ CHF, constrictive pericarditis, peritoneal carcinomatosis, TB, Budd-Chiari
Total protein syndrome, fungal
• <2.5 g/dL (low-protein ascites)
◦ Cirrhosis, nephrotic syndrome

• ≥1.1 g/dL (indicates portal hypertension)


SAAG ◦ Cardiac ascites, cirrhosis, Budd-Chiari syndrome
• <1.1 g/dL (absence of portal hypertension)
◦ TB, peritoneal carcinomatosis, pancreatic ascites, nephrotic syndrome





Dyspepsia H pylori is a urease-


• Epigastric pain often described as "burning" producing organism that
Symptoms • ± Nausea, vomiting, epigastric fullness, commonly causes dyspeptic
heartburn symptoms. It colonizes the
stomach and induces excessive
• Functional (75%)
• Malignancy (eg, gastric, esophageal) production of gastric acid by
Etiology the parietal cells, which can
• Peptic ulcer, NSAIDs, Helicobacter pylori
infection, GERD eventually lead to duodenal
ulcer formation. Other
• Age ≥60: Upper endoscopy
• Age <60: common symptoms of
◦ Testing and treatment for H pylori duodenal ulcers include
Workup
◦ Upper endoscopy in high-risk patients (eg, nocturnal pain (due to
overt GI bleeding, significant weight loss, >1 circadian rhythm of gastric
alarm symptom)
acid secretion), worsening of
• Progressive dysphagia the pain with fasting,
• Iron deficiency anemia postprandial bloating, and
Alarm • Odynophagia nausea. Biopsy of the gastric
symptoms • Palpable mass or lymphadenopathy antrum during endoscopy can
• Persistent vomiting
• Family history of GI malignancy confirm infection.

16)Although both Achalasia and gastroesophageal reflux disease (GERD)


result from lower esophageal sphincter dysfunction and may cause dyspepsia, bleeding
is not typical. In addition, achalasia typically presents with weight loss, dysphagia for
solid foods, and regurgitation of undigested food. GERD presents with heartburn
(retrosternal burning pain) and much milder regurgitation.
Manifestations of sarcoidosis
• Hilar lymphadenopathy*
Pulmonary
• Interstitial infiltrates

• Papules, nodules & plaques


Cutaneous
• Erythema nodosum*

• Anterior & posterior uveitis


Ophthalmologic
• Keratoconjunctivitis sicca

• Facial nerve palsy


Neurologic • Central diabetes insipidus
• Hypogonadotropic hypogonadism

• AV block
Cardiovascular
• Dilated or restrictive cardiomyopathy

• Hepatosplenomegaly
Gastrointestinal
• Asymptomatic LFT abnormalities

• Hypercalcemia
• Peripheral lymphadenopathy
Other • Parotid gland swelling
• Polyarthritis*
• Constitutional symptoms (fever*, malaise)

*Manifestations of Löfgren syndrome.



Nonalcoholic fatty liver disease Acute Liver Failure


• Hepatic steatosis on imaging or biopsy Acute liver failure (ALF) is a serious condition
Definition • Exclusion of significant alcohol use characterized by severe acute liver injury in a patient
• Exclusion of other causes of fatty liver without cirrhosis or underlying liver disease. 

Severe hyperbilirubinemia is common in ALF and results


from a combination of impaired conjugation and impaired
excretion of bilirubin.  However, it is not a requirement for the
• Mostly asymptomatic diagnosis of ALF.
Clinical • Metabolic syndrome Decreased urine output (due to intravascular volume
features • ± Steatohepatitis (AST/ALT ratio <1) depletion and decreased renal perfusion) and elevated
• Hyperechoic texture on ultrasound portal venous pressure (due to increased resistance to
blood ow through the in amed liver) are both common in
ALF.  However, neither is speci c for ALF or part of the
diagnostic criteria.

Treatment
• Diet & exercise Liver transplantation.
• Consider bariatric surgery if BMI ≥35

20)Chronic alcohol Use is thought to potentiate acetaminophen hepatotoxicity by


depleting glutathione levels and impairing the glucuronidation process of NAPQI. On the
other hand, N-acetylcysteine increases glutathione levels and binds to NAPQI, so it is an
effective antidote for acetaminophen overdose when given early. Acute renal insufficiency is
common in ALF, especially when acetaminophen induced, due to the drug's direct renal
tubular toxicity.

21)Approximately only half of Patients with ALF will survive without liver
transplantation. Survival without liver transplantation is low in patients with acute liver
failure and indications that the disease is worsening or failing to improve. Reliable indicators
of worsening ALF include rising serum bilirubin and PT. Acute renal insufficiency, and the
degree of hepatic encephalopathy is also of prognostic importance as grade III hepatic
encephalopathy (characterized by marked confusion and incoherence) is associated with an
only 40%-50% chance of spontaneous recovery. Cerebral edema is a potential complication
of ALF that may lead to coma and brain stem herniation, and is the most common cause of
death.
In ALF due to acetaminophen toxicity, liver transplantation is firmly indicated in patients
with grade III or IV hepatic encephalopathy, PT >100 seconds, and serum creatinine >3.4 mg/
dL. One-year survival following liver transplantation for ALF is approximately 80%.
fl

fl

fi

Clinical features of irritable bowel syndrome Other common symptoms include


abdominal bloating, belching, atulence,
Recurrent abdominal pain/discomfort ≥1
day/week for past 3 months & ≥2 of the tenesmus, or a mucus discharge with
Rome IV following: stooling. 
diagnostic • Related to defecation (improves or Acute gastroenteritis can also induce
criteria worsens)
postinfectious IBS, which may last for
• Change in stool frequency
• Change in stool form months to years.

• Older age of onset (≥50) IBS is a clinical diagnosis and management


• Gastrointestinal bleeding
• Nocturnal diarrhea includes a limited workup (eg, complete
• Worsening pain blood count, serologies for celiac disease,
Alarm features • Unintended weight loss in ammatory markers) and reassurance. 
• Iron deficiency anemia
However, the presence of alarm features
• Elevated C-reactive protein
Positive fecal lactoferrin or calprotectin
suggests organic disease and requires

• Family history of early colon cancer or IBD further evaluation. 

Clinical manifestations of celiac disease

• Diarrhea, ± steatorrhea, weight loss Peripheral neuropathy, seen in up


• Abdominal pain to 50% of patients, and other
Gastrointestinal • Flatulence/bloating
• Late manifestations: ulcerative jejunitis,
neuropsychiatric manifestations may
enteropathy-associated T-cell lymphoma be due to autoantibody
production, rather than nutritional
• Dermatitis herpetiformis
Mucocutaneous
• Atrophic glossitis
de ciency, because they often precede
symptoms of malabsorption. E.g;
• Vitamin D deficiency Progressive paresthesia and sensory
Endocrine
• Secondary hyperparathyroidism
loss in a distal, symmetric, stocking-
• Osteomalacia/osteoporosis (adults) glove distribution,(length-
Bone disorders
• Rickets (children) dependent axonal
Hematologic • Iron deficiency anemia polyneuropathy)

• Peripheral neuropathy
Neuropsychiatric
• Depression/anxiety

Stool osmotic gap Secretory diarrhea occurs due to toxins (eg, produced by
Vibrio cholerae), hormones (eg, produced by VIPomas),
290 mOsm/kg − 2 x (stool Na + stool K)
congenital disorders of ion transport (eg, cystic brosis),
<50 Secretory diarrhea
or bile acids (in postsurgical patients).  It is caused by
secretion of electrolytes and water into the intestine, resulting
50-125 Indeterminate in a low SOG (<50 mOsm/kg).  The diarrhea is typically large
>125 Osmotic diarrhea in volume and persists while fasting and at night.

Secretory diarrhea can occur after bowel resection or cholecystectomy, when unabsorbed bile acids reach the
colon and result in the direct stimulation of luminal ion channels.  Resection of the ileocecal area additionally
reduces the ability of the intestines to actively absorb sodium ions against the electrochemical gradient.
fl
fi

fl

fi
Risk factors for colon cancer
• Frequent consumption of red/processed
Lifestyle
meat For individuals with a history of
factors
• Tobacco, alcohol use APR, colon cancer screening
should commence at an earlier age
• Personal/family history of adenomatous (eg, age 30-40) than that
polyps or colon cancer recommended for average-risk
• Inherited colon cancer syndromes (eg,
patients (eg, age 50).
Medical/family familial adenomatous polyposis, Lynch
history syndrome)
Colonoscopy is preferred,
• Ulcerative colitis although acceptable alternatives
• Diabetes/obesity include fecal occult blood testing
• Prior abdominopelvic radiation and fecal DNA testing.

Protective • High-fiber diet


factors • Aspirin/NSAID use

COLON CANCER SCREENING SURGERY INDICATIONS


Guidelines generally recommend annual Proctocolectomy should be performed in
Patients with classic FAP
screening sigmoidoscopies for children patients who initially present with CRC
can develop >1000 or adenomas with high-grade
starting at age 10-12, followed by annual
polyps and almost dysplasia.  Other indications for colorectal
colonoscopies once colorectal adenomas
universally develop surgery include severe symptoms from
are detected or if the patient is age >50.  colonic neoplasia (eg, hemorrhage) or
colorectal cancer (CRC)
Patients with the attenuated version of FAP can a signi cant increase in polyp number
if left untreated; some
have a delayed start of screening (age 25) during the screening interval. 
patients may have an
and longer screening intervals (1-2 years).  In patients with classic FAP who do not have
attenuated form with a In addition to colonoscopies, patients with FAP any of the above ndings, surgery does
slightly decreased risk should also undergo regular screening for not need to be performed urgently nd
of CRC.  upper gastrointestinal tract tumors. may be delayed until early twenties .

Achalasia
Clinical • Chronic dysphagia to solids & liquids, regurgitation
presentation • Heartburn, weight loss

• Manometry: ↑ LES resting pressure, incomplete LES relaxation, ↓ peristalsis of


distal esophagus
Diagnosis
• Barium esophagram: smooth "bird beak" narrowing at gastroesophageal
junction can be helpful in patients with nondiagnostic manometry.

• Upper endoscopy to exclude malignancy


Management • Laparoscopic myotomy or pneumatic balloon dilation
• Botulinum toxin injection, nitrates & CCB

28)Adenomatous polyps carry increased malignant potential and warrant


enhanced colonoscopic surveillance. Large polyps (>1 cm),High number (eg, >3
concurrent adenomas) and adenomas with high-grade dysplasia or villous features carry
an even greater malignant potential.
Prolonged isoniazid therapy can interfere with metabolism of tryptophan and
occasionally lead to pellagra.

fi

fi

29)Acute diverticulitis occurs due to inflammation associated with


microperforation of a diverticulum; it typically presents with left lower quadrant pain,
nausea/vomiting, and fever.CT ndings include focal thickening of the colon wall and
pericolonic fat stranding, diverticula are not always seen.Initial management includes
antibiotics(eg,ciprofloxacin plus metronidazole) and bowel rest.
Because colonic malignancy may mimic the presentation and CT findings seen in
diverticulitis, follow up colonoscopy (typically 4-8 weeks later) is often recommended to
rule out malignancy and evaluate the extent of diverticulosis.

30)Esophageal manometry and pH monitoring should be considered in patients with


persistent GERD symptoms or normal upper GI endoscopy to assess for other conditions (eg,
motility disorders) that can occasionally mimic GERD.
Helicobacter pylori can cause gastric and duodenal ulcers. Testing is indicated in
patients with active or past history of peptic ulcer disease. Routine screening and
empiric treatment for H pylori infection may also be considered for patients who have
dyspepsia but not GERD.

31)All patients with MALT lymphomas should be tested for H pylori infection
(It is present in 90% of patients with tumors) and patients with a positive result who
have early-stage MALT lymphoma should undergo H pylori eradication
therapy(eg,quadruple therapy) The majority of patients achieve complete remission
with antibiotic treatment. Patients with more advanced malignancies or with H pylori-
negative tumors should be considered for radiation therapy, immunotherapy (eg,
rituximab), or single-agent chemotherapy.
Although various autoimmune disorders such as Sjögren syndrome and Hashimoto
thyroiditis are associated with an increased risk for MALT lymphomas,
pernicious anemia is not. Instead, pernicious anemia is related to an increased risk of
gastric adenocarcinoma and gastric carcinoid tumors.

fi

Causes CLINICAL FEATURES


LI is a common cause of adult-onset diarrhea due to malabsorption and presents in adults age
20-40. Lactase enzyme deficiency is the most common cause of LI. Symptoms occur as the
osmotic load of undigested lactose passes through the intestines, drawing water into the lumen
Lactose and decreasing transit time. In addition, colonic bacteria ferment lactose, creating short-chain fatty
intolerance acids and hydrogen gas (bloating).
If the diagnosis is inconclusive (eg, atypical age or ethnicity), a lactose breath hydrogen test can be
performed.  Treatment consists of diet change or supplementation with lactase.

Insuf cient bile Insuf cient bile salt absorption by the terminal ileum can result in diarrhea in the immediate postoperative
salt absorption period after cholecystectomy, but symptoms typically resolve within a few weeks to months.

The use of proton pump inhibitors has been associated with Clostridium dif cile infections. 
USE OF PPI However, C dif cile infections are typically associated with acute onset of watery diarrhea and low-grade
fever.  Fecal occult blood testing is frequently positive.

Celiac disease results from an autoimmune reaction to the gluten-derived peptide gliadin, which results in
CELIAC destruction of enterocytes and villous atrophy in the small intestine.  Patients with celiac disease commonly report
Disease fatigue and foul-smelling, greasy diarrhea.

Small intestinal bacterial overgrowth (SIBO) occurs when bacteria from the colon are inappropriately present in
the small intestine.  The diarrhea is frequently greasy.  SIBO is usually associated with
SIBO underlying motility disorders (eg, diabetes mellitus) or anatomical abnormalities (eg,
jejunocolic stula). 

Exocrine Patients with exocrine pancreatic insuf ciency have impaired digestion of fat and protein and
Pancreatic symptoms of steatorrhea.  However, they typically have risk factors such as chronic pancreatitis,
Insufficiency cystic brosis, or a history of bowel or pancreatic resection

An increase in both fecal calprotectin and fecal leukocytes occurs in patients with in ammatory bowel
IBD disease, which causes abdominal pain but is associated with severe, watery diarrhea or hematochezia,
not steatorrhea.
fi
fi
fi
fi
fi

fi
fi
fl

GASTROENTEROLOGY:

Pedriatric:


BLOCK 20
Custom Id: 215184617


Malrotation with midgut volvulus


• Failure of normal embryonic gut rotation
Pathogenesis • Narrow mesenteric base allows for ↑ small bowel mobility
• Twisting of small bowel around superior mesenteric artery → gut ischemia & necrosis

• Most common in infancy (usually age <1 month)


• Acute: bilious emesis, abdominal distension(usually seen in infants)
Clinical
• Chronic: intermittent abdominal pain & vomiting, failure to thrive due to
features
intermittent volvulus (older children and adults)
• Untreated: hematochezia, peritonitis & shock

• Abdominal x-ray: ± dilated bowel, air-fluid levels, pneumoperitoneum


Diagnosis • Upper gastrointestinal series (gold standard): ligament of Treitz on
right; corkscrew, or bird's-beak, duodenum

• Emergency laparotomy (to relieve volvulus)


Treatment
• Ladd procedure (to reposition malrotated bowel)

Infant constipation
Functional Pathologic causes

• Introduction of solid foods • Down syndrome


Risk factors • ↓ Water intake • Abnormal physical findings (eg, displaced
• ↓ Fiber diet anus, tuft at gluteal cleft)

• Infrequent defecation
• Hard, painful stools
• Large-caliber or pellet-like stools
• Delayed passage of meconium
• ± Anal fissure
• Fever or vomiting
• Growth is normal, and physical
Clinical features • Ribbon stools
examination is often unremarkable. 
• Poor growth
However, mild abdominal distension and
• Severe abdominal distension
left lower quadrant rmness or palpable
stool may be present due to stool
retention. 

• Workup for serious organic cause


• Add undigestable, osmotically
• Hirschsprung disease (barium enema)
Management active carbohydrate (eg,
• Cystic fibrosis (sweat chloride test)
prune,pear or apple juice/puree)
• Spinal dysraphism (MRI)

fi

Delayed passage of meconium


Hirschsprung disease Meconium ileus

Pathophysiology • Failure of neural crest cell migration • Obstruction by inspissated stool

Level of obstruction • Rectosigmoid • Ileum

• Increased tone • Normal tone


Rectal examination
• Positive squirt sign* • Negative squirt sign*

Meconium • Normal • Inspissated


consistency
• Dilated proximal colon ± small bowel • Dilated small bowel
Imaging
• Narrow rectosigmoid • Microcolon

Associated disorder • Down syndrome • Cystic fibrosis

Meconium ileus
• Inspissated stool causes obstruction at
Pathophys
terminal ileum
iology
• Strong association with CF

• Failure to pass meconium within 24 hr of


Clinical birth
features • Abdominal distension
• No stool in rectal vault
• ± Bilious emesis
• X-ray: dilated loops of small bowel
Workup • Contrast enema: microcolon
• Diagnostic evaluation for CF (eg, sweat test)

• Hyperosmolar enema
Treatment
• ± Surgical management

Bicuspid aortic valve is common with Turner syndrome, which is associated with an increased risk of
in ammatory bowel disease (IBD).
fl

Necrotizing Enterocolitis
• Gut mucosal wall invasion by gas-producing
Pathogenesis bacteria
• Intestinal inflammation, necrosis

• Prematurity
Risk
factors • Very low birth weight (<1.5 kg [3.3 lb])
• Enteral feeding

• Nonspeci c: apnea, lethargy, vital sign instability


• Gastrointestinal
Clinical ◦ Abdominal distension
findings
◦ Feeding intolerance, bilious emesis
◦ Bloody stools The hallmark x-ray finding in NEC is air in
the bowel wall, or pneumatosis intestinalis
(red arrows). In addition, portal venous air
X-ray • Pneumatosis intestinalis (air in bowel wall)
(yellow arrows) is seen as branching areas
findings • Pneumoperitoneum (free air under diaphragm) of lucency over the liver due to gas-
producing bacteria and the transmigration of
• Sepsis, disseminated intravascular coagulation gas from the bowel wall to the mesenteric
Complications
• Late: strictures, short-bowel syndrome and portal veins.

Management of necrotizing enterocolitis


• Discontinuation of enteral feeds
Immediate • Nasogastric decompression
interventions • Blood cultures & empiric antibiotics
• Intravenous uid repletion

• Serial complete blood count & electrolytes


Monitoring
• Serial abdominal examinations & imaging

• Bowel perforation (pneumoperitoneum)


Indications Because bacteremia is common with NEC, blood cultures
• Clinical deterioration despite medical should ideally be obtained prior to administering antibiotics to
for surgery help guide therapy.  Response to therapy is monitored
management (suggestive of bowel necrosis) closely via serial blood work, abdominal examinations,
and imaging.



fi

fl


Celiac disease

• First-degree relative with celiac disease


Risk factors • Down syndrome
• Autoimmune disorders (eg, type 1 diabetes, autoimmune thyroiditis)

Classic
• ± Abdominal pain, distension, bloating, diarrhea
symptoms

• General: failure to thrive/weight loss, short stature,* delayed puberty/menarche*


Extraintestinal • Oral: enamel hypoplasia, atrophic glossitis
manifestations • Dermatologic: dermatitis herpetiformis
(may be sole • Hematologic: iron deficiency anemia (due to malabsorption)
presentation) • Neuropsychiatric: peripheral neuropathy, mood disorders (eg, anxiety, depression)
• Musculoskeletal: arthritis, osteomalacia/rickets* (due to vitamin D malabsorption),

• ↑ Tissue transglutaminase IgA antibody


Diagnosis
• Proximal intestinal biopsy (villous atrophy, crypt hyperplasia, intraepithelial
lymphocytosis)
• Gluten-free diet
Treatment
• Dapsone for dermatitis herpetiformis

*Pediatric findings.

254)Celiac Disease is a chronic malabsorptive disorder caused by an immune-mediated


hypersensitivity to gluten; patients with certain autoimmune diseases (eg, type 1 diabetes, autoimmune
thyroiditis) are at particularly high risk.  Pediatric patients often have growth delay, nonspeci c
gastrointestinal symptoms, and microcytic anemia.  The diagnosis can be con rmed serologically with anti-
tissue transglutaminase antibodies or by duodenal biopsy.

255)A Target sign on Ultrasound is diagnostic of intussusception and should prompt immediate
treatment with an air or water-soluble contrast enema.  Retrograde pneumatic (ie, air enema) or hydrostatic
(ie, contrast enema) pressure reduces the telescoped bowel in most cases.  Laparotomy is indicated if enema
reduction is ineffective, if a pathological lead point is identi ed, or if the patient has signs of perforation (eg,
free air on x-ray, rigid abdomen) .
256)Vitamin A de ciency usually manifests in the second or third year of life as impaired adaptation to
darkness (which may progress to night blindness), photophobia, dry scaly skin, dry conjunctiva (xerosis
conjunctiva), dry cornea (xerosis cornea) and a wrinkled, cloudy cornea (keratomalacia).  Bitot spots (dry, silver-
gray plaques on the bulbar conjunctiva) and follicular hyperkeratosis of the shoulders, buttocks, and extensor
surfaces are less common ndings.

fi
fi

fi

fi

fi

258)Button batteries create an external current that can lead to tissue corrosion.  In addition,
leaking alkaline battery solution causes liquefaction necrosis of surrounding mucosa.  As with any
lodged object, pressure necrosis can also occur because of local in ammation and ischemia. 
Prolonged impaction can result in esophageal ulceration and perforation, hemorrhagic shock, and
death.Patients are often initially asymptomatic but may experience nausea/vomiting, anorexia, and
chest pain.  Fever, hematemesis, and shock are signs of life-threatening perforation.

259)Ingestion of ≥2 magnets, especially if swallowed separately, can entrap bowel via


attraction across varying intestinal segments and lead to necrosis, stulae, or perforation.  Therefore,
magnets visualized in the esophagus or stomach on x-ray require immediate endoscopic
removal even if patients are asymptomatic. Magnets identi ed beyond the stomach can be
removed via colonoscopy or followed with serial x-rays (every 4-6 hr).  If symptoms (eg, abdominal
pain, gastrointestinal bleeding) develop or if the magnets do not progress in the gastrointestinal tract,
surgical removal is indicated.
An asymptomatic patient who ingested a small, blunt object (eg, coin) may be observed with
repeat x-rays after 24 hours to con rm progression of the object beyond the stomach.  Likewise,
although ingestion of a single magnet may cause bowel entrapment (via attraction to
external metal such as belts or buttons), observation and serial imaging can be considered because of
a relatively lower risk of complications.  Observation is contraindicated in a patient with ≥2
esophageal magnets due to the risk of necrosis and perforation.

fi
fi
fl
fi

Food protein–induced allergic proctocolitis


FPIAP is a non–IgE-mediated reaction that
• Non–IgE-mediated reaction usually occurs in response to proteins in
Pathophy • Eosinophilic inflammation of rectosigmoid cow's milk (eg, casein, whey), such as those
siology colon
• Common triggers: cow's milk & soy proteins in standard formula.  Because breast milk
contains proteins derived from the maternal
Risk • Eczema
factors • Family history of food allergies diet, FPIAP can also occur in breastfed infants.

• Age 1-4 weeks (up to 6 months) Although Meckel diverticulum classically manifests in
Clinical • Well appearing older children as painless hematochezia due to
features • Blood- &/or mucus-streaked stools (positive ectopic gastric tissue, bleeding in patients age
Hemoccult)
<6 months is exceedingly rare and typically is
• Clinical diagnosis confirmed by symptom associated with complications (eg, obstruction,
resolution after protein elimination perforation, intussusception).  Ill appearance and
Diagnosis irritability due to pain would be typical in these cases.
◦ Breastfed infants: restrict dairy (± soy)
&
from maternal diet Although antibiotics, a risk factor for Clostridioides
treatment
◦ Formula-fed infants: switch to hydrolyzed dif cile colitis, may be transmitted through breast milk,
formula C dif cile infection is rare in infants, who lack the
intestinal receptors to the toxin until
approximately age 2.  Moreover, other infectious
Prognosis • Tolerance of offending protein by age 1 symptoms (eg, fever, profuse diarrhea) would be
expected.

261)Hypervitaminosis A is usually due to the ingestion of excessive doses of vitamin A for several weeks or months. 
Symptoms include anorexia, pruritus, lack of weight gain, increased irritability, limitation of motion, tender
swelling of the bones, alopecia, seborrheic cutaneous lesions, ssuring of the corners of the mouth, increased intracranial
pressure, and hepatomegaly.
Cystic fibrosis
• Mutation (ΔF508) of CFTR gene
Pathogenesis
• Autosomal recessive

• Recurrent sinopulmonary infections


• Intestinal obstruction (eg, meconium
Clinical
ileus)
features
• Pancreatic insufficiency & diabetes
• Male infertility

• Elevated sweat chloride levels


Diagnosis • CFTR mutation on genetic testing
• Abnormal nasal potential difference

• Nutritional support
• Airway clearance
Management
• Antibiotic coverage (Staphylococcus
aureus, Pseudomonas aeruginosa)
A typical CF growth chart shows normal birth measurements with
subsequent failure to thrive, characterized by weight
deceleration crossing ≥2 major percentiles (eg, 25th, 10th)
followed by a deceleration in linear growth velocity. In addition to
malabsorption, recurrent sinopulmonary infections impair growth
due to poor appetite and increased metabolic needs during illness.

Newborn screening identi es only the most common CFTR mutations; therefore, CF should be considered in a patient with
failure to thrive and recurrent respiratory symptoms (eg, frequent coughing episodes, cough with bilateral rhonchi),
regardless of screening results.
fi
fi

fi

fi
VACCINATION
And
Prophylaxis


HIV-infected patients with a CD4 count >200/
mm3 should receive all the vaccines that an
otherwise healthy person requires.  They should
also receive the pneumococcal conjugate
vaccine (PCV13) followed by the pneumococcal
polysaccharide vaccine (PPSV23) at least 8 weeks
later, again after 5 years, and at age 65.
Human papillomavirus vaccine is
generally administered for patients age
11-26 and is not recommended age >45.

Vaccination for hepatitis A is recommended for adults who are at increased risk of contracting the virus such
as men who have sex with men and travelers to countries where hepatitis A is prevalent (eg,
contaminated food or water consumption).  It is also recommended for adults who have conditions (eg,
chronic liver disease) that increase the risk for severe complications. In the United States, the hepatitis A
vaccine became part of routine childhood vaccinations in 2006.
Recommended vaccines for patients with chronic liver disease

Tetanus Every 10 years


Pneumococcal vaccination with the 23-valent
In uenza Annually pneumococcal polysaccharide vaccine (PPSV23) is
recommended for all adults age ≥65 and for those
Pneumococcal At diagnosis & at age 65
age <65 with certain comorbid conditions that
Hepatitis A Initiate series if not immune increase risk of pneumococcal disease (eg, chronic liver,
lung, heart disease; diabetes mellitus; smoking). 
Hepatitis B Initiate series if not immune

Patients with chronic liver disease require the same immunizations as the general population in
addition to disease-speci c vaccines.
The in uenza vaccine is recommended annually for all adults.  The intramuscular inactivated in uenza
vaccine appears to be more effective than the live attenuated intranasal vaccine and is preferred.

Patients who receive solid organ transplantation require high-dose immunosuppressant


medication to prevent organ rejection.  This creates an immunocompromised state with a high risk for
opportunistic infection, most notably Pneumocystis pneumonia (PCP) and cytomegalovirus.
Other potential infections are also commonly addressed in the peritransplant period.  Vaccinations for
pneumococci and hepatitis B are typically given prior to transplant to ensure an adequate immune
response, and an inactivated, intramuscular in uenza vaccine is usually given yearly.  In uenza and
pneumococcal vaccines are preferred to oseltamivir and penicillin for prophylaxis.  Many
patients also receive prophylaxis with ganciclovir or valganciclovir for cytomegalovirus, depending on
the serostatus of the donor and recipient. Varicella zoster vaccines are not usually administered in
the immediate posttransplant period due to an increased risk of organ rejection and a reduced
ability to generate an immune response (because of immunosuppressive medication).
fl
fl
fi

fl

fl

fl

Meningococcal vaccination
Routine • Age 11-12: primary vaccination In general, a booster vaccine should be provided at age 16
schedule • Age 16: booster (up to age 21) if the primary vaccine was at age <16.

• Complement de ciency
High-risk • Asplenia Primary vaccination can also be given to adults
patients • College students in residential
age >18 who are at increased risk for invasive
(vaccinate if housing (age ≤21)
age ≥2 • Military recruits
meningococcal disease (eg, complement
months) • Travel to endemic area de ciency, functional or anatomic asplenia).
• Exposure to community outbreaks

Neisseria meningitidis postexposure prophylaxis Adolescents should receive the


quadrivalent meningococcal
• Household members
conjugate vaccine at age 11-12 with
• Roommates or intimate contacts
a booster at age 16. In addition, the
• Childcare center workers
Recommended meningococcal serogroup B vaccine
• Persons directly exposed to respiratory or oral
populations* can be considered for those age
secretions
• Person seated next to affected person for ≥8 hr (eg,
16-18. However, no vaccination confers
airline traveler) complete protection, and meningococcal
meningitis has a high rate of mortality. 
• Rifampin Therefore, close contacts of patients with
Prophylaxis
• Ceftriaxone invasive N meningitidis infection should
options
• Cipro oxacin (adults only) receive chemoprophylaxis, regardless of
*Regardless of vaccination status. vaccination status.

N meningitidis has risk of epidemic spread, and most secondary cases develop within 10 days of the initial
patient's diagnosis.  Risk of transmission occurs from 7 days prior to symptom onset until 24 hours
after initiating appropriate antibiotic therapy.Therefore, antimicrobial chemoprophylaxis against N
meningitidis should be given to asymptomatic close contacts, regardless of vaccination status, who
have had exposure during this period.  Healthcare workers with direct exposure to secretions (eg,
during intubation) should also be given prophylaxis.

Vaccination for yellow fever is recommended prior to travel to some countries in central Africa, many of which
require evidence of vaccination for entry.  However, contraindications to this live-attenuated vaccine include:
• Allergy to vaccine components (eg, eggs)
• AIDS (CD4 <200/mm3), certain immunode ciencies (including those associated with thymus
disorder), recent stem cell transplantation
• Immunosuppressive therapy (eg, TNF antagonists, high-dose systemic corticosteroids),
Individuals with these conditions who must travel to an endemic area should be provided with a
medical waiver (documented in an international certi cate of vaccination) and counseled on
mosquito-protective measures.
fi
fl


fi

fi

fi

IgM IgG HBV


HBsAg HBeAg Anti-HBs Anti-HBe
anti-HBc anti-HBc DNA

Acute HBV

Early phase + + + +++

Window phase + +

Recovery phase + + + Likely +

Chronic HBV carrier + +

Acute are of chronic HBV + Likely + + + +

Vaccinated for HBV +

Immune due to natural HBV


+ +
infection

Healthcare workers (HCWs) exposed to blood (ocular, percutaneous, or mucous membrane)


from a suspected or known HB patient should be considered for post-exposure prophylaxis.  HCWs
with previous HB vaccination and known antibody response usually require no post-exposure
prophylaxis.  However, many clinicians recommend HB booster vaccine in these patients.
HCWs without previous HB vaccination or adequate antibody response to the vaccine should receive
the complete HB vaccine series as soon as possible.  The rst dose of the vaccine is preferred
within 12 hours, with the next 2 doses according to the standard schedule.  Unvaccinated
HCWs exposed to an HBsAg-positive source patient should also receive the HB immune globulin
as soon as possible, preferably within 24 hours.

Recommended vaccines for asplenic adult patients All patients should receive
• Sequential PCV13 rst,folllowed by PPSV23 >8 wks later vaccines either ≥14 days
Pneumococcus
• Revaccination with PPSV23 5 years later & at age 65 before scheduled
splenectomy or ≥14 days
Haemophilus
• 1 dose Hib vaccine regardless of previous vaccination status after splenectomy. 
in uenzae

• Meningococcal quadrivalent vaccine Herpes zoster vaccination is not required for young
Meningococcus
• Revaccinate every 5 years adults who undergo splenectomy; reactivation with
this organism is not increased because the immune
response against viral pathogens is primarily
In uenza • Inactivated in uenza vaccine annually
cytotoxic T-cell (not antibody) mediated.

• HAV,HBV
Other vaccines
• Tdap once as substitute for Td, then Td every 10 years
fl
fl

fl

fl
fi

fi

The varicella vaccine is a live attenuated virus that causes a low virulence infection after
inoculation.  Patients with HIV with CD4 counts <200/mm3 may have dif culty clearing the attenuated
virus and should not receive vaccination.  Those with CD4 counts >200/mm3 who have low
titers should receive the varicella vaccine.
Varicella vaccination is typically given to children in 2 doses (age 12-15 months and 4-6 years). 
Adults with HIV should receive vaccination if titers are low (and CD4 count >200/mm3).
All patients with HIV should receive the inactivated vaccines (in uenza, tetanus-diphtheria-
pertussis) recommended for the general population as well as those recommended speci cally for
people with HIV (pneumococcal, hepatitis B).
Varicella is usually self-limited and mild, but serious complications include pneumonia, central nervous
system disease (eg, cerebellar ataxia), and aggressive skin infections.  These complications are more
common in adolescents and adults, particularly those who are immunocompromised or pregnant.
Neonatal varicella-zoster infection

• Fever
Clinical • Vesicular eruption (chickenpox)
features • Systemic involvement (eg, pneumonia,
hepatitis, meningoencephalitis)

Treatment • Acyclovir

• Isolate infant from varicella contact


• Administer varicella-zoster immune
Prevention globulin to infant if maternal
infection developed 5 days before
or 2 days after delivery

VZV is transmitted via aerosolized droplets and is Extremely😱 😳 Contagious.  Therefore,


postexposure prophylaxis (PEP) is recommended for exposed individuals who are not immune,
including patients who have not received 2 doses of VZV vaccine or who have not had con rmed
varicella infection. 
VZV vaccine is the recommended method of PEP for patients who are eligible for the vaccine.  However,
VZV vaccine is a live-virus vaccine and is contraindicated for patients age <1 year.
Antiviral therapy (eg, acyclovir) would be indicated if the infant develops varicella
disease.  However, antiviral therapy has not been shown to be effective for PEP; VZV vaccine or VZIG
(for patients who cannot receive vaccine) are the recommended agents.A single dose of VZV vaccine
does not provide suf cient protection to consider a patient immune.  For immunocompetent
individuals, Post Exposure Prophylaxis is limited to immunization with the live VZV vaccine.
However, for high-risk patients who cannot receive VZV vaccine, PEP is achieved by administering
passive immunization with varicella-zoster immunoglobulin (VZIG).  Patients eligible for VZIG
include neonates, pregnant women, and immunocompromised patients.  VZIG for neonatal PEP is
effective if maternal infection developed 5 days before to 2 days after delivery.

fi

fl
fi

fi

fi

Latest UW Notes for STEP2 CK - 2022


Features:
• Arranged system wise and then subject wise
• There are total 97 Blocks, Each BLOCK is made of 40 or some time 45 -50 different Questions hence
each BLOCK covers the NOTEs of those 40-50 questions,while the total 97 Blocks covers the whole
UWorld (3980 Qs)

• Custom BLOCK Id is mentioned with each BLOCK, so you can make Your own blocks in UWorld
according to the BLOCK id mentioned and read the respective BLOCK,s NOTES here

• Contains All UW tables edited and supplemented with UW Explanations


• All UW Diagrams/Pictures and algorithms annotated and simpli ed
• Some topics are supplemented with UPTODATE information,separate tables are there for these info so
that you can differentiate it form UW info

• Highlighted important info


• Most of the UW Tables are edited with related pictures and de cinet information added to the
tables from UW explanations

• These Notes will be updated regularly


• Notes of the Upcoming NEW Questions will be added on monthly basis

fi
fi

PULMONOLOGY:
MEDICINE
Notes by: Step1usmle59@gmail.com
https://www.reddit.com/u/USMLEe59/?
utm_source=share&utm_medium=ios_app
&utm_name=iossmf

A Normal lung is Resonant to Percussion, and


auscultation at the periphery demonstrates

Vesicular breath sounds that consist of a


Quiet Inspiratory Phase and an almost
Inaudible Expiratory Phase. 

BLOCK 22
Custom Id: 215892229


01)The diaphragm is the most important muscle for enlarging the thoracic cavity and creating
negative pressure during inhalation.  In COPD, the diaphragmatic attening and muscular
shortening caused by hyperin ation result in more dif culty in decreasing intrathoracic pressure
during inspiration and therefore increase the work of breathing.
Clinical features of asbestosis
• Prolonged asbestos exposure (eg, shipyard, mining)
Clinical • Symptoms develop ≥20 years after initial exposure
presentation • Progressive dyspnea, basilar ne crackles, clubbing
• Increased risk for lung cancer & mesothelioma

• Pleural plaques on chest imaging


Diagnostic
• Imaging, PFT & histology consistent with pulmonary
evaluation
brosis

02)Patients with ASBESTOSIS primarily develop progressive dyspnea over a period of months; cough,
sputum production, and wheezing are uncommon.  Typical chest imaging ndings in asbestosis include
interstitial thickening of the lower lung elds in a reticular (ie, net-like) pattern, and pleural plaques (the
hallmark of the disease). 

Pulmonary hypertension is de ned as a mean pulmonary arterial pressure ≥25 mm Hg at rest.  It


is seen following the narrowing of the precapillary vessels, the loss of the pulmonary capillary bed, or
passive backpressure from the post capillary vessels from any cause.  It can be classi ed as follows:

Group 1 Pulmonary arterial hypertension (PAH), which can be idiopathic (primary PAH) or due to


drugs, HIV, or connective tissue diseases

Group 2 Pulmonary hypertension due to left heart disease

Group 3 Pulmonary hypertension due to chronic lung disease or hypoxemia

Group 4 Pulmonary hypertension due to chronic thromboembolic disease

Group 5 Pulmonary hypertension due to other causes (eg, hematologic, systemic disorders) 

Dyspnea, weakness, and fatigue are early complaints, which may be followed by chest
Clinical pain, hemoptysis, syncope, or hoarseness (due to compression of the recurrent laryngeal
nerve).Chest x-ray would show enlargement of the pulmonary arteries with rapid
Features
tapering of the distal vessels (pruning). 
fi
fi

fl
fi

fi

fi
fl

fi
fi

04)Hypertrophic osteoarthropathy (HOA) is a condition where digital clubbing is


accompanied by sudden-onset arthropathy, commonly affecting the wrist and hand joints. 
Hypertrophic pulmonary osteoarthropathy (HPOA) is a subset of HOA where the clubbing and
arthropathy are attributable to underlying lung disease like lung cancer, tuberculosis,
bronchiectasis, or emphysema.
05)In Pneumonia, the alveoli become lled with in ammatory exudate, leading to hypoxemia due to
marked impairment of alveolar ventilation in the affected portion of the lungs.  The result is right-to-left
intrapulmonary shunting, an extreme form of ventilation/perfusion (V/Q) mismatch in which there is
perfusion of lung tissue in the absence of alveolar ventilation (V ≈ 0).  Depending on how much of the lungs are
affected by intrapulmonary shunting, the resulting hypoxemia may or may not correct with an increase
in the fraction of inspired oxygen (FiO2). 
Severe Pneumocystis jiroveci pneumonia in particular may cause widespread intrapulmonary shunting
with hypoxemia that is dif cult to correct with supplemental oxygen.
Causes of hypoxemia

A-a Corrects with


Examples
gradient supplemental O2?

Reduced PiO2 High altitude Normal Yes

CNS depression, morbid


Hypoventilation Normal Yes
obesity

Diffusion
Emphysema, ILD Increased Yes
limitation

Small PE, lobar


V/Q mismatch* Increased Yes
pneumonia

Large intrapulmonary
shunt
Diffuse pulmonary edema Increased No

Large dead-space Massive PE, right-to-left


Increased No
ventilation intracardiac shunt

*Caused by localized dead-space ventilation and/or intrapulmonary shunting.

06)Altered mental status in the setting of an acute exacerbation of chronic obstructive


pulmonary disease (COPD) raises suspicion for symptomatic hypercapnia and should be
promptly investigated with arterial blood gas analysis.  Despite tachypnea, some patients with COPD
retain CO2 during an exacerbation due to shallow breathing and high levels of ventilation-perfusion
mismatch leading to acute hypercapnic respiratory failure.
The clinical manifestations of symptomatic hypercapnia are predominantly neurologic.  Patients with
mild to moderate degrees of hypercapnia may have headaches or hypersomnolence, whereas
higher blood CO2 levels (eg, >75-80 mm Hg) may cause delirium, confusion, lethargy, and
eventually coma (CO2 narcosis) or seizures.
fi

fi

fl

Pulmonary function test


Restrictive lung disease
Normal Obstructive lung disease
(including obesity)
FEV1 >80%(of predicted) Decreased Decreased

FEV1/FVC >70% Decreased Normal to increased

FVC >80% (of predicted) Normal to decreased Decreased

Asthma vs COPD
Asthma COPD Late-stage COPD

FVC Normal/↓ Normal/↓ ↓/↓↓

FEV1 ↓ ↓ ↓↓

FEV1/FVC ↓ ↓ ↓↓

Bronchodilator
 Partially reversible/ Usually


Reversible
response nonreversible nonreversible

Hyperin ation, loss


Chest x-ray Normal Normal
of lung markings

DLCO Normal/↑ Normal/↓ ↓

Although asthma is typically diagnosed in younger individuals, many patients may be diagnosed later. 
Adult-onset asthma tends to be more severe and less likely to go into remission.
Both COPD and Asthma may show air ow obstruction, but only asthma shows complete reversibility of the
obstruction with bronchodilators.  Patients with COPD may show partial reversibility in early disease, but
complete reversibility is unusual.  The diffusion capacity of carbon monoxide (DLCO) may also be useful as a
differentiating feature.  The DLCO in asthma is usually normal or increased (severe asthma).  It is never
increased in patients with COPD (though it may be normal in early chronic bronchitis).

Causes of hemoptysis
Hemoptysis is de ned as any expectoration of blood
• Bronchitis originating from the lower respiratory tract, with a
Pulmonary • Lung cancer
wide spectrum from minimal blood streaking in
• Bronchiectasis
sputum to the presence of frank blood and/or clots. 
• Mitral stenosis/acute pulmonary
Cardiac
edema
Chronic bronchitis is de ned as a chronic productive
• Tuberculosis cough for >3 months in 2 successive years, and
• Lung abscess
Infectious
• Bacterial pneumonia cigarette smoking is the leading cause. 
• Aspergillosis

Hematologic • Coagulopathy Bronchiectasis refers to irreversible dilation and


destruction of bronchi, resulting in chronic cough and
• Pulmonary embolism
Vascular
• Arteriovenous malformation inadequate mucus clearance. 
Compared with chronic bronchitis, bronchiectasis is
Systemic • Granulomatosis with polyangiitis
disease • Goodpasture syndrome more likely associated with a history of recurrent
respiratory tract infections and chronic cough with daily
• Trauma
Other
• Cocaine use (inhalation)
production of copious mucopurulent sputum.
fl

fi

fi

fl

Diagnostic challenges in Anaphylaxis


• No prior allergic reactions (ie, rst episode)

Nonclassic

• Absence of hypotension, respiratory symptoms, or cutaneous signs

presentation • Protracted or biphasic presentation


• Preexisting conditions with similar symptoms (eg, asthma, COPD)

Challenging

• Ongoing physiologic shifts (eg, childbirth, hemodialysis)
clinical situations
• Impaired communication (eg, sedation, severe psychiatric illness)

• Directly activate mast cells (eg, opiates, NSAIDs)

Medication

• Interfere with epinephrine e ect (eg, β-blockers,
e ects
• α-adrenergic blockers)

Diagnostic criteria for Anaphylaxis


Anaphylaxis is likely if there is Rapid symptom onset & any 1 of the following criteria:

Skin/mucosa involvement (eg, hives, lip/tongue swelling)  & either


1 hypotension or respiratory distress
Involvement of ≥2 organ systems after exposure to a likely
allergen
2 •


Skin/mucosa (eg, hives, lip/tongue swelling)
Respiratory (eg, wheezing, stridor, dyspnea)
Cardiovascular (eg, hypotension, tachycardia, syncope)
• Gastrointestinal (eg, abdominal pain, vomiting, diarrhea)

3 Hypotension after exposure to a known allergen

Management of Anaphylaxis
Epinephrine is the most important treatment for anaphylaxis.  It has alpha-1 and beta-2 agonist effects that
quickly cause vasoconstriction, increasing blood pressure, and bronchodilation, improving wheezing. 
Epinephrine also decreases further systemic release of in ammatory mediators from mast cells and basophils,
halting anaphylaxis.  Intramuscular injection, performed rapidly, is the preferred route of administration and
can be repeated every 5-15-minutes (up to 3 doses) if symptoms persist.

• Epinephrine (most important):


◦ IM preferred, may be repeated (eg, 3 doses)

Immediate
 ◦ IV in severe/refractory cases


management • IV crystalloid & Trendelenburg positioning for hypotension
• Albuterol for bronchospasm

• Early intubation for upper airway obstruction


• H1/H2 antihistamines

• Glucocorticoids

Adjunct

• Glucagon for patients on beta blockers (reversal)
management • Hospital admission for severe initial presentation (eg, shock) or ongoing
symptoms despite treatment
ff

fi
ff

fl

10)Patients with Hodgkin lymphoma who are treated with chemotherapy and radiation are at
high risk for long-term, treatment-related sequalae, particularly cardiac disease, radiation-induced
hypothyroidism, and secondary malignancy later in life (generally >10 years after therapy). 
Solid tumors in the lung (especially in smokers), breast, and gastrointestinal system (eg,
colorectal, esophageal, gastric tumors) are most common.

11)Aspergillus typically infects a preexisting lung cavity caused by tuberculosis, sarcoidosis,


bronchial cysts, or neoplasm.  Manifestations include hemoptysis, cough, dyspnea, and fever. 
Imaging usually shows a cavitary mass with air in the periphery, not a solid lung lesion. 

12)Pulmonary tuberculosis typically presents with progressive fever/night sweats, cough,


hemoptysis, and weight loss.  Chest x-ray usually shows apical cavitary lesions.

13)The anticholinergic effect of oral antihistamine (eg, diphenhydramine) is useful in treating


chronic cough due to upper airway cough syndrome (postnasal drip), which is unlikely in the absence
of rhinorrhea.

14)Imaging in Aspiration Pnemonia reveals in ltrate in the lower lobes or right middle lobe
(aspiration while upright) or the posterior segment of the upper lobes (aspiration while recumbent). 
The infectious organisms are frequently oral ora (mixed aerobic and anaerobic).  Broad-spectrum antibiotics
with good anaerobic coverage (eg, clindamycin, amoxicillin-clavulanate) are the mainstay of treatment.

15)Pulmonary venous congestion occurs in patients with congestive heart failure.  Characteristic
chest x-ray ndings include cardiomegaly, cephalization of pulmonary vessels with prominent vascular
markings, and pleural effusions, which are not present in this patient.

16)Patients with chronic hypoventilation due to COPD, obesity hypoventilation syndrome, or


neuromuscular causes have gradual increases in PaCO2 that result in chronic respiratory acidosis; to
compensate, the kidneys increase HCO3- retention, creating a chronic secondary metabolic
alkalosis.  With an acute exacerbation of COPD, the PaCO2 can increase further.
Increased dead space ventilation is the main cause of hypercapnia in COPD and acts to worsen the
respiratory acidosis.

17)On average, smokers with AAT de ciency present in their 30s, whereas nonsmokers present
in their 40s.  AAT de ciency is frequently associated with liver disease, most commonly resulting in
neonatal hepatitis, cirrhosis, or hepatocellular carcinoma.
AAT de ciency should be considered in a number of situations, including in patients with:
• COPD at a young age (age ≤45)
• COPD with minimal or no smoking history
• basilar-predominant COPD
• history of unexplained liver disease
Diagnosis is con rmed by measuring serum AAT levels, and pulmonary function testing should also be
performed.  Treatment includes intravenous supplementation with pooled human AAT.
Bronchoprovocation testing with methacholine can be used to con rm asthma diagnosis in
selected groups of patients such as those with typical asthma symptoms but normal spirometry.
fi
fi
fi
fi

fl
fi

fi

fi

18)The most common etiologies of chronic cough(>8 weeks) include upper airway cough syndrome
(postnasal drip), gastroesophageal re ux disease, and asthma.  Other causes include drugs (ACE
inhibitors), airway disease (eg, nonasthmatic eosinophilic bronchitis, chronic bronchitis, bronchiectasis,
malignancy), and pulmonary parenchymal disease (eg, lung abscess, interstitial lung disease).Patient
having a chronic cough that worsens overnight and did not improve with antihistamine therapy should be
suspected of having asthma and therefore should be evaluated with pulmonary function tests
(spirometry) to assess bronchodilator response. 
A methacholine challenge test can be performed if no bronchodilator response is seen. 
Nocturnal or early morning peak expiratory ow rate measurements can be used to diagnose patients
with only nocturnal symptoms.An alternate approach is to treat empirically with 2-4 weeks of inhaled
glucocorticoids.  If the cough improves, a diagnosis of asthma can be made.
Myasthenic crisis A crisis occurs when increased disease activity causes
signi cant generalized muscle weakness, resulting in
• Infection or surgery diaphragmatic impairment and respiratory failure;
Precipitating • Pregnancy or childbirth bulbar weakness (eg, dysphagia) may also be a
factors • Tapering of immunosuppressive drugs prominent feature.  Patients with myasthenia gravis are at
• Medications (eg, aminoglycosides, CCB, beta particular risk for underrecognition of the severity of
blockers,Magnessium, and oroquinolones) respiratory impairment due to the inability to effectively
recruit weakened accessory muscles.  Patients may be near
Signs/ • ↑ Generalized & oropharyngeal weakness respiratory collapse by the time respiratory distress is
symptoms • Respiratory insuf ciency/dyspnea apparent.

Management commonly involves admission to the intensive care unit for close monitoring of
respiratory function (eg, vital capacity [VC]).  Elective intubation should be performed in patients with
evidence of impending respiratory failure, such as an ill-appearing patient with tachypnea, shallow
breaths, muscle weakness, and dif culty to clear secretions (eg, choking sensation).  Objective
Management
measurements that can also indicate impending respiratory failure include severely reduced VC
and respiratory acidosis.  To bridge to the slower-acting, steroid-sparing medications (eg,
azathioprine), patients should also receive rapid-acting therapy such as plasma exchange or
intravenous immunoglobulin and high-dose corticosteroids.

20)Lower Respiratory Tract involvement may lead to Tracheal Narrowing with Ulceration (highly
characteristic of GPA), and chest x-ray ndings of Multiple lung Nodules with Cavitation that can be
accompanied by Alveolar Opacities.
Patients typically also have anemia of chronic disease.  Diagnosis of GPA is made with antineutrophil
cytoplasmic antibodies and, de nitively, by tissue biopsy that can show granulomatous in ammation.
fi

fi

fl
fi
fi

fi

fl

fl

fl

Hypersensitivity pneumonitis
Etiology • Immunologic response to inhaled antigen (eg, mold, animal protein)

Acute
• Abrupt-onset fever, chills, cough, dyspnea, fatigue,hypoxemia, ne crackles, leukocytosis
• Episodes often recurrent and self-resolving
• Chest x-ray: Normal OR scattered micronodular interstitial opacities
Clinical presentation
Chronic
• Progressive cough, dyspnea, fatigue, weight loss
• Hypoxemia that worsens with exertion
• Chest x-ray: diffuse reticular interstitial opacities

• PFT: restrictive pattern, ↓ DLCO (chronic only)


• BAL: high relative lymphocyte count
Diagnosis
• Lung biopsy: lymphocytic in ltrate, poorly formed noncaseating granulomas, interstitial
in ammation or brosis (chronic only)

• Remove antigen exposure (resolves acute disease)


Treatment
• Glucocorticoids and/or lung transplantation (chronic only)

Manifestations of sarcoidosis

• Bilateral hilar adenopathy


Pulmonary
• Interstitial infiltrates

• Papular, nodular, or plaquelike lesions


Cutaneous
• Erythema nodosum

• Anterior uveitis (iridocyclitis or iritis)


Ophthalmologic • Posterior uveitis
• Keratoconjunctivitis sicca

• Peripheral lymphadenopathy
Reticuloendothelial • Hepatomegaly
• Splenomegaly

• Acute polyarthritis (especially ankles)


Musculoskeletal
• Chronic arthritis

• Atrioventricular block
Cardiovascular
• Dilated or restrictive cardiomyopathy Sarcoidosis is frequently detected incidentally on chest
• Facial nerve palsy x-ray, and bilateral hilar adenopathy is the rst
CNS/endocrine • Central diabetes insipidus
• Hypercalcemia manifestation of disease in 50% of patients.  It can also
• Erythema nodosum present with symptoms of cough, dyspnea, fever,
• Hilar adenopathy
Löfgren syndrome
• Migratory polyarthralgia fatigue, and weight loss.
• Fever
fl

fi

fi

fi
fi


BLOCK 23
Custom Id: 216341971


42)Comorbid gastroesophageal re ux disease (GERD) is common in patients with asthma and


can worsen asthma symptoms as a result of microaspiration. Clues suggesting comorbid GERD,
includes sore throat, morning hoarseness, worsening cough only at night, and increased
need for albuterol inhaler following meals. Other symptoms suggestive of GERD are dysphagia,
chest pain/heartburn, and sensation of regurgitation.  In asthma patients with signs and/or symptoms
suggestive of comorbid GERD, proton-pump inhibitor therapy has shown bene t in improving asthma
symptoms and peak expiratory ow rate.
Oral corticosteroids are used for acute asthma exacerbation (eg, dyspnea at rest, wheezing).
The anticholinergic effect of diphenhydramine is useful in the treatment of chronic cough caused
by rhinitis in patients with upper airway cough syndrome (UACS), previously known as
postnasal drip syndrome. (Rhinorrhea, sensation of something "dripping" into throat).
Aspirin-exacerbated respiratory disease occurs in patients with asthma and chronic rhinosinusitis. 
Symptoms include a sudden worsening of asthma and nasal congestion 30 minutes to 3 hours after
ingestion of nonsteroidal anti-in ammatories. 
Interstitial lung disease
• Sarcoidosis, amyloidosis, alveolar proteinosis
• Vasculitis (eg, granulomatosis with polyangiitis)
Common • Infection (eg, fungal, tuberculosis, viral pneumonia)
etiologies • Environmental exposure (eg, silicosis, HP)
• Connective tissue disease (eg, SLE, scleroderma)
• IPF, cryptogenic organizing pneumonia

• Progressive exertional dyspnea, dry cough Those without an identi able environmental,
Clinical
• >50% of patients have signi cant smoking history infectious, or autoimmune etiology,are typically
presentation
• Fine inspiratory crackles ± digital clubbing diagnosed with idiopathic pulmonary
brosis (IPF).
• Chest x-ray: reticulonodular interstitial opacities Hypercapnia (increased PaCO2) is typically a late
• HRCT: brosis, honeycombing, traction nding in ILD, in part because CO2 diffuses across
Diagnosis
bronchiectasis the alveolar-capillary membrane much more rapidly
• PFT: restrictive pattern with ↓ DLCO than O2. In ILD, hypoxemia, especially with exertion,
is more characteristically observed than
HP = hypersensitivity pneumonitis; HRCT = high resolution hypercapnia.

Multifocal atrial tachycardia


• Exacerbation of pulmonary disease (eg, COPD)
Etiology • Electrolyte disturbance (eg, hypokalemia)
• Catecholamine surge (eg, sepsis)

• Typically asymptomatic
Clinical
• Rapid, irregular pulse
ndings
• ECG: >3 P-wave forms & atrial rate >100/min

• Correct underlying disturbance


Treatment
• AV nodal blockade (eg, verapamil) if persistent
fi
fi
fi
fi

fi

fi

fl
fl

fl

fi

Characteristics of high-altitude pulmonary edema versus multifocal pneumonia

High-altitude pulmonary edema Multifocal pneumonia

• Recent arrival at high altitude (<1 week) • Persistent stay at high altitude
• Absent or mild leukocytosis • Leukocytes >15,000 mm3 with bands
• Procalcitonin normal • Procalcitonin elevated
• Marked early improvement with O2 • Minimal early improvement with O2

In response to the low partial pressure of inspired oxygen (PiO2) at high altitude, the lungs undergo
hypoxic vasoconstriction to optimize alveolar-capillary gas exchange.  Some individuals have a genetic
predisposition to developing HAPE because they experience unevenly distributed hypoxic
vasoconstriction, which exposes the pulmonary capillaries in relatively less vasoconstricted areas to high
perfusion pressure.  This causes regional disruption of the alveolar-capillary membrane and leads to characteristic
patchy, bilateral pulmonary edema (noncardiogenic pulmonary edema).
Common types of hypersensitivity pneumonitis
Patients with a history of atopy (eg, asthma,
Condition Causative agent
eczema) are more likely to develop HP, and
Farmer's lung
Moldy hay
smoking decreases the risk of developing
(thermophilic actinomycetes, Aspergillus spp)
HP, possibly due to blunting of the
Bird fancier's
pulmonary immunologic response.
Proteins in avian droppings or feathers
lung

Hot tub lung Mycobacterium avium complex, Cladosporium spp

Mushroom
worker's lung
Thermophilic actinomycetes
Potato riddler's
lung

Wine maker's
Botrytis spp
lung

Cheese
Penicillium spp, Aspergillus spp
maker's lung

Chemical
Spray paints, dyes, varnishes
worker's lung

Patchy interstitial inflammation Extensive network of interstitial inflammation and


47)Approximately 10% of patients with Pulmonary Embolism have occlusion of a peripheral


pulmonary artery by thrombus, causing pulmonary infarction.  These small peripheral thrombi are
more likely to cause pleuritic chest pain and hemoptysis, due to in ammation and irritation of the
lung parenchyma and adjacent visceral and parietal pleura.

Causes of hypercapnia

Mechanism Examples

↓ Central respiratory • Drugs (eg, opioids, benzodiazepines)


drive • CNS trauma, stroke, or encephalitis

↓ Respiratory • Spinal cord lesions


• Amyotrophic lateral sclerosis
neuromuscular function • Myasthenia gravis

• Obesity hypoventilation syndrome


↓ Thoracic cage or pleural
• Pneumothorax
function
• Rib fractures, ail chest

• Obstructive sleep apnea (upper airway)


Airway obstruction
• Chronic obstructive pulmonary disease (lower airway)

• Cardiogenic pulmonary edema


Impaired gas exchange
• Interstitial lung disease

48)Management of patients with acute benzodiazepine intoxication should rst involve


protection of the airway and ventilatory support as needed.  Flumazenil, a nonspeci c competitive
antagonist, is sometimes given as a reversal agent; however, it does not consistently reverse respiratory
depression and can potentiate seizures in patients who have developed a tolerance to
benzodiazepines.

49)Although viral pneumonia can rarely present as alveolar consolidation with bilateral lung crackles,
interstitial in ammation without alveolar consolidation (or lung crackles) is more typical. 
fl
fl

fl
fi

fi
50)Low pleural uid glucose also indicates an exudative pleural effusion.  Pleural uid glucose
<60 mg/dL is usually due to rheumatoid pleurisy, complicated parapneumonic effusion or empyema,
malignant effusion, tuberculous pleurisy, lupus pleuritis, or esophageal rupture.  (Pleural glucose
<30 mg/dL in particular suggests an empyema or rheumatic effusion.)  Glucose concentration in
empyema is decreased due to the high metabolic activity of leukocytes (and/or bacteria) in the uid.
51)PFT in chronic pulmonary thromboembolism typically demonstrates a normal
FEV1, FVC, and FEV1/FVC ratio. The DLCO is decreased due to ventilation/perfusion
mismatch and impaired gas exchange.
Diffuse alveolar hemorrhage
• Pulmonary capillaritis: ANCA vasculitis, SLE, antiphospholipid antibodies
Etiology & • Bland hemorrhage: mitral stenosis, anticoagulation
pathogenesis • Alveolar damage: viral pneumonitis, ARDS, drug-induced (eg, cocaine,
amiodarone)

• Dyspnea, hypoxemia, hemoptysis (absent in ~50%) & blood loss anemia


Clinical presentation &
diagnosis
• CXR or CT: diffuse ground-glass opacities
• Bronchoscopy: progressive blood on serial lavage

• Treat underlying (eg, rheumatologic, infectious) cause


Management
• Supportive care: oxygen, mechanical ventilation; avoid anticoagulation

Diffuse airspace opaci cation

De nition: Alveolar lling processes that can involve multiple lobes


• Transudative edema: cardiogenic
Fluid
• Exudative edema: noncardiogenic (ARDS)

• Neutrophils (pus): infectious pneumonia


• Lymphocytes & macrophages: hypersensitivity pneumonitis
Cells
• Eosinophils: acute eosinophilic pneumonia
• Malignant cells: carcinomatosis

• Diffuse alveolar hemorrhage


Blood
• Aspirated nonalveolar hemorrhage

• Fat: lipoid pneumonia (eg, vaping oils)


Others
• Protein: pulmonary alveolar proteinosis

52)Cocaine can be insuf ated (snorted) or inhaled.  Central stimulation of sympathetic out ow causes
hypertension, tachycardia, and mydriasis.  Burns occur due to handling of hot pipes (while nociception is
blunted) and are a speci c indicator of inhalant abuse.  When inhaled, cocaine undermines the blood-
alveolar barrier via thermal damage (scorching hot smoke), ischemia-reperfusion injury (intense
pulmonary vasospasm), and direct cellular toxicity.  This can result in DAH, a form of widespread
alveolar capillary bleeding.
fi
fl
fi
fi

fi
fl

fl
fl
fl

Common etiologies of chronic cough


For cough following upper
Upper airway • Upper airway cough syndrome (postnasal drip) respiratory infection (which can be
disorders • Chronic sinusitis due to UACS or effects of the virus), the
best approach is to treat empirically
• Asthma with an oral rst-generation
Lower airway • Post-respiratory tract infection antihistamine (eg, chlorpheniramine)
& • Chronic bronchitis or combined antihistamine-
parenchymal • Bronchiectasis decongestant (eg, brompheniramine
disorders • Lung cancer and pseudoephedrine).  This approach
• Nonasthmatic eosinophilic bronchitis differs from that used for symptoms
thought to be due to rhinitis, in which
• Gastroesophageal re ux
Other causes intranasal glucocorticoids are rst-line.
• ACE inhibitors

Obstructive sleep apnea


• Relaxation of pharyngeal muscles, leading to closure of airway
Pathophysiology
• Loud snoring with periods of apnea
• Daytime somnolence
• Nonrestorative sleep with frequent awakenings
Symptoms
• Morning headaches
• Affective & cognitive symptoms,impotence,depression
• Systemic hypertension
Sequelae
• Pulmonary hypertension & right-sided heart failure

54)Aspiration of gastric contents can occur with gastroesophageal reflux (GERD),


which can cause nighttime coughing or choking sensations and often occurs in obese
patients. However, GERD often causes reflux symptoms (eg, burning epigastric pain).

Breath sounds represent air movement within the lung and tactile fremitus is caused by tissue
vibration that occurs during air movement. 

fi

fl

fi

Initial pharmacotherapy for acute asthma exacerbation


Release of histamine and leukotriene
Class Medication Use causes immediate
bronchoconstriction as well as
• Bronchodilator
Short-acting increased vascular permeability (ie,
• Inhaled albuterol • Continuous (if severe) or
beta-2 agonist bronchial edema) and mucus
repeated dosing
production.  Hours after the
• Bronchodilator immediate response, a late-phase
• Inhaled ipratropium • Repeated dosing only in
Anticholinergic reaction often occurs due to continued
bromide acute setting & in
conjunction with albuterol mobilization of in ammatory cells (eg,
eosinophils).  This late-phase
• Prednisone, • Anti-in ammatory reaction is characterized by
methylprednisolone, • Delayed effect (6 hr)
Glucocorticoid recurrent bronchoconstriction
dexamethasone (oral/ • Multiday dosing for control
intravenous) of late-phase in ammation and in ammation.

Pulmonary changes in pregnancy

Pathophysiology • Progesterone-induced hyperventilation

• Dyspnea of pregnancy
Clinical features
• ↑ PaO2, ↓ PaCO2 (respiratory alkalosis)

• ↑ Minute ventilation (mostly via ↑ tidal volume)


Lung volumes • ↓ Residual volume & functional residual capacity
• Normal vital capacity & FEV1

56)Most women experience dyspnea during pregnancy due to the normal physiologic changes that
occur.  Progesterone likely triggers a sensation of dyspnea and stimulates the respiratory drive in the
medulla, resulting in increased minute ventilation, mostly via increased tidal volume and a slight
increase in respiratory rate.  Basilar lung compression by the gravid uterus decreases residual volume and
functional residual capacity. However, diaphragmatic excursion is not impaired and actually
increases, which, combined with increased chest wall expansion, allows for larger tidal volume.  Although
breathing takes place at a lower baseline lung volume (ie, lower FRC), vital capacity is unchanged.
fl
fl

fl

fl

57)Bilateral wheezing can occur in acute PE due to cytokine-induced bronchoconstriction in response to


hypoxia and infarction. Low-grade fever and mild leukocytosis may also occur with PE.
An associated pleural effusion is often present on the same side as the infarct (ie, exudative
effusion resulting from infarction-induced in ammation). 

58)Both short- and long-term use of nitrofurantoin can result in lung injury.  Short-term use
is associated with an acute hypersensitivity involving the lungs, whereas long-term use can lead to
interstitial lung disease. Acute nitrofurantoin-induced pulmonary injury is due to a hypersensitivity that
can present with fevers, shortness of breath, dry cough, and erythematous rash.  Symptoms usually
begin 3-9 days from medication initiation.  Basilar Crackles, Bilateral basilar opacities and Unilateral
pleural effusions are common.  Treatment involves cessation of nitrofurantoin.
fl

Latest UW Notes for STEP2 CK - 2022


Features:
• Arranged system wise and then subject wise
• There are total 97 Blocks, Each BLOCK is made of 40 or some time 45 -50 different Questions
hence each BLOCK covers the NOTEs of those 40-50 questions,while the total 97 Blocks
covers the whole UWorld (3980 Qs)

• Custom BLOCK Id is mentioned with each BLOCK, so you can make Your own blocks in UWorld
according to the BLOCK id mentioned and read the respective BLOCK,s NOTES here

• Contains All UW tables edited and supplemented with UW Explanations


• All UW Diagrams/Pictures and algorithms annotated and simpli ed
• Some topics are supplemented with UPTODATE information,separate tables are there for
these info so that you can differentiate it form UW info

• Highlighted important info


• Most of the UW Tables are edited with related pictures and de cinet information added to
the tables from UW explanations

• These Notes will be updated regularly


• Notes of the Upcoming NEW Questions will be added on monthly basis

fi
fi


RENAL/Urinary System
&
ELECTROLYTEs:
MEDICINE
Notes by: Step1usmle59@gmail.com
https://www.reddit.com/u/USMLEe59/?
utm_source=share&utm_medium=ios_app
&utm_name=iossmf

BLOCK 29
Custom Id: 218075649


01) Hypoalbuminemia is usually due to excessive albumin loss (eg, nephrotic syndrome, protein-
losing enteropathy) or decreased albumin synthesis (eg, cirrhosis or severe malnutrition).  It can cause
signi cant peripheral edema but usually does not cause pulmonary edema.  Alveolar capillaries have a
higher permeability to albumin at baseline (reducing oncotic pressure difference) and greater
lymphatic ow than skeletal muscle, protecting the lungs from edema.

02) Chronic alcohol use is associated with a high incidence of several electrolyte abnormalities,
of which hypomagnesemia is the most common (likely due to poor nutritional intake, alcohol-
induced renal losses, and diarrhea).  Hypomagnesemia commonly occurs together with hypokalemia
and is a well-known cause of refractory hypokalemia (hypokalemia that cannot be corrected with
potassium replacement). Intracellular magnesium is thought to inhibit potassium secretion by
renal outer medullary potassium (ROMK) channels in the collecting tubules of the kidney. 

Common medications that cause hyperkalemia


Medication Mechanism

ACE inhibitor, ARB Decreases aldosterone secretion (inhibition of AT II/AT II receptor) + inhibits ENaC

Cyclosporine Blocks aldosterone activity

Digitalis Inhibits Na+/K+-ATPase Corticosterone


Heparin Blocks aldosterone production

Nonselective Interferes with β2-mediated intracellular


β-blocker potassium uptake Aldosterone
Decreases renal perfusion → decreased potassium
NSAID
delivery to the collecting ducts

Potassium-sparing
Inhibits ENaC or aldosterone receptor Aldosterone Receptor
diuretic

Causes extracellular leakage of potassium through


Succinylcholine
acetylcholine receptors
ENaC
Trimethoprim Inhibits ENaC

03) Patients with chronic hyperkalemia may be asymptomatic until the potassium gradually rises >7.0
mEq/L.  However, acute hyperkalemia can cause symptoms at lower levels.  Patients may develop
ascending muscle weakness with accid paralysis and electrocardiogram (ECG) changes (eg, peaked T
waves, followed by short QT interval, QRS widening, and sine wave with ventricular brillation).
Initial evaluation of hyperkalemia includes ECG to evaluate for conduction abnormalities.  Acute
therapy (eg, calcium gluconate, insulin with glucose) is typically reserved for patients with ECG
changes, potassium >7.0 mEq/L without characteristic ECG changes, or rapidly rising
potassium due to tissue breakdown. 
TMP also competitively inhibits renal tubular creatinine secretion, which may cause an artificial increase in serum
creatinine; however, glomerular filtration rate is unchanged.

fi
fl

fl

fi

04) FSGS is the most common cause of nephrotic syndrome in adults, especially in African
Americans.  FSGS can also be associated with HIV, heroin use, and morbid obesity.
Minimal change disease is more common among children.  In adults, it is associated with use
of nonsteroidal anti-in ammatory drugs and lymphoma (usually Hodgkin).

05) Patients with acute rejection are usually asymptomatic, but they may have fever, decreased urine
output, or graft tenderness.  The process is usually recognized by acutely increased serum
creatinine, often accompanied by proteinuria.  Diagnostic con rmation requires renal biopsy, which
characteristically reveals lymphocytic in ltration of the intima with in ammatory tubular
disruption.  Intimal arteritis is often present as well.  Acute rejection is mostly reversible, and
treatment involves the administration of high-dose intravenous glucocorticoids, usually
accompanied by increased dosing of the patient's maintenance immunosuppression regimen to help
prevent further episodes.

BK virus reactivation likely results from excessive immunosuppression in renal allograft recipients and
leads to tubulointerstitial nephritis.  Patients typically experience an asymptomatic, acute increase
in serum creatinine just as they do in acute rejection; however, renal biopsy reveals
intranuclear inclusions and a mixed lymphocytic and neutrophilic in ltrate.

Acute toxicity to calcineurin inhibitors (eg, tacrolimus) involves vasoconstriction of the afferent and
efferent renal arterioles, leading to prerenal acute kidney injury and hypertension.  A blood urea
nitrogen/creatinine ratio >20 is expected, and renal biopsy is typically unremarkable.

06) Analgesic nephropathy is the most common form of drug-induced chronic renal
failure.  Papillary necrosis and chronic tubulointerstitial nephritis are the most common
pathologies seen.  Polyuria and sterile pyuria (WBC casts may also be seen) are early manifestations. 
Microscopic hematuria and renal colic may occur following sloughing of renal papilla.  Hypertension,
mild proteinuria, and impaired urinary concentration commonly occur as the disease advances.  In
severe cases, nephrotic range proteinuria can be seen.  Patients with chronic analgesic abuse are
also more likely to develop premature aging, atherosclerotic vascular disease, and urinary
tract cancer.

07) Volume overload and ascites are common complications in patients with decompensated
liver cirrhosis, and loop diuretics such as furosemide are a common treatment, often in combination
with spironolactone.Potential side effects include hypokalemia, metabolic alkalosis, and prerenal kidney
injury.
Hypophosphatemia is common in patients with alcohol use disorder and, when severe, can result in weakness,
rhabdomyolysis, paresthesias, and respiratory failure.  It does not directly contribute to hypokalemia.

fl

fi
fi
fl

fi

Metabolic acidosis
Type Normal anion gap Elevated anion gap

• Accumulation of unmeasured
Mechanism • Loss of bicarbonate
acidic compounds

• Severe diarrhea
• Lactic acidosis
• Renal tubular acidosis
• Diabetic ketoacidosis
• Excess saline infusion
Etiologies • Renal failure (uremia)
• Intestinal or pancreatic
• Methanol, ethylene glycol
stula
• Salicylate toxicity
• CAI & MRA diuretics

CAI = carbonic anhydrase inhibitor; MRA = mineralocorticoid receptor antagonist.

53) There are 3 major types of RTA, all of which cause NAGMA due to a net loss of HCO3−.
Type 1 (distal) RTA results from impaired H+ excretion by alpha-intercalated cells in the distal
tubule.  Hypokalemia is typically present due to reduced K+ reabsorption, and urine pH
>5.5 is expected due to markedly impaired capacity to acidify the urine.

Type 2 (proximal) RTA results from impaired HCO3− reabsorption in the proximal tubule, and
hypokalemia is usually present.  Because distal tubular H+ excretion is able to somewhat
compensate for the impaired proximal HCO3− reabsorption, urine pH is variable and often <5.5.

Type 4 RTA results from reduced aldosterone activity, leading to impaired H+ and K+ excretion in
the collecting duct.  In contrast to type 1 and type 2 RTA, hyperkalemia is typical.  The urine pH is
usually <5.5.

Increased gastrointestinal HCO3− loss (eg, severe diarrhea) is a common cause of NAGMA. 
While Intracellular shift of HCO3− occurs with infusion of excess sodium chloride and leads to
NAGMA.  These and most other etiologies of acidosis occur with intact distal tubular H+ excretion;
therefore, low urine pH (eg, <5.5) is expected as the kidneys are able to appropriately acidify the urine
and excrete excess acid.
fi

Systemic lupus erythematosus Headache is common among patients with


SLE; it can be a manifestation of CNS lupus
• Gradual symptom onset (along with cognitive abnormalities, seizures,
Clinical • Malar or discoid rash and psychiatric changes) but can also be due to
presentation • Joint, renal, serosal &/or neurologic unrelated conditions (eg, migraine or tension-
involvement type headaches).

• Anemia, leukopenia, thrombocytopenia


Laboratory
• Positive ANA, anti–double-stranded DNA, anti-Smith
abnormalities
• Low complement levels, increased immune complexes

Lupus nephritis occurs in half of patients with SLE; severity can range from asymptomatic hematuria to rapidly
progressive glomerulonephritis and end-stage renal disease.  Immune complexes composed of double-stranded
DNA (dsDNA) and anti-dsDNA antibodies deposit in the mesangium and/or subendothelial space, triggering an intense
in ammatory reaction with activation of the complement system, lowering C3 and C4 levels.  Immune complexes may
also deposit in the subepithelial space, leading to nephrotic syndrome without hypocomplementemia.

Metabolic effects of thiazide diuretics At lower doses, chlorthalidone is more


Adverse effect Mechanism effective than other thiazides and is
associated with a lower rate of heart failure
Inhibition of Na+/Cl− cotransporters in the distal
Hyponatremia
convoluted tubule
compared with other antihypertensive
agents (possibly due to its ability to achieve
Hypokalemia Compensatory rise in renin & aldosterone secretion lower blood pressure as a single agent).
Hypercalcemia Increased reabsorption in distal tubule The metabolic adverse effects of thiazide
diuretics are dose-dependent, therefore
Hyperglycemia Decreased insulin secretion & increased signi cant blood pressure reduction can be
Hypercholesterolemia
insulin resistance achieved with minimal side effects when
low doses are used.
Hyperuricemia Increased reabsorption in proximal tubule
fl
fi

Regulation of calcium metabolism


• ↑ Bone resorption
• ↑ Renal reabsorption of calcium & ↓
Parathyroid
reabsorption of phosphate
hormone
• ↑ Conversion of 25-hydroxyvitamin D to 1,25-
dihydroxyvitamin D

• ↑ Intestinal absorption of calcium &


phosphate
Vitamin D • ↑ Renal reabsorption of calcium & phosphate
• ↓ Parathyroid hormone secretion
• ↑ Mineralization of bone

• ↓ Bone resorption
Calcitonin
• ↓ Renal calcium reabsorption

Management of hypercalcemia Severe hypercalcemia (serum calcium


>14 mg/dL), especially with a rapid rise in
Short-term (immediate) treatment
serum calcium, can cause weakness,
• Normal saline hydration plus calcitonin
Severe gastrointestinal distress, and
• Avoid loop diuretics unless volume overload
(calcium >14 mg/dL)
(heart failure) exists neuropsychiatric symptoms (eg, confusion,
or symptomatic stupor, coma).  Patients are typically volume
Long-term treatment
• Bisphosphonate (zoledronic acid) depleted due to both polyuria from
hypercalcemia-induced nephrogenic diabetes
Moderate • Usually no immediate treatment required insipidus and decreased oral intake.
(calcium 12-14 unless symptomatic Patients with severe hypercalcemia require
mg/dL) • Treatment is similar to that for severe hypercalcemia aggressive saline hydration (ie, several
Asymptomatic or • No immediate treatment required liters of normal saline) to restore
mild • Avoid thiazide diuretics, lithium, volume intravascular volume and promote
(calcium <12 mg/dL) depletion & prolonged bed rest urinary calcium excretion. 

58) Hemodialysis is an effective treatment for hypercalcemia.  However, it is typically reserved


for patients with renal insuf ciency or heart failure in whom aggressive hydration cannot be
administered safely.
Glucocorticoids inhibit the formation of 1,25-dihydroxyvitamin D by activated mononuclear
cells in the lungs and lymph nodes.  Glucocorticoids can be used to treat hypercalcemia due to
excessive vitamin D intake, granulomatous diseases (eg, sarcoidosis), and certain
lymphomas. 

59) Low calcitriol (which corresponds to low serum calcium) and hypercalciuria in the setting of
high PTH (which corresponds to high renal calcium reabsorption) suggest intrinsic renal injury blocking
reabsorption (eg, distal renal tubular acidosis).



fi


RENAL/Urinary System
&
ELECTROLYTEs:
Pediatrics

BLOCK 32
Custom Id: 219467409

Urinary tract infection in INFANTS Escherichia coli is the most common etiology, followed by
other gram-negative bacteria (eg, Klebsiella, Proteus).  UTIs
• Females often present with vague symptoms in infants and should be
Risk • Uncircumcised males
suspected in a child with fever (especially temperature ≥39
factors • Vesicoureteral re ux
C [102.2 F]), fussiness, and poor feeding, as seen in this
• Constipation
patient.
• Fever Children are typically treated with a third-generation
Clinical
• Fussiness, poor feeding cephalosporin, such as ce xime, to cover the most
features
• Decreased urine output common pathogens.  Antibiotics may be modi ed based on
the urine culture results.
Laboratory • Pyuria
Although uoroquinolones treat E coli, use is limited in
ndings • Bacteriuria
children due to the risk of cartilage damage.  Cipro oxacin is
• Antibiotics reserved for treatment of UTIs caused by Pseudomonas
Management • Renal ultrasound (if febrile) aeruginosa, which typically occur in patients with an
• ± Voiding cystourethrogram indwelling catheter.

Urinary tract infection in CHILDREN Children age <2 years are at increased risk of
complications from UTIs (eg, renal scarring/damage,
• Female sex hypertension) and should be treated with 1-2 weeks
• Uncircumcised male infants
of antibiotics (eg, third-generation cephalosporin). 
Risk factors • Vesicoureteral re ux, anatomic defects
Dysfunctional voiding
In addition, all children age <2 with a rst febrile

• Constipation UTI should undergo a renal and bladder
ultrasound to evaluate for any anatomic
• Dysuria abnormalities (eg, urinary obstruction, vesicoureteral
Clinical • Fever re ux) that might predispose them to UTIs.  The
features • Suprapubic pain (cystitis) &/or ank/back pain
ultrasound should be performed after fever and
(pyelonephritis)
symptoms have resolved to minimize false positive
Laboratory • Pyuria results from acute in ammation.  If the patient has
ndings • Bacteriuria on urine culture persistent or worsening symptoms, an ultrasound
should be performed immediately to assess for renal
• Antibiotic therapy
Management abscess.
• ± Renal ultrasound & voiding cystourethrogram

A patient with a UTI,that has ndings of: Persistent fever and costovertebral tenderness despite
oral antibiotic therapy have a concern for renal infection, either pyelonephritis or renal
abscess.  The next steps in management are to broaden antibiotics to cover a resistant organism
and to obtain a renal and bladder ultrasound.  An ultrasound evaluates kidney involvement as
well as anatomic abnormalities that may predispose the patient to UTI (eg, hydronephrosis
suggestive of vesicoureteral re ux).
Children age >2 and adults generally do not need further evaluation of a rst-time UTI due to
the lower likelihood of predisposing anatomic issues, lower risk of complications, and
lower risk of recurrent UTI.
A voiding cystourethrogram is appropriate if hydronephrosis or scarring is seen on renal ultrasound.  It
is also indicated in children with recurrent UTIs or a rst febrile UTI from an organism other than E coli.
fi
fi
fl

fl


fl
fl

fl

fl
fi

fi

fl

fi

fi
fl
fi
fi

Latest UW Notes for STEP2 CK - 2022
Features:
• Arranged system wise and then subject wise
• There are total 97 Blocks, Each BLOCK is made of 40 or some time 45 -50 different Questions
hence each BLOCK covers the NOTEs of those 40-50 questions,while the total 97 Blocks
covers the whole UWorld (3986 Qs)

• Custom BLOCK Id is mentioned with each BLOCK, so you can make Your own blocks in UWorld
according to the BLOCK id mentioned and read the respective BLOCK,s NOTES here

• Contains All UW tables edited and supplemented with UW Explanations


• All UW Diagrams/Pictures and algorithms annotated and simpli ed
• Some topics are supplemented with UPTODATE information,separate tables are there for
these info so that you can differentiate it form UW info

• Highlighted important info


• Most of the UW Tables are edited with related pictures and de cinet information added to
the tables from UW explanations

• These Notes will be updated regularly


• Notes of the Upcoming NEW Questions will be added on monthly basis

fi
fi


HEMATOLOGY
&
ONCOLOGY:
MEDICINE
Notes by: Step1usmle59@gmail.com
https://www.reddit.com/u/USMLEe59/?
utm_source=share&utm_medium=ios_app
&utm_name=iossmf

BLOCK 33
Custom Id: 220040788

01)MULTIPLE MYELOMA is a plasma cell neoplasm.  Pathology stems from bone marrow
in ltration (fractures, hypercalcemia, anemia) and monoclonal protein production (renal
insuf ciency).  Serum protein electrophoresis (SPEP) is a common screening test for MM that
detects elevated serum monoclonal protein (M-spike).  Bone marrow biopsy, a more invasive
procedure, can then con rm diagnosis (>10% clonal plasma cells).
Bone scans are not useful in the diagnosis of MM as they detect only osteoblastic activity (as is
often seen in metastatic solid tumors).  In MM, the lesions are osteoclastic (lytic), so whole body cross-
sectional bone imaging (eg, low dose CT scan), is needed to identify bone involvement.

02)Chronic myeloid leukemia (CML) is a myeloproliferative neoplasm caused by the BCR-ABL


fusion gene.  It is marked by a dramatic leukocytosis (often but not always >100,000/mm3), absolute
basophilia, and a preponderance of early immature neutrophil precursors (promyelocytes,
myelocytes).CML is often found on routine blood work in asymptomatic patients but may present
with fatigue, weight loss, night sweats, or abdominal fullness (from splenomegaly). 
Thrombocytosis and anemia are common.
Auer rods are seen in many forms of acute myeloid leukemia (AML).  AML tends to present
relatively acutely with manifestations related to pancytopenia such as bleeding or infection.  In
addition, myeloblasts would be expected on peripheral smear examination.

03)The anemia of lymphoproliferative disorders, such as leukemia and lymphoma, is due to the
replacement of RBC progenitor cells with cancer cells in the bone marrow.  Treatment of the
malignancy may improve the patient's anemia, though many chemotherapeutic agents are
myelosuppressive.

04)Hodgkin lymphoma is generally treated with combination chemotherapy and radiation therapy. 
Although >75% of patients are cured, treatment can lead to long-term complications, most notably
secondary malignancy (eg, solid organ, hematologic i.e breast and lung) and cardiovascular disease.
Secondary malignancies are the leading cause of death in those who have been cured of HL. Other
common treatment-related complications include pulmonary disease (eg, brosis, bronchiectasis) and
hypothyroidism from chest radiation and neuropathy from chemotherapy.
fi
fi

fi

fi

05)Sideroblastic Anemia can be congenital (presenting in children or adolescents) or acquired (most


common).  Although acquired sideroblastic anemias may be related to clonal abnormalities (eg,
myelodysplastic syndrome), they are frequently due to reversible etiologies (eg, alcohol,
malnutrition, medications) that lead to pyridoxine-dependent impairment in the early steps of
protoporphyrin synthesis within mitochondria.
Sideroblastic anemia frequently manifests as microcytic hypochromic anemia, simulating iron de ciency
anemia. As can sometimes be seen with iron de ciency anemia, usually 2 groups of red blood cells (RBCs)
can be demonstrated on microscopy: hypochromic and normochromic ("dimorphic" RBC population),
likely re ecting the defective heme synthesis.
Unlike iron de ciency anemia, iron studies typically reveal increased serum iron concentration and normal
or decreased total iron binding capacity due to mitochondrial mishandling of iron (explaining the
mitochondrial iron granules seen in ring sideroblasts).

Drug-induced immune-mediated hemolytic anemia

• Drug coats erythrocytes → IgG binding → splenic


destruction of RBCs (extravascular hemolysis) Most patients have normocytic anemia
Pathogenesis • Drug triggers immune complexes → complement- with a normal or slightly elevated mean
mediated destruction of RBCs (intravascular corpuscular volume due to the presence of
hemolysis) reticulocytes.  Splenomegaly can also
• Sudden onset (within hours of exposure) occur due to increased red blood cell
• Anemia: fatigue, pallor, dyspnea sequestration in the spleen.
• Hemolysis: jaundice, dark urine, abdomen or The diagnosis is made with direct
back pain
Manifestations antiglobulin (Coombs) testing, which
◦ ↑ Reticulocytes, indirect bilirubin & LDH
◦ ↓ Haptoglobin identi es IgG or C3 (a complement
◦ Spherocytes on peripheral blood smear fragment) attached to the erythrocyte
◦ + Direct Coombs test (anti-IgG, anti-C3) surface. 
• Discontinue offending drug
Treatment • Transfusion (if severe)
• ± Glucocorticoids, IVIG

07)Patients with HIV are at a dramatically elevated risk of malignancy due to immune
dysregulation and impaired cytokine signaling.  This limits the ability of patrolling cytotoxic T-cells to induce
apoptosis in cells that have accumulated genomic mutations.  It also promotes reactivation of latent
oncogenic viruses, which can cause AIDS-de ning malignancies such as invasive cervical carcinoma
(human papillomavirus), Kaposi sarcoma (human herpesvirus 8), CNS system lymphoma (Epstein-Barr
virus [EBV]), and NHL. Approximately 10% of patients with AIDS develop NHL; nearly all cases in this
population are linked to the reactivation of EBV and the subsequent production of oncogenic viral
proteins.  Major manifestations include diffuse LAD, which may involve the epitrochlear, supraclavicular, and
mediastinal (enlarged cardiac silhouette) lymph nodes.  B symptoms (eg, night sweats, fevers, weight loss)
are generally present.  Diagnosis is usually made by excisional lymph node biopsy.

Human papillomavirus is oncogenic and contributes to invasive cervical and esophageal cancers.  It has tropism for
epithelial cells, not B-lymphocytes, so it does not increase risk of lymphoma.
fi

fl

fi

fi

fi

fi
08)Patients with chronic autoimmune diseases (eg, SLE, Sjögren syndrome) are at
increased risk for NHL due to persistent B-cell stimulation, immune dysregulation, and
exposure to immunosuppressive agents (eg, hydroxychloroquine).  NHL often presents with
progressive, painless lymphadenopathy and B symptoms (eg, weight loss, fever, night sweats). 
Laboratory evaluation often reveals elevated LDH, likely due to tumor burden, liver involvement,
or occasional hemolysis.  Con rmation is usually made by excisional lymph node biopsy.
Although the risk of most lymphoproliferative disorders is elevated in patients with
autoimmune disease, acute lymphoblastic leukemia generally manifests in children, not adults. 
Bone pain is often a prominent complaint.  In addition, complete blood count usually shows
signi cant neutropenia and thrombocytopenia, with a white blood cell count that can be
decreased or markedly elevated.

Immune thrombocytopenia
ITP is an autoimmune disorder
• Platelet autoantibodies in which IgG autoantibodies
Etiology
• Preceding viral infection against platelet membrane
glycoproteins cause increased
Clinical • Petechiae, ecchymosis
platelet destruction and
ndings • Mucosal bleeding (eg, epistaxis, hematuria)
inhibition of platelet
Laboratory • Isolated thrombocytopenia <100,000/mm3 production.
ndings • Few platelets (size normal to large) on peripheral smear Other than bruising, physical
examination is generally
• Children
unremarkable.  Patients have a
◦ Observe if cutaneous symptoms only
normal liver span (eg, 6-12
◦ Glucocorticoids, IVIG, or anti-D if bleeding
Treatment cm at midclavicular line) and
• Adults
nonpalpable spleen.
◦ Observation if cutaneous symptoms AND platelets ≥30,000/mm3
◦ Glucocorticoids, IVIG, or anti-D if bleeding or platelets <30,000/mm3

Clinical features of androgen abuse


• Exogenous (eg, testosterone replacement therapy)
Types of androgens • Synthetic (eg, stanozolol, nandrolone)
• Androgen precursors (eg, DHEA)

• Reproductive
◦ Men: decreased testicular function & sperm production, gynecomastia
Side effects/ ◦ Women: acne, hirsutism, voice deepening, menstrual irregularities
clinical presentation • Cardiovascular: left ventricular hypertrophy, possible ↓ HDL & ↑ LDL
• Psychiatric: aggressive behavior (men), mood disturbances
• Hematologic: polycythemia, possible hypercoagulability

10)Polycythemia vera (PV) is a chronic myeloproliferative disorder that causes erythrocytosis.  It is often
asymptomatic, but aquagenic pruritus (itching after a warm bath or shower), bleeding, transient neurologic
symptoms, and erythromelalgia (burning cyanosis of the hands or feet) are common presenting symptoms. 
fi
fi
fi

fi


Thrombotic thrombocytopenic purpura TTP is life threatening and must
be treated emergently with
• ↓ ADAMTS13 level → uncleaved vWF multimers →
plasma exchange (PEX).  PEX
Pathophysiology platelet trapping & activation
removes the patient's plasma
• Acquired (autoantibody) or hereditary
and replaces it with donor
• Hemolytic anemia (↑ LDH, ↓ haptoglobin) with plasma.  This replenishes
schistocytes ADAMTS13 and removes the
• Thrombocytopenia (↑ bleeding time, normal PT/PTT) autoantibodies.  Without
Sometimes with: emergent PEX, the mortality
Clinical features
• Renal failure rate is approximately 90%.
• Neurologic manifestations (eg, headache,
confusion, coma, stroke) Cryoprecipitate contains
• Fever clotting factors, brinogen,
• Plasma exchange and von Willebrand factor. 
Management • Glucocorticoids It is often used in patients
• Rituximab with DIC. 

Glucose-6-phosphate dehydrogenase de ciency

• Hemolytic anemia due to oxidative stress (infection, sulfa Peripheral blood smear typically
Epidemiology drugs, fava beans) reveals Heinz bodies
• X-linked: Asian, African, or Middle Eastern descent (oxidized/precipitated
hemoglobin) and bite cells
• Pallor & fatigue
(bite-shaped membrane defects
Manifestations • Dark urine, jaundice & icterus
due to removal of Heinz bodies
• Abdominal/back pain
by splenic macrophages).
• Hemolysis: ↓ hemoglobin, ↓ haptoglobin, ↑ bilirubin &
LDH, ↑ reticulocytes
Laboratory • Peripheral smear: bite cells & Heinz bodies
ndings • Negative Coombs test
• ↓ G6PD activity level (may be normal during
attack)

• Remove or treat responsible agent/condition


Management
• Provide supportive care

12)G6PD activity testing can be normal (low sensitivity) in acute hemolytic episodes because older
erythrocytes (lowest G6PD activity) have been hemolyzed and young erythrocytes/reticulocytes (higher
G6PD activity) are disproportionally elevated.  Therefore, in such settings, this test cannot be used to
rule out G6PD de ciency; repeat testing should be done at 3 months to con rm the diagnosis
in those whose initial test was negative.  No intervention is generally required other than
removing or treating the inciting oxidative stress.
fi

fi
fi

fi


fi
Hemophilia A & B
The most common sites of
Inheritance • X-linked recessive bleeding are the joints (80%),
• Delayed/prolonged bleeding after mild trauma especially the knee. 
◦ Hemarthrosis, intramuscular hematomas Hemarthrosis presents with
Clinical
◦ Gastrointestinal or genitourinary tract bleeding joint pain and swelling after
features
◦ Intracranial hemorrhage
minimal or no trauma, and
• Complications: hemophilic arthropathy
episodes typically begin during
• ↑ Activated PTT toddlerhood when the child is
Laboratory • Normal platelet count & PT
ambulatory.  Hemorrhage into the
ndings • Absent or ↓ factor VIII (hemophilia A) or factor
IX (hemophilia B) activity skeletal muscle (ie, hematoma)
after minor trauma is also
• Factor replacement
Treatment common.
• Desmopressin for mild hemophilia A

Iron de ciency anemia & thalassemias


Parameter Iron de ciency anemia Alpha-thalassemia minor Beta-thalassemia minor

MCV ↓ ↓ ↓

RDW ↑ Normal Normal

RBCs ↓ Normal Normal

Peripheral smear Microcytosis, hypochromia Target cells Target cells

Serum iron ↓ Iron & ferritin Normal/↑ iron & ferritin Normal/↑ iron &
studies ↑ TIBC (RBC turnover) ferritin (RBC turnover)

Response to iron supplementation ↑ Hemoglobin No improvement No improvement

Hemoglobin electrophoresis Normal Normal ↑ Hemoglobin A2

Microcytic anemia (eg, mean corpuscular volume [MCV] <80


µm3) is most often due to iron de ciency or thalassemia
minor.  Although both can cause hypochromia and target
cells.Laboratory ndings in thalassemia minor typically
demonstrate mild hematocrit reduction (>28%),
signi cantly low MCV (55-75 µm3), and
disproportionally elevated red blood cell (RBC) counts. 
The disproportionately elevated RBC count (eg, normal or
high rather than low) and decreased MCV help distinguish
thalassemia minor from iron de ciency anemia, which does not
typically cause microcytosis until hemoglobin is <10 g/
dL. Patients with thalassemia minor show no improvement in
hemoglobin with iron supplementation and do not require
speci c treatment.
fi
fi
fi
fi

fi


fi

fi
fi

Heparin Induced Thrombocytopenia Heparin induces a conformational change to a


platelet surface protein (platelet factor 4 [PF4]),
which exposes a neoantigen.  In patients with
type 2 HIT, the immune system responds by
forming an IgG autoantibody (HIT antibody)
that then coats the surface of platelets and forms
complexes (heparin-PF4-HIT antibody), resulting in:
• Thrombocytopenia
• Arterial and venous thrombus
Typically, type 2 HIT manifests with a >50% drop in
platelets 5-10 days after the initiation of heparin,
but it may occur earlier (sometimes <1 day) in
patients previously exposed. 
Diagnosis is con rmed by functional assay (eg,
serotonin-release assay [gold standard]) or
high-titer immunoassay of the blood.

16)Most cases of Traumatic Macrovascular Hemolysis arise in the setting of a dysfunctional


mechanical or severely calci ed aortic valve; high pressure gradients across the irregular valve cause
shearing of passing erythrocytes, leading to intravascular hemolysis.  Damaged erythrocytes that are not
lysed in the intravascular space are generally destroyed by splenic macrophages, leading to concurrent
extravascular hemolysis(Hepatospelnomegaly).Patients also frequently have mildly low platelets due to
concurrent mechanical platelet injury.  A transthoracic echocardiogram should be performed to
visualize valve function and determine valvular pressure gradients.

Delayed hemolytic transfusion reaction The antigen is minor (non-ABO) and


typically causes a low, often undetectable
• Anamnestic antibody response: antibody production against
Pathogenesis
minor RBC antigen to which patient was previously* exposed antibody response after initial exposure
(ie, alloimmunization, similar to that seen
• Onset >24 hours to a month after transfusion in a Rhesus-negative primigravida with a
• Often asymptomatic Rhesus-positive fetus).  However, on
Clinical
• Fatigue, dyspnea, tachycardia
ndings subsequent exposure via repeat
• Jaundice
Low-grade fever
transfusion, an anamnestic antibody

response occurs in which memory B
• Hemolytic anemia (present at baseline in patient with SCD) cells rapidly produce more
Laboratory ◦ ↑ indirect bilirubin, LDH, reticulocyte count antibodies; these antibodies bind to
ndings ◦ ↓ hemoglobin, haptoglobin donor RBCs, causing extravascular
• New positive direct antiglobulin (Coombs) test hemolysis.Most patients are asymptomatic. 
Management • Supportive (eg, uids) However, sickle cell disease patients are
more likely to have symptomatic, severe
• Review transfusion history & prior antibody screens anemia due to sickle cell-related
Prevention • Transfuse when necessary with extended-antigen intravascular hemolysis in addition to
cross-matched blood DHTR-related extravascular hemolysis. 

*Prior transfusion (eg, sickle cell disease patients), pregnancy, transplant.


fi
fi


fl
fi

fi

Common causes of macrocytic anemia* Because vitamin B12 serves as a crucial cofactor for DNA
synthesis, de ciency often disproportionately affects
• Folate de ciency rapidly dividing cells; common manifestations include:
• Vitamin B12 de ciency • Megaloblastic anemia (eg, macrocytosis,
• Myelodysplastic syndromes
hypersegmented neutrophils, anemia, mild
• Acute myeloid leukemias
leukopenia/thrombocytopenia) due to
• Drug-induced (eg, hydroxyurea, zidovudine,
chemotherapy agents) impaired hematopoiesis
• Liver disease • Glossitis due to impaired replication of
• Alcohol abuse gastrointestinal epithelium
• Hypothyroidism The diagnosis of pernicious anemia can be con rmed by
the detection of anti–intrinsic factor antibodies; treatment
*Mean corpuscular volume >100 μm3. with vitamin B12 supplementation is generally curative.

18)Patients with hypothyroidism from autoimmune thyroiditis usually develop normocytic,


normochromic anemia (due to bone marrow hypoproliferation).  Occasionally, mild macrocytic
anemia can occur, but no megaloblastic changes would be present.  Most cases arise in young
women.

Treatment of acute deep venous thrombosis/pulmonary embolism


Oral factor Xa inhibitors Warfarin

Mechanism of action Direct factor Xa inhibition Vitamin K antagonism

Therapeutic onset 2-4 hours 5-7 days

Overlap needed? No Yes, overlap with UFH or LMWH for ~5 days

Laboratory monitoring No Prothrombin time/INR

LMWH = low-molecular-weight heparin; UFH = unfractionated heparin.

19)Oral direct factor Xa inhibitors (eg, rivaroxaban, apixaban) are as effective as warfarin in the
treatment of acute DVT or PE and do not increase the risk of bleeding complications.  These drugs
have the advantage of rapid onset of action, no requirement for laboratory (eg, INR) monitoring,
and no requirement for overlap therapy with heparin.  Therefore, these agents are becoming
preferred for the treatment of acute DVT and PE.  These drugs are an especially good option in patients
who have dif culty with the dietary restrictions or frequent INR monitoring required with warfarin. 
However, these agents should not be used in patients with severely impaired renal function or
in those with DVT or PE secondary to malignancy.

Thrombolytic therapy is typically reserved for hemodynamically unstable patients with PE.  Less
commonly, it is used for massive proximal DVT associated with signi cant symptomatic
swelling and/or limb ischemia.  It is not indicated in patients with moderate pretibial edema and
absence of hypotension and tachycardia.
fi

fi

fi

fi

fi

fi
19)Prostate Adenocarcinoma is the most common cancer in men and often presents with
manifestations of metastatic disease.  Symptoms usually arise after spread to the axial skeleton
(vertebral bodies, ribs) with resultant progressive lower back pain or acute functional issues (eg,
motor weakness, incontinence) from pathologic fracture or spinal cord impingement.
Unlike many other forms of metastatic cancer to the spine, prostate cancer usually causes pure
osteoblastic lesions.  This leads to normal or low serum calcium (unlike osteolytic disease), elevations
in alkaline phosphatase, and imaging evidence of focal, sclerotic bone lesions.

Complement activation contributes to hemolytic uremic syndrome, which is associated with


microangiopathic hemolytic anemia, thrombocytopenia, and acute renal injury.  Urinalysis typically
shows hematuria and proteinuria. 

Budd-Chiari syndrome
• Hepatic venous out ow obstruction The etiology of some of these
• Usually due to: ndings is unclear, but
Etiology ◦ Myeloproliferative disorder (eg, PV) improvement in pruritus and
◦ Malignancy (eg, hepatocellular carcinoma) erythromelalgia with aspirin
◦ Oral contraception use/pregnancy may re ect a causative role for
prostaglandins (which are
• Acute pruritogenic).
◦ Jaundice, hepatic encephalopathy, variceal bleeding Risk of venous and arterial
◦ Prolonged INR/PTT; elevated transaminases thrombosis is signi cantly
Manifestations • Subacute/chronic increased in PV due to increased
◦ Vague, progressive abdominal pain blood viscosity and
◦ Hepatomegaly, splenomegaly, ascites abnormalities in platelet and
◦ Mild/moderate elevation in bilirubin, transaminases leukocyte function.

• Abdominal Doppler ultrasound – ↓ hepatic vein ow


Diagnosis
• Investigation for underlying disorders (eg, JAK2 testing for PV)
fi

fl

fi
fl

fl

Diamond-Blackfan anemia
DBA is caused by a congenital (inherited or sporadic)
Pathogenesis • Congenital erythroid aplasia defect of erythroid progenitor cells, which leads to
increased apoptosis of red blood cells, resulting in
• Craniofacial abnormalities
Clinical profound anemia.  Most patients have other anomalies,
• Triphalangeal thumbs
ndings such as short stature, cleft palate, and webbed
• Increased risk of malignancy
neck. DBA typically presents in infancy with progressive
• Macrocytic anemia pallor and poor feeding due to anemia.Laboratory
Laboratory
• Reticulocytopenia testing differentiates the type of anemia.  In contrast to
ndings
• Normal platelets, white blood cells Fanconi anemia (pancytopenic bone marrow failure), DBA
is a pure red cell aplasia with reticulocytopenia and
• Corticosteroids
Treatment normal platelet and white blood cell counts
• Red blood cell transfusions

Anemia of prematurity
• ↑ Oxygenation at birth → ↓ EPO
production
◦ Impaired transition from hepatic to renal EPO*
Patho— • Exacerbating factors in premature
—genesis infants:
◦ ↓ RBC life span
◦ Frequent blood draws
◦ Iron depletion

• Often asymptomatic After birth, increased oxygenation (ie, breathing, ductus


Clinical closure) triggers reduced erythropoietin (EPO)
• Tachycardia, poor weight gain, apnea,
features production by the liver and kidney.  Low levels of
hypoxia
circulating EPO impair erythropoiesis in the bone
• Normocytic, normochromic anemia marrow, which normally causes a mild, transient
Laboratory
• Inadequate reticulocyte response anemia that reaches a nadir of around 9-11 mg/dL at
ndings
• Normal platelets and Leukocytes age 2-3 months in term infants (ie, physiologic anemia
of infancy).  Compared to term infants, preterm infants
• RBC transfusion if severe or symptomatic have a lower initial hematocrit at birth, a shorter
Manage-
• Minimize exacerbating factors (eg, iron erythrocyte life span, and less capacity to produce
-ment
supplementation, limit blood draws)
renal EPO.  Therefore, their erythrocyte nadir tends to
be more severe (eg, ~7 mg/dL) and occur earlier
*Typically occurs in the 3rd trimester.
(eg, 1-2 months), resulting in AOP. 

115) Anisocytosis (red blood cells of unequal size) is often the rst nding of Iron De ciency
Anemia on peripheral smear, and the increased red blood cell distribution width (RDW) is a
quanti cation of this abnormality.  Gradually, the mean corpuscular volume (MCV) and mean
corpuscular hemoglobin (MCH) decline, correlating with microcytosis and hypochromia on
peripheral smear.  The iron stores become depleted, resulting in a low reticulocyte count, decreased
transferrin saturation (serum iron/total iron binding capacity [TIBC]), and increased TIBC.
fi
fi
fi

fi

fi
fi

fi
Iron de ciency anemia & thalassemias
Parameter Iron de ciency anemia Alpha-thalassemia minor Beta-thalassemia minor

MCV ↓ ↓ ↓

RDW ↑ Normal Normal

RBCs ↓ Normal Normal

Peripheral smear Microcytosis, hypochromia Target cells Target and Tear drop cells

Serum iron ↓ Iron & ferritin Normal/↑ iron & ferritin Normal/↑ iron &
studies ↑ TIBC (RBC turnover) ferritin (RBC turnover)

Response to iron supplementation ↑ Hemoglobin No improvement No improvement

Hemoglobin electrophoresis Normal Normal ↑ Hemoglobin A2

Alpha- and beta-thalassemia minor (also known as


A normal red cell distribution width (RDW) and
thalassemia trait) are often asymptomatic and found
normal-to-elevated total RBCs typically result in a
incidentally on laboratory testing.  Patients with
Mentzer index (MCV/RBC) <13.  Reticulocyte count
thalassemia minor are often diagnosed by universal
may be slightly elevated due to a compensatory bone
screening for anemia around age 1.Both types of
marrow response to anemia. IDA have Mentzer
thalassemia minor are characterized by abnormally small red
index of >13( due to a decline in RBC count).
blood cells (RBCs), re ected by a low MCV. 

Neonatal polycythemia Other than a ruddy/plethoric appearance, most


neonates with polycythemia are asymptomatic. 
De nition • Hematocrit >65% in term infants

• Increased erythropoiesis from intrauterine hypoxia: maternal diabetes, hypertension, or


smoking; IUGR
Causes
• Erythrocyte transfusion: delayed cord clamping; twin-twin transfusion
• Genetic/metabolic disease: hypothyroidism or hyperthyroidism; genetic trisomy (13, 18, 21)

• Asymptomatic (most common) As the hematocrit level rises due to postbirth uid
• Ruddy skin shifts, the increased blood viscosity may cause
Clinical • Hypoglycemia, hyperbilirubinemia decreased blood ow to organs.  Common
presentation • Respiratory distress, cyanosis, apnea consequences include lethargy, irritability, and
• Irritability, jitteriness jitteriness.  Severe symptoms include respiratory
• Abdominal distension
distress, tachypnea, poor feeding, and cyanosis. 
• Intravenous uids The increased red blood cell mass can lead to
Treatment • Glucose hypoglycemia and hypocalcemia due to
• Partial exchange transfusion increased cellular uptake. 

117) Transient tachypnea of the newborn is a common cause of respiratory distress in term neonates. 
Chest x-ray will show increased pulmonary vascular markings or uid in the ssures.  In
contrast, infants with polycythemia have normal x-rays because the total uid volume in the lungs is
normal.
fi
fi


fi


fl

fl
fl

fl

fl
fl
fi
Immune thrombocytopenic purpura ITP is an acquired disorder
• Commonly acquired form of thrombocytopenia marked by the formation of
Epidemiology • Autoantibody formation autoantibodies to platelet
• Often recent viral infection or comorbidity (eg, HIV, HCV, CLL) surface glycoproteins, which
increases platelet clearance in
• Frequently asymptomatic
the spleen and inhibits
• Mucocutaneous bleeding (eg, menorrhagia, epistaxis)
Manifestations megakaryocytosis. 
• Ecchymoses, petechiae, purpura
• Severe hemorrhage is rare In ITP, leukocyte count,
erythrocyte count, and
• Diagnosis of exclusion coagulation studies are
• Normal coagulation tests, platelet morphology (peripheral smear)
normal; peripheral blood smear
Diagnosis • HIV, HCV, CMV testing (commonly induces ITP)
Additional testing is needed when signs and symptoms of other
usually shows normal platelet

disorders (eg SLE,Thyroid dz ) are present. morphology (both normal-sized
and large platelets may be
• Observation if no bleeding & platelets >30,000/mm3 seen due to increased platelet
Treatment • Corticosteroids if platelets <30,000/mm3
turnover).
• IVIG & platelet transfusion if hemorrhage

Immune thrombocytopenia ITP is an autoimmune disorder in


which IgG autoantibodies against
• Platelet autoantibodies platelet membrane glycoproteins
Etiology cause increased platelet
• Preceding viral infection
destruction and inhibition of
Clinical • Petechiae, ecchymosis platelet production.
ndings • Mucosal bleeding (eg, epistaxis, hematuria) Other than bruising, physical
examination is generally
Laboratory • Isolated thrombocytopenia <100,000/mm3 unremarkable.  Patients have a
ndings • Few platelets (size normal to large) on peripheral smear normal liver span (eg, 6-12 cm at
midclavicular line) and
• Children nonpalpable spleen.
◦ Observe if cutaneous symptoms only Platelet transfusions are reserved
◦ Glucocorticoids, IVIG, or anti-D if bleeding for life-threatening hemorrhage
Treatment
• Adults in patients with ITP because
◦ Observation if cutaneous symptoms AND platelets ≥30,000/mm3 antibody production destroys the
◦ Glucocorticoids, IVIG, or anti-D if bleeding or platelets <30,000/mm3 transfused platelets. 

118) Most cases of ITP self-resolve within 3 months; however, some patients continue to have platelets
<100,000/mm3 for >1 year, which is known as chronic ITP.  Adolescents with mild initial presentation
and no preceding viral trigger are more likely to develop chronic ITP.
Second-line therapies (eg, rituximab, thrombopoietin receptor agonists) or splenectomy are
considered for those patients who have persistent bleeding and thrombocytopenia despite repeated
pharmacologic interventions (eg, glucocorticoids, anti-D immune globulin (if Rh-positive and
Coombs-negative), intravenous immunoglobulin).
Anti-D immune globulin is a potential rst-line therapy for ITP in patients with the Rh antigen.  Anti-
D binding to Rh(D)-positive erythrocytes is thought to saturate Fc receptors on macrophages within the
reticuloendothelial system (RES), thereby limiting the ability of the RES to clear platelets.
fi
fi

fi


Avascular necrosis Slipped Capital Femoral Epiphysis (SCFE)

• Steroid use
• Alcohol abuse
• Systemic lupus erythematosus
• Hemoglobinopathies (eg, sickle cell)
Etiology
• Antiphospholipid syndrome
• Decompression sickness
• Infections (eg, osteomyelitis, HIV)
• Renal transplantation

• Chronic Groin/hip/Buttock pain on


weight bearing
Clinical
• Pain on hip abduction & internal rotation Slipped capital femoral epiphysis (SCFE)
manifestations
• No erythema, swelling, or point results in displacement of the proximal femur due
tenderness to external shearing forces (eg, obesity, trauma). 
Laboratory • Normal white blood cell count Patients with SCFE typically report hip or groin
ndings • Normal ESR & CRP pain with weight bearing.  Because SCFE relies on
an open growth plate, it is more common in
Radiologic • Crescent sign seen in advanced stage
younger adolescents.
imaging • MRI is most sensitive modality

Electrophoresis patterns in sickle cell syndromes

Remain same Almost remain same

HbA HbA2 HbF HbS HbC

Normal ++++ + + None None

Sickle cell trait +++ + + +++ None

Sickle cell anemia (SCA) None + + ++++ None

SCA on hydroxyurea None + ++ (10-30%) +++ None

Hemoglobin SC disease None + + +++ +++

Just a Game of HbA and HbS

120) Individuals with sickle cell trait are generally asymptomatic and can lead a healthy life.  Although
patients are at increased risk for renal issues, the most common of these is painless microscopic or gross
hematuria that results from sickling in the renal medulla.  Hyposthenuria (impairment in concentrating
ability) is also common and can present as nocturia and polyuria.  Less commonly, there may be an increased
risk of urinary tract infections, particularly during pregnancy.  Splenic infarctions are uncommon but can occur
at high altitudes.
Sickle cell disease (SCD) is marked by an absence of normal beta globulin due to mutations in both beta globulin
genes.  Because patients with SCD are unable to generate normal beta globulin, they cannot produce HbA;
therefore, HbA will be undetectable on electrophoresis.
fi



Latest UW Notes for STEP2 CK - 2022
Features:
• Arranged system wise and then subject wise
• There are total 97 Blocks, Each BLOCK is made of 40 or some time 45 -50 different Questions hence
each BLOCK covers the NOTEs of those 40-50 questions,while the total 97 Blocks covers the whole
UWorld (3986 Qs)

• Custom BLOCK Id is mentioned with each BLOCK, so you can make Your own blocks in UWorld
according to the BLOCK id mentioned and read the respective BLOCK,s NOTES here

• Contains All UW tables edited and supplemented with UW Explanations


• All UW Diagrams/Pictures and algorithms annotated and simpli ed
• Some topics are supplemented with UPTODATE information,separate tables are there for these info so
that you can differentiate it form UW info

• Highlighted important info


• Most of the UW Tables are edited with related pictures and de cinet information added to the
tables from UW explanations

• These Notes will be updated regularly


• Notes of the Upcoming NEW Questions will be added on monthly basis

fi
fi


NERVOUS SYSTEM:

MEDICINE
Notes by: Step1usmle59@gmail.com
https://www.reddit.com/u/USMLEe59/?
utm_source=share&utm_medium=ios_app
&utm_name=iossmf

BLOCK 38
Custom Id: 220696009


01) Fibromyalgia presents most commonly in young to middle-age women with widespread
pain, fatigue, and cognitive/mood disturbances.  Patients tend to have fairly normal physical
examination ndings except for point tenderness. Initial FM treatment should emphasize patient
education, regular aerobic exercise, and good sleep hygiene.  Patients who do not respond to
conservative measures may require medications. Amitriptyline is an effective initial therapy. 
Pregabalin, duloxetine, and milnacipran are alternate therapies for patients not responding to tricyclic
antidepressants. Patients with persistent symptoms may require combination drug therapy, referral for
supervised rehabilitation, pain management consultation, or cognitive-behavioral therapy.
02) Papilledema is caused by increased intracranial pressure and presents with transient vision
loss lasting a few seconds with changes in head position.  It requires urgent diagnostic
evaluation (eg, ophthalmologic examination, neuroimaging, and/or lumbar puncture) as persistent
papilledema can lead to Permanent vision loss.

Anterior uveitis presents with eye pain and redness, whereas posterior uveitis is often painless and
associated with oaters or reduced visual acuity. 

Exertional heat stroke


• Strenuous activity during hot & humid weather
• Dehydration, poor acclimatization
Risk factors
• Lack of physical tness, obesity
• Medications: anticholinergics, antihistamines, phenothiazines, tricyclics, antipsychotics

• Core temperature >40 C (104 F) with CNS dysfunction (eg, AMS, seizure)
• Organ or tissue damage: renal or hepatic failure, disseminated intravascular coagulation, acute respiratory
Clinical
distress syndrome
manifestation
• Tachycardia, tachypnea, and ushing.  Diaphoresis may or may not be present, depending on uid status,
and hypotension may occur. 

• Rapid cooling: ice water immersion preferred; can consider high- ow cold water dousing, ice-wet towel
rotation, evaporative cooling
Management • Fluid resuscitation, electrolyte correction
• Management of end-organ complications
• No role for antipyretic therapy

03) Unlike Nonexertional heat stroke, which typically affects individuals at the extremes of age who are incapable
of obtaining adequate fluids and removing themselves from a hot environment, EHS occurs most commonly in
those exposed to hot/humid environments while performing extreme physical activity (eg, landscaping). 
fi
fi

fl

fl

fl

fl

Differential Diagnosis of Dementia Subtypes AD typically begins with memory loss for
recent events (eg, poor short-term memory,
• Early, insidious short-term memory loss
Alzheimer naming current president), executive
• Language de cits & spatial disorientation
disease dysfunction, and impaired visuospatial
• Later personality changes
skills, followed by language dif culties,
• Visual hallucinations
apraxia, and behavioral changes (eg, apathy,
Dementia with • Spontaneous parkinsonism
disinhibition, suspiciousness).  Psychotic
Lewy bodies • Fluctuating cognition
• Rapid eye movement behavior disorder features (delusions, hallucinations) may
develop in the middle and late course of the
Normal • Ataxia early in disease disease. Delusions with paranoid themes
pressure • Urinary incontinence are common.
hydrocephalus • Dilated ventricles on neuroimaging

• Behavioral changes Pseudodementia is seen in older depressed


Prion disease • Rapid progression patients who present with cognitive de cits similar
• Myoclonus &/or seizures to neurodegenerative dementias.

• Early personality changes Early personality changes (eg, euphoria,


Frontotemporal
• Apathy, disinhibition & compulsive behavior disinhibition, apathy), compulsive behaviors
dementia
• Frontotemporal atrophy on neuroimaging (eg, peculiar eating habits), hyperorality.

• Stepwise decline Early problems with executive function (eg,


Vascular • Early executive dysfunction
planning activities, making decisions)
dementia • Cerebral infarction &/or deep white matter
changes on neuroimaging associated with focal neurologic de cits.

04) Neuroimaging in patients with VaD classically demonstrates a cortical and/or subcortical
infarction with or without deep white matter changes from chronic ischemia.
Normal pressure hydrocephalus
• Gait instability (wide-based) with frequent falls
• Cognitive dysfunction
Clinical
• Urinary urgency/incontinence
features
• Depressed affect (frontal lobe compression)
• Upper motor neuron signs in lower extremities

• Marked improvement in gait with spinal uid removal: Miller Fisher (lumbar tap) test
Diagnosis
• Enlarged ventricles out of proportion to the underlying brain atrophy on MRI

Treatment • Ventriculoperitoneal shunting

06) Patients with large or embolic ischemic strokes and those treated with thrombolytics are
at high risk for hemorrhagic transformation.  This condition usually occurs within 48 hours of the
stroke and often manifests with deteriorating mental status.  Diagnosis requires emergent
noncontrast CT scan of the head as urgent surgical decompression is often needed.


fi

fi

fi
fi

fl

Type of Tremor Clinical features

• Bilateral action tremor of the hands, usually without leg involvement


• Possible isolated head tremor without dystonia
Essential
• Usually no other neurologic signs
• Relieved with alcohol in many cases

• Resting tremor (4-6 Hz) that decreases with voluntary movement


Parkinson
• Usually involves legs & hands
disease
• Facial involvement less common

• Usually associated with ataxia, dysmetria, or gait disorder


Cerebellar
• Tremor increases steadily as the hand reaches its target

• Low amplitude (10-12 Hz), not visible under normal conditions


Physiologic • Acute onset with increased sympathetic activity (eg, drugs, hyperthyroidism, anxiety, caffeine)
• Usually worse with movement & can involve the face & extremities

07) Although features of ET are highly variable, it most often affects the wrists and hands, especially
when the arms are extended.It worsens with movement (ie, action tremor) and improves,but
may not completely abate, at rest.  Although ET is not typically unilateral,it is often asymmetric.
It can also affect the head and voice but generally spares the trunk and legs.  ET is slowly
progressive and often hereditary; it can be passed in an autosomal dominant fashion.
Essential tremor

• Family history often present


Epidemiology
• Up to 5% of population affected
Any agent that increases adrenergic activity
(eg, caffeine) can exacerbate physiologic
• Action tremor (eg, hands outstretched) tremor.  However, caffeine does not
Clinical • Usually bilateral seem to be correlated with the
features • Hands > arms > head >> legs
severity of essential tremor.
• Worsens with stress, improves with alcohol

• Beta blockers: propranolol


Treatment
• Anticonvulsants: primidone

08) Neuropathic tremor occurs with large- ber neuropathies (eg, chronic in ammatory
demyelinating polyneuropathy).  Although it is also an action tremor that predominantly affects the
distal extremities, patients with neuropathic tremors often show other signs or symptoms of
peripheral neuropathy (eg, proprioceptive or sensory loss, weakness).
A Dystonic tremor of the arm can worsen with the arms outstretched and vanish with rest. 
However, it is usually unilateral, is less rhythmic, and is often task speci c (eg, writing, typing).

Apathy is characterized primarily by a loss of motivation, more speci cally a decreased motivation
to speak or act.  It is a common nding in patients with Parkinson Dz and may occur with or
without depression. 

fi

fi

fi
fi

fl

Brain abscess
• Staphylococcus aureus
Microbiology • Viridans streptococci
• Anaerobes

• Direct spread (eg, otitis media, mastoiditis, sinusitis)


Pathogenesis
• Hematogenous spread (eg, endocarditis)

• Headache, vomiting
Clinical • Fever
ndings • Focal neurologic de cits, seizure
• Ring-enhancing lesion on neuroimaging (CT, MRI)

CT-guided aspiration or surgical biopsy to obtain


Diagnosis tissue for Gram stain and culture (bacterial, fungal,
mycobacterial).

• Aspiration/surgical drainage
Treatment • Prolonged antibiotic therapy. Most patients are treated with empiric, intravenous antibiotics (eg,
metronidazole, ceftriaxone, and vancomycin) and aspiration of the lesion.

09) Herpes Encephalitis usually causes acute symptoms of altered mental status, focal de cits,
and/or focal seizures. CT scan of the brain often reveals hypodense lesions in the temporal lobe.  Ring
enhancement is uncommon.

Guillain-Barré syndrome GBS is often preceded by an


acute infection.  It may be
• Immune-mediated demyelinating polyneuropathy caused by an immune
Pathophysiology • Preceding gastrointestinal (Campylobacter) or
response that cross-reacts (ie,
respiratory infection
molecular mimicry) with
• Paresthesia, neuropathic pain peripheral nerve components
• Symmetric, ascending weakness (eg, myelin).  Although
Clinical features • Decreased/absent deep tendon re exes gastrointestinal and respiratory
• Autonomic dysfunction (eg, arrhythmia, ileus)
infections are most commonly
• Respiratory compromise
implicated, acute HIV
• Clinical infection can also trigger
• Supportive ndings GBS.  In patients with HIV, GBS
◦ Cerebrospinal uid: ↑ protein, normal leukocytes most commonly occurs prior
Diagnosis
◦ Abnormal electromyography & nerve conduction to the onset of AIDS (ie,
◦ MRI: normal or enhancement of anterior nerve
while there is still a robust
roots/cauda equina
immune response) and can be
• Monitoring of autonomic & respiratory function the initial HIV
Management
• Intravenous immunoglobulin or plasmapheresis presentation.
Patients have ascending motor de cits, hypore exia, and back pain; sensation is classically
unaffected (although paresthesias are common). 
fi

fi
fl

fi

fl

fi

fl

fi
Common Fibular Neuropathy

Causes: Compressive injuries from leg immobilization (eg,


cast, bedrest), prolonged leg crossing (eg, during yoga or
meditation), or protracted squatting. 
Manifestations are typically transient and re ect impaired
functioning of the bular nerve as follows:
• Unilateral foot drop
• Numbness/tingling over the dorsal foot and lateral shin
• Impaired ankle dorsi exion (walking on heels) and
great toe extension
• Preserved plantar exion (walking on toes) and
re exes
Diagnosis is typically made with electromyography and
nerve conduction studies.  Treatment has limited effect but
includes reducing pressure on the nerve (eg, avoiding crossing
the legs), an ankle-foot orthosis splint, and physical therapy.

Myasthenic crisis Myasthenia gravis usually presents in


the second to third decade in women
• Infection or surgery
(sixth to eighth decade in men) with
Precipitating • Pregnancy or childbirth
uctuating weakness that worsens
factors • Tapering of immunosuppressive drugs
• Medications (eg, aminoglycosides, beta blockers) over the course of the day and most
often involves the extraocular (eg,
Signs/ • ↑ Generalized & oropharyngeal weakness ptosis, diplopia), bulbar (eg,
symptoms • Respiratory insuf ciency/dyspnea fatigable chewing, dysphagia,
nasal speech), proximal limb, and
• Intubation for deteriorating respiratory status
Treatment respiratory muscles.
• Plasmapheresis or IVIG as well as corticosteroids
fl
fl


fi

fi

fl

fl

fl

Initial workup of suspected cognitive impairment


• Mini-Mental State Examination (score <24/30 suggestive of MCI/dementia)
Cognitive
• Montreal Cognitive Assessment (score <26/30)
testing
• Mini-Cog (abnormal 3-word recall &/or clock-drawing test)

• Routine: CBC, vitamin B12, TSH, Complete metabolic panel


Laboratory
• Selective (speci c risk factors): folate, syphilis, vitamin D level
testing
• Atypical (early onset): CSF

• Routine: CT scan or MRI of the brain


Imaging
• Atypical: EEG

CBC = complete blood count; CSF = cerebrospinal uid; MCI = mild cognitive impairment;

64) The rst step in evaluation of suspected MCI is to perform a standardized cognitive examination
using a cognitive screening tool such as the Montreal Cognitive Assessment (MoCA) or mini-
mental state examination (MMSE). The MoCA is a widely used 30-point scale that assesses
orientation, attention/concentration, language, delayed word recall, and visuospatial/
executive function.  Scores of <26/30 on the MoCA are suggestive of clinically signi cant
cognitive impairment.  If the results of the MoCA suggest clinically signi cant cognitive impairment,
the patient should undergo a workup for potentially reversible causes of cognitive
impairment, which includes depression screening, laboratory evaluation (eg, vitamin B12 level,
TSH, complete blood count, complete metabolic panel), and neuroimaging (eg, MRI).
Clinical features of Alzheimer disease
• Anterograde memory loss (ie, immediate recall affected, distant memories preserved)
Early • Visuospatial de cits (eg, lost in own neighborhood)
ndings • Language dif culties (eg, dif culty nding words)
• Cognitive impairment with progressive decline

• Neuropsychiatric (eg, hallucinations, wandering)


• Dyspraxia (eg, dif culty performing learned motor tasks)
Late
• Lack of insight regarding de cits
ndings
• Noncognitive neurologic de cits (eg, pyramidal & extrapyramidal motor, myoclonus, seizures)
• Urinary incontinence

65) AD is the most common cause of dementia in the United States and usually occurs after
age 65.  Death usually occurs within 10 years of diagnosis regardless of the treatment offered. 
Patients experience a progressive functional decline over the course of their disease. 
Later ndings usually include personality and behavioral changes (eg, apathy, agitation),
neuropsychiatric changes (eg, delusions, paranoia), neurologic manifestations (eg, myoclonus,
seizures), and apraxia (eg, dif culty with motor tasks).  Urinary incontinence may develop in later
stages of AD, secondary to severe cognitive dysfunction.
fi
fi


fi
fi


fi
fi
fi
fi
fi
fi
fi

fi
fi

fl

fi

fi

66) CRAO is an ophthalmologic emergency, and attempts at recovering vision (eg, anterior
chamber paracentesis, ocular massage, revascularization) should be considered.  Workup also
includes noninvasive imaging of the carotids to evaluate for stenosis.  Atherosclerotic treatment
(eg, aspirin, statin) and, in cases of cardioembolic phenomenon, long-term anticoagulation (eg,
warfarin) are often initiated.

Patients with a detached retina typically have light ashes, oaters, or a curtain across their visual eld
(usually starting from the periphery rather than dropping vertically) that is not transient.  Funduscopic
ndings can include a wrinkled or detached retina.

Clinical Characteristics of Major Stroke Subtypes


• Atherosclerotic risk factors (eg, uncontrolled hypertension, diabetes), ± history of transient ischemic attack
Ischemic
• Local obstruction of an artery (eg, carotid, cerebral, vertebral)
(thrombotic)
• Symptoms may alternate with periods of improvement (stuttering progression)

• History of cardiac disease (eg, atrial brillation, endocarditis) or carotid atherosclerosis


Ischemic
• Onset of symptoms abrupt & usually maximal at the start
(embolic)
• Multiple infarcts in different vascular territories

• History of uncontrolled hypertension, coagulopathy, illicit drug use (eg, amphetamines, cocaine)
Intracerebral • Symptom progression over minutes to hours
hemorrhage • Focal Neurologic symptoms that appear early, followed by features of increased intracranial
pressure (eg, vomiting, headache, bradycardia, reduced alertness)

• Bleeding from arterial saccular ("berry") aneurysm or arteriovenous malformation


Spontaneous
• Severe headache at onset
subarachnoid
• Meningeal irritation (eg, neck stiffness)
hemorrhage
• Focal de cits uncommon
fi
fi





fi

fl

fl

fi

BLOCK 41
Custom Id: 221955555


120) HD is classically associated with chorea (ie, involuntary contractions that seem to ow from one
muscle group to the next).  Early in the disease course, chorea may be mild and mistaken for
restlessness, and patients may unconsciously assimilate mild chorea into purposeful movements (eg,
smoothing one's hair).  Other neurological ndings include hyperre exia and delayed initiation
of voluntary saccades. Cognitive impairment is most evident in the domain of executive
function.

Huntington disease
• Motor: chorea, delayed saccades
Clinical
• Psychiatric: depression, irritability, psychosis, obsessive-compulsive symptoms
features
• Cognitive: executive dysfunction

• Genetic: autosomal dominant CAG trinucleotide repeat expansion disorder


Findings • Neuropathology: loss of GABA-ergic neurons
• Imaging: caudate nucleus & putamen atrophy

Management/ • Treatment: supportive


prognosis • Survival: 10-20 years

121) Dream enactment occurs because the muscle atonia that usually accompanies REM sleep
is absent or incomplete in RBD.  Episodes typically occur at least 90 minutes after sleep onset,
coinciding with the onset of REM sleep, and are more frequent in the second half of the night when
the percentage of REM sleep increases. RBD is more likely to occur in men age >50 years and has a
very strong association with the future development of alpha-synuclein neurodegenerative
disorders(either Parkinson disease (PD) or another disorder of alpha-synuclein neurodegeneration
(dementia with Lewy bodies or multiple system atrophy). Idiopathic RBD is considered a
prodromal marker of PD and other synucleinopathies.

Subarachnoid hemorrhage
• Hypertension
• Smoking
Risk
• Alcohol use
factors
• Arteriovenous malformations
• ADPKD

• Ruptured berry aneurysm (arterial bleed)


Clinical • Thunderclap headache ("worst of my life")
features • Meningeal signs (eg, photophobia, stiff neck)
• Usually no focal de cits

• CT scan without contrast


Diagnosis
• Lumbar puncture (eg, xanthochromia, ↑ ICP)


fi

fi

fl

fl
Angle-closure glaucoma

• Symptoms: headache, ocular pain, nausea,


Clinical decreased visual acuity
features • Signs: conjunctival redness; corneal opacity;
xed, mid-dilated pupil

• Tonometry (measures intraocular pressure)


Diagnosis
• Gonioscopy (measures corneal angle)

• Topical therapy: multidrug topical therapy (eg,


timolol, pilocarpine, apraclonidine)
Treatment • Systemic therapy: acetazolamide (consider
mannitol)
• Laser iridotomy

123) In patients predisposed to ACG, the lens is located more forward against the iris, which impairs
the normal ow of aqueous humor through the pupil into the anterior chamber, thereby increasing
IOP.  Sudden angle closure can then occur due to pupillary dilation from anticholinergic medications
(eg, tolterodine), sympathomimetics, or low ambient light.  Sulfonamides (eg, trimethoprim-
sulfamethoxazole) can occasionally trigger ACG due to swelling of structures (eg, lens, retina,
choroid) in the posterior chamber.
Distal symmetric polyneuropathy
• Diabetes mellitus (diabetic neuropathy)
• Long-standing HIV infection,Uremia
Triggers
• Medications (eg, uoroquinolones, metronidazole) Chemotherapy (eg, cisplatin, paclitaxel)
• Toxicity (eg, alcohol abuse, heavy metal exposure)

Pathophysiology • Damage to distal sensory peripheral nerve axons

• Symptoms begin in toes/feet; progresses proximally over time


• Distal numbness, tingling, pins & needles sensation
Manifestations
• ↓ Pain, temperature, touch & vibration sensation
• ↓ Ankle/Babinski re exes,Normal B/L Motor strength

• Treat underlying cause


Treatment
• Pain management: gabapentinoid, tricyclic antidepressant, duloxetine &/or capsaicin cream

124) Bilateral distal extremity numbness/pain with loss of sensation and loss of ankle jerk
re ex is consistent with distal symmetrical polyneuropathy.Although the exact mechanism is unclear,
HIV neuropathy is thought to occur due to the accumulation of HIV antigens (eg, gp120) on distal sensory
nerve bers, which triggers macrophage/T-cell in ltration, cytokine release, and subsequent
axonal damage.  The risk is greatest in older patients with long-standing HIV, particularly those who have
high viral loads, low CD4 counts, and a history of exposure to neurotoxic antiretroviral
medications (eg, didanosine, stavudine).
fi
fl
fi

fl
fl

fl

fi

Epidural hematoma

Trauma to sphenoid bone with tearing of middle


Pathogenesis
meningeal artery

• Brief loss of consciousness followed by lucid interval


• Hematoma expansion leads to:
Clinical ◦ ↑ Intracranial pressure (impaired consciousness,
features headache, nausea/vomiting)
◦ Uncal herniation (ipsilateral pupillary dilation &
contralateral hemiparesis)

CT scan of the head: biconvex (lens-shaped)


Diagnosis
hyperdensity that does not cross suture lines

Treatment Urgent surgical evacuation for symptomatic patients

125) Symptomatic patients with focal de cits or signs of intracranial hypertension require emergent
neurosurgical hematoma evacuation because treatment delay may lead to death secondary to
brain herniation.
Close observation and serial (every 6-8 hours) CT scans may be appropriate in patients with a
small epidural hematoma without signi cant neurologic dysfunction.  A lethargic patient
with EDH requires urgent surgical intervention.

When the diagnosis is suspected but the clinical


examination or MRI is not classic of MS, lumbar
puncture should be performed for cerebrospinal
uid (CSF) analysis; such analysis shows
oligoclonal bands in 85%-95% of patients.
In MS, the opening pressure and CSF appearance
are typically normal.  CSF analysis usually shows a
normal cell count with T lymphocyte
predominance. There is also usually a normal
total protein concentration, but immunoglobulin
levels (particularly IgG) are high relative to other
proteins.

127) The elderly are especially vulnerable to the cumulative adverse effect burden of polypharmacy. 
In particular, medications with anticholinergic properties are strongly associated with the risk for
cognitive impairment, which may produce a medication-induced syndrome that mimics dementia. 
Review of medications is an important rst step in evaluating a patient on a complex medication
regimen with new-onset or worsening cognitive impairment.
fl

fi
fi

fi

Midbrain strokes typically present with ipsilateral oculomotor nerve palsy, ataxia (due to
damage of the superior cerebellar peduncle), and contralateral hemiparesis (cerebral peduncle).
The Putamen is a common site of hypertensive intraparenchymal brain hemorrhage. The internal
capsule that lies adjacent to the putamen is almost always involved, leading to contralateral
hemiparesis, sensory loss, and conjugate gaze palsy toward the side of the lesion.

81) Multiple system atrophy (Shy-Drager syndrome) is a degenerative disease


characterized by the following:
1. Parkinsonism
2. Autonomic dysfunction (postural hypotension, abnormal sweating, disturbance of bowel or
bladder control, abnormal salivation or lacrimation, impotence, gastroparesis, etc)
3. Widespread neurological signs (cerebellar, pyramidal or lower motor neuron)
Always consider Shy-Drager syndrome when a patient with Parkinsonism experiences orthostatic
hypotension, impotence, incontinence, or other autonomic symptoms.  The accompanying bulbar
dysfunction and laryngeal stridor may be fatal.  Anti-Parkinsonism drugs are generally
ineffective, and treatment is aimed at intravascular volume expansion with udrocortisone, salt
supplementation, alpha-adrenergic agonists, and application of constrictive garments to the lower
body.

82)Riley-Day syndrome or familial dysautonomia is an autosomal-recessive disease seen


predominantly in children of Ashkenazi Jewish ancestry.  It presents at birth with feeding
problems and low muscle tone; it is characterized by gross dysfunction of the autonomic
nervous system (eg, no tears) with severe orthostatic hypotension (swings from severe
hypertension to postural hypotension).

UpToDate
Patients with very mild signs and symptoms of PD do not necessarily need any antiparkinson
therapy if symptoms are not interfering with quality of life and they prefer to avoid medication
side effects. For most patients who desire medication at this stage, we suggest an MAO B inhibitor
over other antiparkinson therapies. Amantadine monotherapy is an alternative,
particularly for those with predominant tremor.
When motor symptoms of PD begin to interfere with daily function and quality of life, symptomatic
therapy with a DA or levodopa is indicated.For patients 65 years of age and younger, we suggest
either a DA or levodopa as initial therapy. For older patients (>65 years) with symptoms of PD that
affect daily life, we suggest initiation of levodopa. DAs are less well tolerated in older adults and
those with cognitive dysfunction, and levodopa is more effective for improving motor function and
quality of life.

fl

Latest UW Notes for STEP2 CK - 2022


Features:
• Arranged system wise and then subject wise
• There are total 97 Blocks, Each BLOCK is made of 40 or some time 45 -50 different Questions
hence each BLOCK covers the NOTEs of those 40-50 questions,while the total 97 Blocks
covers the whole UWorld (3986 Qs)

• Custom BLOCK Id is mentioned with each BLOCK, so you can make Your own blocks in UWorld
according to the BLOCK id mentioned and read the respective BLOCK,s NOTES here

• Contains All UW tables edited and supplemented with UW Explanations


• All UW Diagrams/Pictures and algorithms annotated and simpli ed
• Some topics are supplemented with UPTODATE information,separate tables are there for
these info so that you can differentiate it form UW info

• Highlighted important info


• Most of the UW Tables are edited with related pictures and de cinet information added to
the tables from UW explanations

• These Notes will be updated regularly


• Notes of the Upcoming NEW Questions will be added on monthly basis

fi
fi

INFECTIOUS DISEASE:

MEDICINE
Notes by: Step1usmle59@gmail.com
https://www.reddit.com/u/USMLEe59/?
utm_source=share&utm_medium=ios_app
&utm_name=iossmf

BLOCK 47
Custom Id: 224714523

01) MAC is diagnosed with blood cultures or bone marrow aspirate.  First-line treatment
includes a macrolide (eg, clarithromycin, azithromycin) combined with ethambutol.  Antiretroviral
therapy should also be initiated a few days after MAC therapy begins. 
Mycobacterium avium complex
• M avium, M intracellulare
Epidemiology
• Patients with advanced AIDS (CD4 <50/mm3)

• Fever & weight loss


• Abdominal pain & diarrhea
Manifestations
• LAD & (often) hepatosplenomegaly
◦ ↑ Alkaline phosphatase

Diagnosis • Blood/lymph node culture → nontuberculous mycobacteria

• Macrolide + ethambutol
Treatment
• Primary prophylaxis (azithromycin) no longer recommended

UpToDate
Azithromycin is typically preferred if there are concerns for drug interactions or intolerance with
ethambutol.For those failing ART, we suggest that a third agent be added for the treatment of MAC. If
possible, we prefer rifabutin. However, if rifabutin cannot be used, uoroquinolones or a parenteral
aminoglycoside can be administered. If a patient is on ART at the time of MAC diagnosis, ART should
be continued and optimized if viremia is not suppressed. If a patient is treatment naïve or is not
currently taking ART, we recommend that HIV ART be started soon (typically a few days) after starting MAC
therapy.All patients with HIV who are diagnosed with MAC infection should receive a minimum of 12
months of MAC therapy. In addition, the CD4 count should be stably above 100 cells/microL for at least
six months before discontinuing treatment.

Sporotrichosis Cat scratch fever is marked by a cutaneous


lesion (vesicular, erythematous, or papular)
• Sporothrix schenckii (dimorphic fungus)
Epidemiology • Decaying plant matter/soil
with prominent, tender, regional
• Gardeners & landscapers lymphadenopathy and possible systemic
spread.  A patient with no lymphadenopathy,
• Subacute/chronic has cat scratch fever unlikely.
• Inoculation → skin papule → ulceration →
Manifestations nonpurulent, odorless drainage Hookworm larvae may cause a papular lesion at
• Proximal lesions form along lymphatic chain the site of entry (usually lower extremity) and
• Distant spread & systemic symptoms are rare proximal, intensely pruritic, reddish-
brown tracks from larval migration. 
Diagnosis • Cultures (aspirate uid or biopsy)
Multiple, nodular, ulcerating lesions that are
Treatment • 3-6 months of oral itraconazole mildly pruritic would be uncommon.

fl

fl

Acute HIV infection

Epidemiology • Typically presents 2-4 weeks after exposure Two notable clues (not
always present) are painful
• Mononucleosis-like syndrome (eg, fever, lymphadenopathy,
Clinical sore throat, arthralgias)
oral ulcerations and/or
features • Generalized macular rash oval, pink/red macular
• Gastrointestinal symptoms lesions that usually
• Viral load is markedly elevated (>100,000 copies/mL)
develop soon after acute
Diagnosis • HIV antibody testing may be negative (not yet seroconverted) illness and last
• CD4 count may be normal approximately a week.
• Combination antiretroviral therapy
Management
• Partner noti cation; consider secondary prophylaxis

03) Although most new cases arise in individuals with traditional HIV risk factors (eg, intravenous drug
use, incarceration, receptive anal intercourse, multiple sexual partners), patients frequently do not fully
disclose their social history; therefore, the absence of risk factors should not preclude testing.
Infectious urethritis in men

• Neisseria gonorrhoeae The diagnosis of infectious urethritis is


• Chlamydia trachomatis
Etiology made when ≥1 of the following are
• Mycoplasma genitalium
• Trichomonas vaginalis
present:
• Purulent or mucopurulent
• Dysuria
Manifestations penile discharge
• Discharge
• ≥2 leukocytes per eld on
• Gram stain & culture urethral swab Gram stain
Diagnosis
• Nucleic acid ampli cation testing
• Positive leukocytes on urine
• GU alone: ceftriaxone dipstick or ≥10 leukocytes/hpf
• GU + Chlamydia*: ceftriaxone + doxycycline on urinalysis
Treatment
• Chlamydia/Mycoplasma: azithromycin
• Trichomonas: metronidazole

*GU + uncertain Chlamydia status is also treated with ceftriaxone + doxycycline. GU = gonococcal urethritis.

04) Evaluation of deep neck space infections may warrant CT scan.  However, patients
with these infections usually have trismus, and examination often reveals uvular deviation (in
peritonsillar abscess) or pharyngeal wall bulging (in retropharyngeal abscess).

A Pathergy test diagnoses Behçet syndrome; it involves obliquely inserting a 20-gauge needle
into the skin and checking the site 24-48 hours later for the development of a >2-mm papule. 
Behçet syndrome can cause recurrent genital ulcers but they are usually small, painful, and on
the scrotum.  Oral ulcers are also frequently present.

fi
fi

fi

Bacterial Causes of Diarrhea


Organism Features

• Diarrhea, abdominal cramping


Bacillus cereus
• Ingestion of preformed toxin in starchy foods such as rice

• Vomiting, abdominal pain


Staphylococcus
• Diarrhea not typical but may occur
aureus
• Caused by preformed toxin with rapid onset of symptoms

• Abdominal pain, watery diarrhea, possible fever


Clostridium
• Bloody stools unusual
dif cile
• Associated with antibiotic exposure

Clostridium • Brief illness with watery diarrhea, cramps & fever


perfringens • Associated with undercooked or unrefrigerated food

• Watery diarrhea, fever, abdominal pain & vomiting


Salmonella • Associated with undercooked foods, especially poultry & eggs
• Antibiotic treatment needed only for severe disease or immunocompromised patients

• Vomiting, diarrhea & abdominal pain


Vibrio vulni cus • Associated with raw or undercooked shell sh
• May cause invasive, life-threatening dz in immunocompromised patients or those with liver dz

• Watery diarrhea, may be bloody if associated with EHEC (Shiga-toxin producing) strain
Escherichia coli
• Associated with undercooked beef or foods contaminated with bovine feces

• Bloody diarrhea with fever & often bacteremia


Shigella
• Associated with contaminated food or water, especially during travel outside the United States

• Abdominal pain, bloody diarrhea


Campylobacter
• Highest incidence in children & young adults
species
• Associated with raw or undercooked meats

05) The most common cause of bloody diarrhea in the absence of fever is E coli.Isolated cases
without a clear source are also common.  The diagnosis can be con rmed with assay for Shiga toxin
in stool.  Stool culture can also identify toxin-producing strains.  Treatment is generally supportive;
antibiotics are not helpful and may increase the risk of hemolytic uremic syndrome.

06) Staphylococcal ecthyma is a deep form of impetigo that typically begins as a vesicle/pustule
on an in amed area of skin and then converts to an indurated, purulent ulcer.  Lesions are most
common on the buttocks, thighs, and legs.

Clindamycin is often used to treat airway and skin aerobic and anaerobic infections.
Trimethoprim-sulfamethoxazole has fairly broad efficacy against gram-negative, gram-
positive, and atypical organisms but does not effectively treat syphilis.
fi
fl
fi

fi

fi

Humans are incidental hosts, and larvae


are typically unable to penetrate the
dermal basement membrane.  As a
result, cutaneous infection without
deeper penetration is the norm.

Urushiol is the irritant component of


poison ivy, poison oak, and poison
sumac.  Patients often develop
intensely pruritic, linear, vesicular
(or plaque or bullous) lesions. 

Pertussis Symptoms are less severe in


adolescents and adults, but
• Catarrhal (1-2 weeks): mild cough, rhinitis pertussis should be suspected in
• Paroxysmal (2-6 weeks): cough with inspiratory
Clinical patients with persistent (>2
whoop,  posttussive emesis; infants: possible apnea
phases weeks) cough and either:
• Convalescent (weeks to months): symptoms resolve
paroxysmal cough; inspiratory
gradually
whoop; or posttussive emesis. 
• Pertussis culture or PCR Lymphocytosis (toxin-induced)
Diagnosis
• Lymphocyte-predominant leukocytosis is often an important diagnostic
clue.  Con rmation can be made
Treatment • Macrolides
by respiratory culture or PCR
Prevention • Acellular pertussis vaccine testing.

UpToDate
For patients who warrant antibiotic therapy for pertussis, we suggest azithromycin or clarithromycin.
For patients who cannot tolerate macrolide antibiotics, trimethoprim-sulfamethoxazole is an
acceptable alternative.
We suggest postexposure antibiotic prophylaxis for the following individuals:
Household contacts of patients with pertussis
Individuals who had close contact to a patient with pertussis and are at risk for severe pertussis illness
(eg, infants, women in the third trimester of pregnancy, and those with chronic health conditions such as
an immunocompromising condition or chronic lung disease)
Individuals (including health care personnel) who had close contact to a patient with pertussis and have
close contact with individuals at risk for severe pertussis illness
The prophylaxis regimen is the same as the treatment regimen. Prior vaccination with Tdap
should not be considered suf ciently protective to eliminate the need for chemoprophylaxis.
fi

fi

18) A minimal response to control antigens in IGRA is most often seen with lymphocyte
immunosuppression due to HIV or the use of immunomodulatory medications (eg,
glucocorticoids).  In patients with HIV, testing should be repeated when CD4 counts improve
with antiretroviral therapy; in those taking immunomodulatory medications, testing should be
repeated after these medications have been discontinued for a couple of weeks.  This
usually restores the ability of lymphocytes to generate interferon-gamma to a control mitogen, thereby
allowing interpretation of interferon-gamma release in response to M tuberculosis antigens.Evaluation
for latent tuberculosis infection with either the interferon-gamma release assay (IGRA) or
tuberculin skin test (TST) should be performed prior to the initiation of anti–tumor necrosis factor
therapy (which increases the risk of reactivation tuberculosis).  Because IGRA uses antigens fairly
speci c for tuberculous mycobacteria, it is more sensitive and speci c than TST.  Furthermore,
because IGRA identi es antigens that are not present in Mycobacterium bovis, test results are
not typically affected by previous vaccination with bacille Calmette-Guérin (a live strain of M bovis
sometimes used as a vaccine against tuberculosis) .

UpToDate
LTBI is a clinical diagnosis that is established by demonstrating prior TB infection and excluding active
TB disease. Available tests to demonstrate prior TB infection include the tuberculin skin test (TST) and
interferon-gamma release assays (IGRAs). IGRAs are diagnostic tools for LTBI. They are in vitro blood
tests of cell-mediated immune response to Mycobacterium tuberculosis and measure T cell release of
interferon-gamma following stimulation by antigens speci c to M. tuberculosis.IGRAs cannot
distinguish between latent infection and active TB disease and should not be used for diagnosis of
active TB. A negative IGRA does not rule out active TB at any age.
The TST assesses the intradermal delayed-type hypersensitivity response produced by administration of
puri ed protein derivative (PPD) from Mycobacterium tuberculosis. TST can be falsely negative
because of biologic problems (such as immunosuppression) or because of technical problems with the TST
administration, PPD material, or reading. The TST result can be false positive because of infection with
nontuberculous mycobacteria or prior immunization with Bacillus Calmette-Guérin (BCG). A period of
up to eight weeks after primary TB infection may be required for TST conversion to occur. Individuals with
close contact to a known infectious case of TB and negative initial TST should have repeat TST 8
to 10 weeks after the end of exposure.

19) Approximately 60% of patients with S gallolyticus IE have colorectal cancer; this correlation may
stem from bacterial adhesion molecules that bind to overexpressed surface ligands on the
cancer, which allows it to aggregate and subsequently enter the bloodstream.  However, other
colorectal pathologies (eg, in ammatory bowel disease) also increase the risk of S gallolyticus IE.
Because S gallolyticus infections are tightly linked to colorectal pathology, patients should undergo
colonoscopy to evaluate for occult malignancy.  Liver function tests and hepatitis serology are
also usually warranted because damage to the hepatic sinusoids can increase the risk of infection.
fi
fi
fi
fl
fi

fi


Lyme disease clinical features Although early localized
Lyme disease is marked by the
Early localized • Erythema migrans formation of a single EM
(days to 1 • Fatigue, headache lesion at the site of
month) • Myalgias, arthralgias inoculation, hematologic
spread can lead to the
• Multiple erythema migrans formation of multiple EM
Early
• Unilateral/bilateral CN palsy (eg, CN VII) lesions, a sign of early
disseminated
• Meningitis disseminated Lyme
(weeks to
• Carditis (eg, AV block) disease.  Early disseminated
months)
• Migratory arthralgias disease usually occurs weeks
or months after inoculation,
Late • Arthritis (Monoarticular) and patients frequently do
(months to • Encephalitis not remember a previous
years) • Peripheral neuropathy tick bite or rash. 

Diagnosis in Early localized lyme dz is based on characteristic symptoms because B burgdorferi serology is
generally negative at this early stage of illness (the humoral response is still being formed).  Empiric treatment
with a 14-day course of oral doxycycline is curative in most patients.
Because patients with early disseminated and late Lyme disease have a fully formed humoral antibody response,
the diagnosis is made with Lyme serology.  Treatment with a prolonged course of oral doxycycline is usually
curative, but patients with more serious complications (eg, carditis, meningitis) require intravenous ceftriaxone.

33) Selenium sul de is often used to treat tinea versicolor, a lipid-dependent yeast infection of the
skin.  Although patients typically develop large, macular lesions on the trunk or upper extremities, the
rash is scaly and does not typically have central clearing.
Acute epididymitis
• Age <35: sexually transmitted (chlamydia, gonorrhea)
Etiology
• Age >35: bladder outlet obstruction (coliform bacteria)

• Unilateral, posterior testicular pain


• Epididymal edema
Manifestations
• Pain improved with testicular elevation
• Dysuria, frequency (with coliform infection)

• NAAT for chlamydia & gonorrhea


Diagnosis
• Urinalysis/culture



fi

35) Travelers to Malaria endemic countries with Chloroquine resistance Malaria are usually given
chemoprophylaxis with atovaquone-proguanil, doxycycline, or me oquine.  Me oquine
treatment should begin >2 weeks prior to travel, continued during the stay, and discontinued 4
weeks after returning.  Neuropsychiatric side effects (eg, anxiety, depression, restlessness) occur in
approximately 5% of patients and should prompt a change to an alternate medication.
Chloroquine chemoprophylaxis is used only in
areas without resistance (eg, parts of Central
America, Caribbean).  Resistance is widespread in
Africa, Asia (including India), and Oceania. 
Primaquine can be used for chemoprophylaxis only
in areas that have >90% P vivax.
Mosquito bite prevention (eg, insecticide,
insecticide-treated bed nets) alone is indicated for
travelers to areas that have sporadic cases of
malaria (eg, Belize). 

Meningococcal meningitis Early meningococcal infection typically causes


nonspeci c symptoms (eg, fever, headache,
• Neisseria meningitidis vomiting).Nonsuppurative pharyngitis, is a rare
Epidemiology • Most common in young children and presentation of meningococcal meningitis that may
young adults
be mistaken for streptococcal or viral pharyngitis. 
• Initial: nonspeci c fever, headache, However, patients with pharyngitis due to
Clinical vomiting, myalgias, sore throat meningococcus have rapid disease progression
features • Within 12-24 hr: petechiae/purpura, (<12 hours) and often describe their illness as the
meningeal signs, AMS "worst I have ever felt."  Other key features
distinguishing meningococcal disease from more
Treatment • Ceftriaxone
benign illnesses include:
• Shock • Severe myalgias(diffuse leg pain)
Complications • Disseminated intravascular coagulation • Cold hands/feet
• Adrenal hemorrhage • Mottled skin or pallor
These features likely re ect the signi cant
• Droplet precautions in ammatory response to meningococcal infection
Prevention
• Chemoprophylaxis* for close contacts
(eg, interleukin-6 causing myocardial depression
*Rifampin, cipro oxacin, or ceftriaxone. and poor perfusion). 

36) Treatment of meningococcal meningitis should be administered as soon as possible.  Although


cerebrospinal uid culture is diagnostic, treatment should not be delayed for lumbar puncture in
patients with suspected meningococcal meningitis due to signi cantly increased mortality.  Empiric antibiotic
regimens should also include vancomycin for coverage of penicillin-resistant Streptococcus pneumoniae, a
common cause of bacterial meningitis in all age groups.  Although lumbar puncture can be delayed, blood
cultures should be obtained prior to antibiotic initiation.  Even with appropriate treatment, meningococcal
meningitis is potentially fatal due to complications of shock, multiorgan failure, and/or disseminated
intravascular coagulation.
fl

fi
fl
fl
fi

fl

fi

fi
fl

fl
Syphilis: Diagnostic serology
• Antibody to cardiolipin-cholesterol-lecithin antigen Quantitative (eg, titers) and
Nontreponemal • Quantitative (titers) identi es the reactivity of infected
(RPR, VDRL) • Possible negative result in early infection serum to cardiolipin-cholesterol-
• Decrease in titers con rms treatment lecithin antigen.

• Antibody to treponemal antigens Qualitative (either positive or


Treponemal • Qualitative (reactive/nonreactive) negative) and identi es
(FTA-ABS, TP-EIA) • Greater sensitivity in early infection antibodies to speci c treponemal
• Positive even after treatment antigens

FTA-ABS has the highest diagnostic sensitivity (>97%) in patients with early primary syphilis and can be used to establish the diagnosis in
patients with negative initial VDRL results when there is a high suspicion for syphilis (as in patient with a chancre). 

Syphilis treatment All patients with syphilis require


nontreponemal titers (eg, RPR) at
Stage First-line Alternate
the time of treatment and
Primary (chancre) again at 6-12 months to ensure
Penicillin G IM × 1 Doxycycline × 14 days adequate response (a 4-fold drop
Secondary (diffuse rash)
in titers).  This is especially
Latent (asymptomatic) important for patients receiving
Penicillin G IM × 3 Doxycycline × 28 days alternate regimens because the
Tertiary (eg, CV, gummas) risk of treatment failure is
greater than in those who receive
Neurosyphilis Penicillin G IV ×
Ceftriaxone IV×14 days* penicillin.
(eg, meningitis, ocular) 10-14 days

*Penicillin desensitization followed by IV penicillin is preferred for those with penicillin allergy who have neurosyphilis, but
ceftriaxone can be used in those unable to be desensitized. CV = cardiovascular; IM = intramuscular; IV = intravenous.

39) The varicella vaccine is a live attenuated virus that causes a low virulence infection after
inoculation.  Patients with HIV with CD4 counts <200/mm3 may have dif culty clearing the attenuated
virus and should not receive vaccination.  Those with CD4 counts >200/mm3 who have low titers
should receive the varicella vaccine.
Varicella vaccination is typically given to children in 2 doses (age 12-15 months and 4-6 years). 
Adults with HIV should receive vaccination if titers are low (and CD4 count >200/mm3).
All patients with HIV should receive the inactivated vaccines (in uenza, tetanus-diphtheria-
pertussis) recommended for the general population as well as those recommended speci cally for
people with HIV (pneumococcal, hepatitis B).
fi

fi

fi
fi


fl
fi
fi

Manifestations include >1 chronic,


anesthetic, macular (often
hypopigmented) skin lesions with raised,
well-demarcated borders.  Nearby nerves
often become nodular and tender, and
segmental demyelination may result in
loss of sensation and motor function. 

Mycobacterium tuberculosis can rarely


cause skin manifestations, including
ulcers, nodules, and warty plaques.  A
hypopigmented, anesthetic patch with
nerve nodularity/pain is more likely to be
leprosy.


Latest UW Notes for STEP2 CK - 2022
Features:
• Arranged system wise and then subject wise
• There are total 97 Blocks, Each BLOCK is made of 40 or some time 45 -50 different Questions hence
each BLOCK covers the NOTEs of those 40-50 questions,while the total 97 Blocks covers the whole
UWorld (3986 Qs)

• Custom BLOCK Id is mentioned with each BLOCK, so you can make Your own blocks in UWorld
according to the BLOCK id mentioned and read the respective BLOCK,s NOTES here

• Contains All UW tables edited and supplemented with UW Explanations


• All UW Diagrams/Pictures and algorithms annotated and simpli ed
• Some topics are supplemented with UPTODATE information,separate tables are there for these info so
that you can differentiate it form UW info

• Highlighted important info


• Most of the UW Tables are edited with related pictures and de cinet information added to the
tables from UW explanations

• These Notes will be updated regularly


• Notes of the Upcoming NEW Questions will be added on monthly basis

fi
fi

Rheumatology/Orthopedics
& SPORTs:

MEDICINE
Notes by: Step1usmle59@gmail.com
https://www.reddit.com/u/USMLEe59/?
utm_source=share&utm_medium=ios_app
&utm_name=iossmf

BLOCK 53
Custom Id: 226700166


Clinical features of Psoriatic Arthritis


• Distal interphalangeal joints
• Asymmetric oligoarthritis
Arthritis • Symmetric polyarthritis, similar to rheumatoid arthritis
• Arthritis mutilans (deforming & destructive arthritis)
• Spondylarthritides (sacroiliitis & spondylitis)

• Enthesitis (in ammation at site of tendon insertion into bone)


Soft tissue &
• Dactylitis ("sausage digits") of toe or nger
nail
• Nail pitting & onycholysis
involvement
• Swelling of the hands or feet with pitting edema

• Arthritis precedes skin disease in 15% of patients


Skin lesions
• Skin lesions are present but not yet diagnosed in 15% of patients

Psoriatic Arthritis occurs in 5%-30% of patients who have psoriasis.  Psoriatic arthritis can present with
many different patterns, but the classic presentation involves the distal interphalangeal (DIP) joints. 
Morning stiffness is present, as it is in all in ammatory arthritides.  Deformity of involved joints, dactylitis
("sausage digit," a diffusely swollen nger), and nail involvement are common.  Current treatment
options for psoriatic arthritis include nonsteroidal anti-in ammatory agents, methotrexate, and anti-
tumor necrosis factor agents.
Enteropathic arthritis occurs in 10%-20% of patients with Crohn disease and ulcerative colitis.  It most often
affects the lower extremities and sacroiliac joints and tends to wax and wane with the symptoms
of bowel disease.  These patients have prominent gastrointestinal symptoms and other extraintestinal
manifestations of in ammatory bowel disease.

Patients with sarcoidosis who are asymptomatic are often followed without treatment due to a
high rate of spontaneous remission.  However, patients with symptomatic disease (eg, cough,
dyspnea, fatigue, hypercalcemia) are treated with systemic glucocorticoids.  It is important to rule out
infectious granulomatous diseases with similar presentations, such as tuberculosis (TB) and
histoplasmosis, prior to initiation of glucocorticoids.
Parvovirus B19 infection
• Most patients are asymptomatic or have ulike symptoms
Signs & • Erythema infectiosum ( fth disease): Fever, nausea & “slapped cheek” rash (more common in children)
symptoms • Acute, symmetric arthralgia/arthritis: Hands, wrists, knees & feet (resembles RA)
• Transient pure red cell aplasia; aplastic crisis in patients with underlying hematologic disease (eg, sickle cell)

• Acute infection
◦ B19 IgM antibodies in immunocompetent patients
Diagnosis ◦ NAAT for B19 DNA in immunocompromised patients
• Previous infection: B19 IgG antibodies (documents immunity)
• Reactivation of previous infection: NAAT for B19 DNA
fl

fl
fi

fl

fi
fi

fl

fl

Manifestations of systemic lupus erythematosus


SLE is a multisystem in ammatory
• Constitutional: fever, fatigue & weight loss disorder that can affect almost any
• Symmetric, migratory arthritis organ. Central nervous system
• Skin: butter y rash & photosensitivity
Clinical manifestations commonly include
• Serositis: pleurisy, pericarditis & peritonitis
symptoms
• Thromboembolic events (due to vasculitis & vasculitis, thrombotic or
antiphospholipid antibodies) thromboembolic disease (eg,
• Neurologic: cognitive dysfunction & seizures antiphospholipid syndrome), seizures,
• Hemolytic anemia, thrombocytopenia & leukopenia and transverse myelitis.  Cerebrospinal
• Hypocomplementemia (C3 & C4) uid ndings may be normal or re ect
Laboratory • Antibodies: mild in ammation (eg, slight elevation
ndings ◦ ANA (sensitive) in CSF leukocytes, lymphocytosis).
◦ Anti-dsDNA & anti-Sm (speci c)
• Renal involvement: proteinuria & elevated creatinine

Rheumatologic diseases & commonly associated autoantibodies


Sensitivity (%) Speci city (%)

Rheumatoid arthritis RF: 70-80 Anti-CCP: 95

Systemic lupus erythematosus ANA: 95 Anti-dsDNA/anti-Sm: 96

Drug-induced lupus ANA: 95 Antihistone: 95

Diffuse systemic sclerosis ANA: 95 Anti–Scl-70: 99

Limited systemic sclerosis ANA: 95 Anticentromere: 97

Polymyositis/dermatomyositis ANA: 75 Anti–Jo-1: 99


Initial evaluation of SLE should include a complete blood count, renal function studies (ie, urinalysis,
serum creatinine), and serum complement levels (eg, C3, C4). In addition, autoantibody studies are
recommended to support the diagnosis.  An antinuclear antibody (ANA) titer is very sensitive (95%-100%)
for SLE and should be obtained as the initial rst-order test.  If the ANA test is positive, more speci c
con rmatory tests such as anti–double-stranded DNA and anti-Sm (Smith) antibodies can be obtained. 

Psoriatic arthritis occurs in 15%-30% of patients with psoriasis and usually arises in those with long-
standing psoriatic skin disease (risk is 2%-3% per annum).  However, a minority of patients with PA have little
(eg, dry skin) or no skin involvement at the time of diagnosis.  Manifestations of PA can arise in the peripheral
or axial skeleton; most patients have asymmetric oligoarthritis or symmetric polyarthritis that often
(~50%) involves the DIP joints.  Signi cant morning stiffness and improvement with use are almost
always seen.  Nail lesions (eg, pitting, onycholysis, hyperkeratosis, splinter hemorrhage) occur in
>90% of cases and are an important diagnostic clue.  Soft tissue manifestations such as dactylitis
("sausage digit") or enthesitis (in ammation of insertion points of tendons or ligaments) can also occur.
fl
fi
fi
fi
fi

fl

fl

fl
fi

fl

fl

fi
fi

fi

Paget disease of bone Asymptomatic patients with


minimal disease activity (eg,
• Most patients are asymptomatic
incidental discovery of mildly
• Bone pain & deformity
increased alkaline phosphatase
Clinical ◦ Skull: headache, hearing loss
features ◦ Spine: spinal stenosis, radiculopathy
level) may be observed
◦ Long bones: bowing, fracture, arthritis of adjacent joints without being treated. 
• Giant cell tumor, osteosarcoma However, symptomatic patients
and those with involvement of
• Osteoclast dysfunction
Pathogenesis high-risk skeletal structures (eg,
• Increased bone turnover
skull, weight-bearing long
• Elevated alkaline phosphatase bones) should be treated to
Laboratory
• Elevated bone turnover markers (eg, PINP, urine hydroxyproline) reduce pain and decrease the
testing
• Calcium & phosphorus are usually normal risk of complications. 
• X-ray: osteolytic or mixed lytic/sclerotic lesions Bisphosphonates (eg,
Imaging
• Bone scan: focal increase in uptake alendronate, zoledronic acid)
inhibit osteoclasts and
Treatment • Bisphosphonates
suppress bone turnover and are
PINP = procollagen type I N-terminal propeptide. the preferred therapy.

Clinical features of dermatomyositis


Muscle • Proximal, symmetric
weakness • Weakness in UE = LE

• Gottron sign/papules
Skin ndings
• Heliotrope rash

• Interstitial lung disease Characteristic cutaneous ndings (eg, violaceous


Extramuscular
• Dysphagia poikiloderma over various regions of the body) are:On the
ndings
• Myocarditis face, the eruption is often accompanied by periorbital
erythema (often with edema) and is known as the
• ↑ CPK, aldolase, LDH heliotrope sign.The erythematous rash follows a
• Anti-Mi2, anti-Jo-1 photodistributed pattern on the chest (v sign) and
Diagnosis
• Diagnostic uncertainty: EMG/MRI, back (shawl sign). Violaceous, occasionally scaly, lesions
biopsy (skin/muscle) that can be at (Gottron sign) or raised (Gottron
• High-dose glucocorticoids plus papules) can be present on the metacarpophalangeal joints,
Management glucocorticoid-sparing agent proximal interphalangeal joints, elbows, and knees and are
• Screening for malignancy pathognomonic for dermatomyositis.

CPK = creatine phosphokinase; EMG = electromyography; LDH = lactate dehydrogenase; LE = lower extremity; UE = upper extremity.

Malignancies are more common in patients with dermatomyositis (>15% of adult patients).  The
most common malignancies are adenocarcinomas (eg, ovarian, lung, pancreatic, stomach,
colorectal cancers). 
fi
fi
fl

fi

Distinguishing features of bromyalgia, polymyositis & polymyalgia rheumatica


Clinical features Diagnosis

• Young to middle-aged women


• Chronic widespread pain • ≥3 months of symptoms with widespread
Fibromyalgia • Fatigue, impaired concentration pain index or symptom severity score
• Tenderness at trigger points (eg, midtrapezius, • Normal laboratory studies
costochondral junction)

• Elevated muscle enzymes (eg, creatine


• Proximal muscle weakness (eg, increasing
kinase, aldolase, AST)  
Polymyositis dif culty climbing stairs)
• Autoantibodies (anti–Jo-1, ANA)
• Pain mild/absent
• Biopsy: endomysial in ltrate, patchy necrosis

• Age >50
Polymyalgia • Systemic signs & symptoms • Elevated ESR, C-reactive protein
rheumatica • Stiffness > pain in shoulders, hip girdle, neck • Rapid improvement with glucocorticoids
• Association with giant cell (temporal) arteritis

Erythrocyte sedimentation rate and C-reactive protein are in ammatory markers often used in the
assessment of polymyalgia rheumatica, a syndrome characterized by proximal muscle pain and
stiffness out of proportion to weakness.  It most often affects the shoulders.  In polymyositis,
weakness is typically more prominent than pain. 

Heberden and Bouchard nodes at the distal interphalangeal (DIP) and proximal interphalangeal
(PIP) joints, respectively,represent bony enlargement adjacent to the articular cartilage and are
pathognomonic for OsteoArthritis.Bony hypertrophy is often also found at the rst
carpometacarpal joint (base of thumb). OA of the hands is primarily a clinical diagnosis and is
often apparent at the time of initial presentation.  X-ray may be performed in ambiguous cases and
may show decreased joint space, subchondral sclerosis, and periarticular osteophytes.  However,
imaging is less sensitive than examination ndings and is usually unnecessary for diagnosis or management.
fi


fi

fi

fi

fl
fi

Sjögren syndrome The diagnosis of SS is made in patients with ocular or oral


dryness when there is:
• Keratoconjunctivitis sicca • Evidence of dry mouth and eyes.  In the
Exocrine
• Dry mouth, salivary hypertrophy Schirmer test, a sterile strip of lter paper is placed
features
• Xerosis under the lower eyelid, and the area moistened by the
• Raynaud phenomenon absorbed uid is measured after 5 minutes.
• Cutaneous vasculitis • Either histologic evidence of lymphocytic
Extraglandular in ltration of the salivary glands or presence of
• Arthralgia/arthritis
features
• Interstitial lung disease serum autoantibodies directed against SSA (Ro)
• Non-Hodgkin lymphoma and/or SSB (La).

• Objective signs of decreased lacrimation (eg,


Schirmer test)
• Positive anti-Ro (SSA) &/or anti-La (SSB)
Biopsy of salivary glands can
Diagnostic con rm the diagnosis of SS if other
• Salivary gland biopsy with focal lymphocytic
ndings evaluations are nondiagnostic. 
sialoadenitis
• Classi cation: primary if no associated CTD, secondary if Samples are usually taken from the
comorbid CTD (eg, SLE, RA, scleroderma) minor glands of the lip.

CTD = connective tissue disease; SSA/SSB = Sjögren syndrome (antibody) A/B.

Achilles Tendinopathy
The diagnosis is based primarily on
clinical ndings.  AT presents with ankle
• Athletic activity, increase in activity
Risk factors • Systemic disorders: psoriasis, ankylosing spondylitis pain, swelling, and tenderness that are
• Medications: glucocorticoids, uoroquinolones usually most prominent approximately
2-6 cm proximal to the insertion of
• Swelling, warmth, pain at posterior heel the tendon, where perfusion of the
Clinical
• Tendon rupture: "popping" sensation & acute pain
features gastrocnemius/Achilles tendon complex
following rapid acceleration/direction change
is the lowest.  Management of acute AT
• Swelling, tenderness 2-6 cm proximal to primarily includes activity
Examination
tendon insertion modi cation, cold compresses/
ndings
• Rupture: positive Thompson test* icing, and nonsteroidal anti-
• Clinical ndings in ammatory drugs.  For chronic
Diagnosis • Ultrasound: swelling, neovascularization symptoms, a rehabilitation program
• MRI emphasizing eccentric resistance
exercises (ie, lengthening the muscle
• Acute: activity modi cation, ice, NSAIDs
Management
• Chronic: eccentric resistance exercises
while under load) is recommended.

*With the patient prone and feet off the end of the table, squeeze the calf muscles; the absence of plantar exion indicates tendon rupture.

Calcaneal apophysitis (Sever disease) is a common cause of posterior heel pain and
tenderness.  However, it usually occurs in children and adolescents who participate in running or
jumping sports (eg, basketball) because the apophysis is most susceptible to stress during periods of
rapid growth.  Also, the ndings are typically seen within 2 cm of the insertion of the Achilles tendon.
fi
fi
fi
fl
fi
fi
fi
fi

fi

fl


fi

fi
fl

fi

fl

UpToDate: Achilles Tendinopathy

Acute tendon pain generally develops when athletes abruptly increase their training intensity. Chronic
tendon pain (>3 months) may result from sustained stress, poor running mechanics, or improper footwear.
Rupture occurs when a sudden shear stress is applied to an already weakened or degenerative tendon.Pain arising
from the region between the posterior calcaneus and the gastrocnemius-soleus muscle complex may be caused by
a number of conditions. Ankle sprain is the most common misdiagnosis when tendon rupture is the
actual cause of pain. Do not assume rupture is absent because the patient can plantar ex or walk.
Patients with Achilles tendinopathy typically experience pain or stiffness 2 to 6 cm above the posterior
calcaneus. They are likely to be casual or competitive athletes who have recently increased their training
intensity. Pain increases with activity and diminishes with a period of rest. Tendon rupture occurs when
sudden forces are exerted upon the Achilles during strenuous activities that involve sudden pivoting on a foot or
rapid acceleration. Patients may hear a "pop" and experience severe, acute pain, although the absence
of pain does not rule out rupture. Examination should include palpation of the Achilles tendon for
tenderness, thickening, or a defect. Note that edema or a hematoma may mask a defect in the tendon.
Patients with tendinopathy typically have localized tenderness 2 to 6 cm above the insertion point of the
Achilles tendon. Pain at the insertion point suggests calcaneal bursitis or possibly enthesopathy. The
calf squeeze (ie, Thompson compression) test provides an accurate means of detecting complete Achilles tendon
rupture. A sizable minority of patients with complete tendon rupture are able to ambulate. Achilles
tendinopathy (without rupture) is a clinical diagnosis. Imaging is often unnecessary. Plain radiographs are
generally unhelpful. Musculoskeletal ultrasound and magnetic resonance imaging (MRI) are accurate
tests that can be used to aid in the diagnosis of Achilles tendon rupture, particularly if examination
ndings are unclear but clinical suspicion remains high.
All complete tendon ruptures merit surgical consultation. Partial tendon tears and chronic tendinopathy
that fail to improve with three to six months of conservative treatment may bene t from
consultation with a specialized runners' clinic, sports medicine specialist, physical therapist, or
orthopedic surgeon.
Acute Achilles tendinopathy is treated in standard fashion with rest, ice, and tendon support (eg,
heel lift). For chronic midportion tendinopathy (>3 months symptoms), we suggest using a rehabilitation
program that emphasizes resistance exercise using heavy loads. Rehabilitation protocols that
emphasize concentric and eccentric movements or only eccentric movements are both reasonable
approaches.

Enthesitis is an in ammatory disorder characterized by local pain, tenderness, and swelling at


the insertion of a tendon or ligament.  It is a relatively speci c nding for ankylosing
spondylitis and other spondyloarthropathies and is usually associated with back pain and
stiffness.

Subcutaneous calcaneal (super cial to the tendon) bursitis presents with pain and tenderness
at the calcaneal prominence (Achilles tendon insertion point).  It is typically associated with
swelling, warmth, and, often, erythema.  Pain 3 cm proximal to the calcaneus makes Achilles
tendinopathy more likely.
fi

fl


fi

fi
fi
fi
fl

Management of low back pain In patients with chronic LBP, exercise has
proven to be bene cial in reducing pain and
• Maintain moderate activity improving function.  Patients often start with a
Acute pain • NSAIDs or acetaminophen supervised exercise program that
(<4 weeks) • Consider: muscle relaxants, spinal emphasizes stretching and strengthening of
manipulation the back muscles.  Aerobic exercise is also
helpful.  Subsequently, patients can transition to
Subacute
• Intermittent use of NSAIDs or acetaminophen a home exercise program, which should be
pain
• Exercise therapy (stretching/ continued on a long-term basis.  Short courses of
(4-12 wks)
strengthening, aerobic) acetaminophen or NSAIDs can be used
Chronic pain
• Consider: TCAs, duloxetine intermittently.  Some patients also may bene t
(>12 wks)
from tricyclic antidepressants or duloxetine,
Secondary • Exercise therapy but opioids, benzodiazepines, and muscle
prevention • Education relaxants are not advised for routine use.

UpToDate: Exercise based Low back pain Management


The role of exercise- or movement-based therapy in the management of low back pain (LBP) varies according to
the duration of back pain symptoms. For all patients with subacute (4 to 12 weeks) and chronic (more
than 12) LBP, we recommend exercise therapy rather than no exercise.
Exercise therapy improves pain and function in patients with chronic LBP, although a single mechanism by which
exercise improves symptoms is not clear. There are likely several mechanisms contributing to the derived
bene t, including neurologic, musculoskeletal, and psychological.
Counseling patients regarding their expectations is essential prior to recommending and initiating
an exercise- or movement-based therapy program for LBP. In addition, addressing patient fears and
concerns and assessing tness level, interests, prior participation in exercise, and availability of resources are
necessary in formulating an appropriate exercise plan.
As with any movement-based program, there are risks of injury and other adverse events that may occur with
exercise. Patients may be at risk for musculoskeletal injuries to the hips, knees, and ankles. Other risks of exercise,
including rare serious medical events (eg, arrhythmia, sudden cardiac death, and myocardial infarction), are
discussed separately. No single exercise technique has superiority over others for patients with
subacute and chronic LBP. This may be because exercise in all forms has similar generalized effects on the
abnormal neurologic and in ammatory processes associated with subacute and chronic LBP. Examples of exercise
programs that are bene cial for patients with subacute and chronic LBP include:
Walking,Aerobic exercise,Stretching,Pilates,Yoga,Tai Chi,Alexander Technique,Directional preference,Core
exercise/spine stabilization,Graded activities exercise/back boot camp/functional restoration,Multidisciplinary
(interdisciplinary) rehabilitation.Prolonged activity restriction in patients with uncomplicated LBP is associated
with increased long-term pain and stiffness.  Patients should be counseled to maintain normal activity.
Acute LBP (symptom duration less than four weeks) has a good prognosis. Exercise treatment has not been
shown to be more bene cial for acute LBP when compared with other acute treatments. Patients with
acute LBP should be advised to avoid bedrest and stay as active as possible. Patients who do wish to
exercise, however, may continue to perform their usual exercise regimen, as they can tolerate, or try home-based
exercises. Exercise may prevent incident LBP among those with no prior history of back pain as well as recurrent
back pain episodes among those with a history of previous LBP.

fi

fi
fi
fi
fi

fl

fi

Acute lumbosacral radiculopathy (Sciatica)


• Herniated intervertebral disk (most common)
• Degenerative spondylosis
Causes
• Malignancy
• Epidural abscess

• Pain in low back radiating down posterior leg to foot


Signs & • Positive straight-leg raising test
symptoms • Possible dermatomal sensory loss & weakness The diagnosis is made clinically; because nearly
of hip dorsi exion all cases are due to benign etiologies and symptoms
are typically self-limiting, neuroimaging is usually
• Diagnosis based primarily on clinical features not required.  MRI is recommended only for
• MRI recommended for: patients with severe, progressive, or bilateral
Evaluation ◦ Signi cant/progressive or bilateral neurologic de cits (eg, saddle anesthesia, urinary
neurologic de cits retention, severe weakness) or in those with high
◦ Suspected malignancy or epidural abscess concern for malignancy or epidural abscess (eg,
fever, intravenous drug use).  In uncomplicated
• Activity modi cation lumbosacral radiculopathy, initial management
focuses primarily on alleviating symptoms.  NSAID
• Nonsteroidal anti-in ammatory drugs (consider
Management and acetaminophen are the preferred rst-line
epidural/systemic glucocorticoids)
drugs.Activity modi cation is often advisable, but
• Surgery for disabling symptoms patients should maintain moderate physical activity.

MRI can con rm disc herniation or other causes of LSR but is expensive and unlikely to change
management in a patient with uncomplicated LSR (no neurologic de cits or history of
malignancy).  X-ray is considered for nonradicular back pain due to compression fracture or
suspected vertebral metastasis but has limited use in LSR.
Electrodiagnostics (eg, nerve conduction studies) are indicated for patients with persistent
neurologic de cits, especially those who are being considered for surgery or have had previous
spinal surgery.
Physical therapy referral for a supervised exercise program can be considered for patients with
persistent pain lasting more than a few weeks.  However, the effectiveness is variable, and most
patients with acute symptoms experience spontaneous improvement.
Surgical intervention is indicated for patients with rapidly progressive neurologic de cits or
severe, persistent symptoms who have not responded to conservative measures.
fi

fi
fl

fi
fi
fi
fi

fi
fl

fi

fi

fi
Most cases of acute (ie, <4 weeks) low back pain have a benign etiology and spontaneous resolution; imaging generally
does not improve outcomes and is not recommended.  However, spinal imaging is indicated for patients at risk for
infection, malignancy, or bony abnormalities (eg, compression fracture). 

Indications for imaging in low back pain

• Osteoporosis/compression fracture Red ag 🚩 🚩 features that warrant imaging include:


• Suspected malignancy • Sudden onset of pain with spine tenderness
X-ray
• Ankylosing spondylitis (eg, insidious onset,
• History of cancer
nocturnal pain, better with movement)
• Constitutional symptoms (eg, fever,
• Sensory/motor de cits unexplained weight loss)
• Cauda equina syndrome (eg, urine retention, • Trauma
saddle anesthesia) • Signi cant or progressive neurologic de cits
MRI
• Suspected epidural abscess/infection (eg,
(eg, acute bowel/bladder incontinence, lower
fever, intravenous drug abuse, concurrent
extremity weakness)
infection, hemodialysis)
• Elevated risk of spinal infection (eg, recent
Radionuclide bone • Indications for, but patient not able to infection, immunosuppression, intravenous
scan or CT scan have, MRI drug use)

UpToDate: Indications for imaging in low back pain


Most patients who present with back pain to primary care settings will have nonspeci c back pain. Such patients will
typically improve over a few to several weeks with conservative or self-care.
Less than 1 percent will have serious etiologies (eg, malignancy or infection). Less than 10 percent will have less serious,
speci c etiologies (eg, vertebral compression fracture, radiculopathy, or spinal stenosis).
A focused history and physical examination are suf cient to evaluate most patients with back pain of less
than four weeks duration. The history and physical examination should identify features that suggest that imaging and/
or other evaluations are indicated. The majority of patients with low back pain of less than four weeks duration do not
require imaging. Among patients seen in primary care, less than 1 percent will require immediate advanced
imaging (eg, MRI or CT).
Any patient with symptoms of spinal cord or cauda equina compression or progressive and/or severe
neurologic de cits should have immediate lumbar spine MRI for further evaluation and urgent specialist
referral. Such symptoms and signs include new urinary retention, incontinence from bladder over ow, new fecal
incontinence, saddle anesthesia, and signi cant motor de cits not localized to a single nerve root. Other patients who may
require imaging on initial evaluation include those with a moderate to high suspicion for spinal infection, risk
factors for metastatic cancer, and suspected vertebral compression fracture. Patients who have not improved after
four to six weeks of conservative therapy and who did not receive imaging on initial evaluation are reevaluated:
Patients with persistent symptoms due to a lumbosacral radiculopathy or spinal stenosis who are candidates for and are
interested in invasive therapies (eg, surgery or epidural injection for radiculopathy) should have a lumbar spine MRI for
further evaluation.
In patients with low back pain who have risk factors for cancer, we evaluate with erythrocyte sedimentation
rate (ESR) or C-reactive protein (CRP) and plain radiographs.
Other patients that may need imaging include those with concerns for ankylosing spondylitis and osteoarthritis.
In other patients where there are no concerns for a particular etiology, we generally treat with conservative therapy for
another eight weeks.
For patients without concerns for a particular etiology who have not improved after 12 weeks total, we
generally image with a plain radiograph and consider referrals for further evaluation and treatment.
fl
fi
fi

fi

fi

fi
fi

fi
fi

fi

fl

In a patients with features suggesting malignant back pain (nocturnal pain, history of malignancy)
that warrants imaging; Plain- lm x-rays can be performed quickly and can often identify signs of
malignancy.  In ammatory markers (eg, erythrocyte sedimentation rate, C-reactive protein) can
increase sensitivity and are also advised.  If the initial tests are abnormal, MRI is done for
diagnostic con rmation.  MRI without a preceding x-ray is indicated for patients with
signi cant neurologic de cits or cauda equina syndrome (eg, saddle anesthesia, urine retention). 
MRI is also used for patients with suspected spinal infection (eg, epidural abscess, osteomyelitis). 
Different guidelines may recommend slightly different approaches to imaging modalities.

Lumbosacral strain
LSS is generally marked by pain in the
• Strain of paraspinal muscles, tendons, lumbar paravertebral muscles that
intervertebral ligaments does not radiate below the level of the
Causes
• Sudden or unbalanced muscle contraction (eg, knee.  Straight-leg raising testing is
lifting, twisting) usually negative, and lower extremity
• Risk factors: obesity, spinal deformity or
neurologic examination (eg, strength,
degeneration, muscle weakness
re exes, sensation) is typically intact. 
• Pain in lumbar area; may radiate to buttocks, hips,
Most cases improve spontaneously and
Clinical thighs (above knee)
features • Paraspinal tenderness
are treated symptomatically with heat,
• No neurologic de cits; negative straight-leg raising test nonsteroidal anti-in ammatory drugs,
nonbenzodiazepine muscle relaxants,
• Moderate activity
Manage
• Nonsteroidal anti-in ammatory drugs and early mobilization.  Therefore, no
ment
• Nonbenzodiazepine muscle relaxants additional work-up is usually needed

A herniated intervertebral lumbar disc generally causes acute low back pain with radiation into
the foot (lumbosacral radiculopathy); straight-leg raising testing is usually positive, and lower
extremity neurologic de cits (eg, dorsi exion weakness) are often present.
Fat embolism syndrome Nonspeci c ndings (eg, anemia,
thrombocytopenia, lipiduria) may or may
• Fracture of marrow-containing bone (eg, femur) not be present.  FES is a clinical diagnosis,
Etiology • Orthopedic surgery and management is supportive. 
• Pancreatitis Supplemental oxygen is given to correct
• 24-72 hr following inciting event hypoxemia.  Systemic corticosteroids are not
• Clinical triad given routinely but may be considered in
Clinical
◦ Respiratory distress life-threatening cases.  Although death can
presentation
◦ Neurologic dysfunction (eg, confusion) occur, it is rare, and most patients make a
◦ Petechial rash full recovery, often within a few days.
Intubation and mechanical ventilation
Diagnosis • Based on clinical presentation
may be necessary for severe FES cases that
progress to respiratory failure or
Prevention • Early immobilization of fracture
obtundation with inability to protect the
& treatment • Supportive care (eg, mechanical ventilation)
airway. 
fl
fi

fi


fi

fi

fl
fi
fl

fl

fi
fi

fi

fl

Rheumatology/Orthopedics
& SPORTs:

Peadiatrics


BLOCK 58
Custom Id: 228835011, 229367281


Ewing sarcoma
Clinical presentation includes subacute
• Second most common pediatric bony malignancy (after or chronic, localized pain and
Epidemio
osteosarcoma)
logy swelling.  Pain is often worse at night
• White, adolescent boys
and with activity and progresses rapidly. 
• Chronic, localized pain & swelling Local erythema and warmth are
Clinical • Long-bone diaphyses & axial skeleton (eg, pelvis) common, and systemic ndings (eg,
features • ± Systemic ndings (eg, fever, leukocytosis) fever, weight loss, leukocytosis, elevated
• Early metastasis to lung,Bone and Bone Marrow in ammatory markers) may occur,
• Central lytic lesion particularly with metastatic disease.
X-ray • "Onion skinning" (lamellated periosteal reaction) Treatment includes chemotherapy as
ndings • "Moth-eaten" appearance well as surgical resection ± radiation.
• Periosteal elevation (Codman triangle)

Unicameral bone cysts occur in children and young adults and are typically located at the proximal
femur or humerus.  A pathologic fracture is the most common presentation, and x-ray reveals a cystic
lesion with well-de ned margins.
Growing pains Children with growing pains should be followed closely to
monitor for pain that increases in frequency or intensity,
• Occurs primarily at night & resolves by morning which may warrant further evaluation.
Clinical
• Affects bilateral lower extremities (eg, thighs, calves)
features
• Normal physical examination & activity Restless leg syndrome is associated with iron-de ciency
• Parental education & reassurance anemia and causes night-time symptoms but is
Treatment associated an urge to move the legs.
• Massage, stretching exercises, heat & analgesics
fi
fl
fi

fi

fi

fi

Stage Lyme disease clinical features Disseminated infection occurs via hematogenous
spread of infection from the site of the tick bite to
Early localized • Erythema migrans various tissues.Early symptoms of Lyme disease (eg,
(days to 1 • Fatigue, headache erythema migrans, fever, fatigue) are often
month) • Myalgias, arthralgias overlooked or misdiagnosed, making joint
• Multiple erythema migrans swelling the rst apparent manifestation of
Early disease in some patients.  After weeks to
• Unilateral/bilateral CN palsy (eg, CN VII)
disseminated months of infection, the disease can progress to an
• Meningitis
(weeks to
• Carditis (eg, AV block) in ammatory monoarticular or an
months)
• Migratory arthralgias asymmetric oligoarticular arthritis, most
commonly involving the knee.  Patients are
Late • Arthritis typically afebrile and able to bear weight. 
(months to • Encephalitis Effusion is often large and may be associated
years) • Peripheral neuropathy with pain or stiffness.

Synovial uid analysis in Lyme arthritis shows in ammation with an average leukocyte count of
25,000/mm3; Gram stain and culture are usually negative.  Detection of Lyme antibodies in the
serum (enzyme-linked immunosorbent assay [ELISA] and Western blot testing) con rms the
diagnosis.  Oral doxycycline is the rst-line treatment.
Reactive arthritis presents 1-4 weeks after an enteric or Chlamydia trachomatis infection.  Chronic,
asymmetric arthritis occurs, typically with urethritis and eye involvement (eg, conjunctivitis, uveitis).
Disseminated gonococcal infection
• Purulent monoarthritis
Manifestations     OR
• Triad of tenosynovitis, dermatitis, migratory polyarthralgia

• Detection of Neisseria gonorrhoeae in urine, cervical, or urethral sample


Diagnosis
• Culture of blood, synovial uid (less sensitive)

Treatment • 3rd-generation cephalosporin intravenously

Although it is unclear why disseminated Neisseria gonorrhoeae infection causes 2 distinct systemic
syndromes, a minority of patients with DGI have overlapping manifestations; this is particularly common
when the condition is untreated for long periods.N gonorrhoeae is transmitted by unprotected
sexual intercourse.  It typically causes infectious urethritis (eg, purulent discharge, dysuria), but
asymptomatic infections are common, particularly in women.  Untreated urogenital infection can spread
through the blood, leading to DGI in 0.5%-3% of cases. Patients with suspected DGI who have joint effusion
require synovial uid sampling.  In contrast to most other forms of bacterial arthritis, DGI is marked by
synovial uid leukocyte count <50,000/mm3.  In addition, synovial uid culture is usually negative
because N gonorrhoeae is a fastidious intracellular organism that does not grow well on standard media.  As
such, DGI is generally diagnosed presumptively based on clinical symptoms and a positive
urogenital nucleic acid ampli cation test (urogenital assays are usually positive even when no
urogenital symptoms are present).  Treatment with intravenous ceftriaxone is curative in most cases.
fl

fl

fl


fi

fl

fl

fi

fi
fl

fl

fi

Juvenile idiopathic arthritis
Subtype Frequency Age of onset Clinical features Sex ratio

• Arthritis in ≥1 joint for ≥6 weeks


• Quotidian fever for ≥2 weeks
Systemic 10% Age <18 • Evanescent rash F=M
• Hepatosplenomegaly
• Lymphadenopathy

Age 2-5 • Arthritis in ≥5 joints


Polyarticular 40% F>M
or 10-14 • May be complicated by uveitis

• Arthritis in <5 joints


Oligoarticular 50% Age 2-4 F>M
• May be complicated by uveitis

Juvenile idiopathic arthritis (JIA), a chronic autoin ammatory condition of childhood. 


Oligoarticular JIA, the most common form, is characterized by involvement of ≤4 joints (ie, <5
joints) within the rst 6 months of disease onset.
A limp that is worse in the morning in a toddler girl (age <5), is a typical presentation of
oligoarticular JIA.  Joint swelling is present, but pain may be minimal.  The large joints (eg, knees,
ankles) are most commonly affected; however, hip involvement is rare.  Systemic symptoms (eg,
fever, rash) do not occur.Uveitis (ie, intraocular in ammation) is a serious complication that
develops in a minority of patients with oligoarticular JIA.  Ophthalmologic screening examinations
are performed regularly because untreated uveitis is often asymptomatic and can lead to
irreversible vision loss.  Although not present in all patients with oligoarticular JIA, antinuclear
antibodies are a risk factor for uveitis.  Additional laboratory ndings (eg, leukocyte count,
hemoglobin, erythrocyte sedimentation rate) are typically normal in oligoarticular JIA.Treatment
is aimed at relieving in ammation and includes nonsteroidal anti-in ammatory drugs,
intraarticular glucocorticoids, and disease-modifying antirheumatic drugs.
Systemic JIA can present any time during childhood and is characterized by:
• Arthritis in ≥1 joint for ≥6 weeks:  Commonly affected joints include the hips, knees, and
hands.  Patients may have arthralgias prior to the onset of arthritis.
• Fever for ≥2 weeks:  Fever occurs in a quotidian pattern in which high temperatures spike
once daily (often in the evening) followed by spontaneous return to normothermia.  An
evanescent pink rash often accompanies the fever.
Hepatosplenomegaly and lymphadenopathy are also common. Supportive laboratory ndings
re ect in ammation, including leukocytosis, elevated in ammatory markers, thrombocytosis,
and hyperferritinemia.  Anemia due to chronic in ammation and iron de ciency is also seen. 
Rheumatoid factor and antinuclear antibodies are typically negati
fl

fl

fi

fl

fl
fl
fl
fl
fi

fl

fi

fi

You might also like